Printable list of all pharmacology and toxicology SAQs

[Click here to toggle visibility of the answers]

[Click here to toggle printing every question on a separate page]

Question 6 - 2000, Paper 1

List the potential complications associated with the management of a patient after intentional corrosive ingestion.

College Answer

Potential complications of intentional corrosive ingestion include: 
•  Acute:Oral, oesophageal, gastric bums of varying thickness Laryngeal oedema and airway obstruction Oesophageal, gastric perforation 
Shock Haemorrhage Mediastinitis Psychiatric problems 
•  Chronic/late: 
Laryngopharyngo fibrosis with airway incompetence and chronic aspiration 
Oesophageal fibrosis, stricture and stenosis 
Psychosocial problems 
Carcinoma

Discussion

This question would benefit from a systematic response.

  • Airway:
    • Airway burns, leading to airway compromise
    • Potential acute tracheo-oesophageal fistula due to corrosive effect on oesophagus
    • Assessment and immediate airway control is a priority
  • Breathing:
    • Potential aspiration of caustic gastric/oesophageal contents, thus acute lung injury
    • Hypoxia may be present; supplemental oxygen may be required. NIV may be contraindicated in case of full-thickness oesophageal injury
  • Circulation:
    • Potential hypovolemic shock due to fluid loss into the corroded gut, or haemorrhage though ulcers
    • Need for rapid fluid replacement or surgical haemostasis
    • CVC access, as this patient is likely to require long-term TPN
  • Neurological state:
    • Potential for disorganised behaviour due to psychiatric condition, or obtundation due to shock
    • Analgesia issues need to be addressed
  • Electrolyte disturbance
    • Absorption of corrosive agent may result in electrolyte and acid-base disturbance
  • Fluid balance
    • Likely, hypovolemia will exist and need correction
    • renal impairment may be present, with implications on drug dosing
  • Gastrointestinal problems:
    • Extent of corrosive damage will need to be assessed by CT and/or direct endoscopy (earlier is better, before significant tissue softenting makes endoscopy risky)
    • Perforation of hollow organs must be ruled out with CXR and/or CT
  • Specific issues
    • Decontamination by NG aspiration may be possible if it is safe to pass an NGT

References

Ramasamy, Kovil, and Vivek V. Gumaste. "Corrosive ingestion in adults." Journal of clinical gastroenterology 37.2 (2003): 119-124.

Question 8 - 2000, Paper 2

Compare and contrast the pharmacodynamics of dopamine and dobutamine.

College Answer

Pharmacodynamics  imply  what  the  drug  does  to  the  body.  Consideration  should  be given  to mechanism of action, effects on various organs, relationship of dose to effect. indications  for use, and type of adverse effects. These drugs are essential parts of the intensivist's armamentarium, and a good level of understanding should have been displayed.

Dopamine:
•  Immediate  precursor  of  noradrenaline,  and  also  serves  as  a  neurotransmitter  in central  and peripheral nervous systems,
•  Doses of Q.5 to 2 meg/kg/min  (via stimulation  of DA-1  and DA-2  receptors)  increase  renal blood flow, urine flow and sodium excretion (inhibit  sodium resorption in proximal tubules)
•  Haemodynamic  effects are due to noradrenaline release (up to 500/o) and direct stimulation  of alpha,  beta  and  noradrenergic   receptors.  Can  lose  effect  with  time  (due  to  depletion  of noradrenaline stores in periphery and heart).
•  Doses of 2-5 meg/kg/min increase cardiac contractility and cardiac output with minimal change in heart rate/BP/SVR. Increasing dose up to 10 meg/kg/min increase CO/HR. and BP.
•  Doses above 10 meg/kg/min result in increasing alpha adrenergic mediated vasoconstriction.
•  Can increase intrapulmonary shunt (increase CO), but pulmonary vasoconstriction can occur.
•  Dopamine also stimulates  receptors in the zona glomerulosa  of the adrenal cortex to decrease aldosterone secretion.
•  A selective  increase  in  renal  and  splanchnic  blood  flow  occurs,  and  low  doses  have  been thought to prevent the vasoconstrictive effects of other agents. The clinical significance of these effects are controversial (?harm to GIT via shunting from mucosa).
•  Dopamine  inhibits  TSH  and  prolactin  release  as  well as other  potential  negative  effects  on anterior pituitary function.
•  Other  side  effects  include  nausea/emesis,  tachyarrhythm.ias (particularly  AF),  anginal pain, profound  vasoconstriction  (including  if local  extravasation  (treat  with  phentolamine]),  and impairment of hypoxic ventilatory drive.
•  Used to increase cardiac output and as a mild vasopressor (cardiogenic or septic shock), and as a diuretic (no evidence to support renal protective role).

Dobutamine:
•  Racemic  mixture  of  + and  - isomers.  + isomer  stimulates  both  beta-adrenergic  receptors. - isomer is potent selective  alpha-1-ad.renergic agonist   No indirect stimulation  of receptors. Metabolite (3-0-methlydobutamine) is potent inhibitor of alpha receptors. Net effect is balance between various receptor effects.

•  At commonly used dose ranges (2-15 meg/kg/min) increases contractility, with little effect on HR at doses < I0  meg/kg/min. Usually little effect on SVR and PVR as balance between alpha- 1 and beta-2 effects (CVP and PAWP usually decrease). Some tolerance with time but less than with dopamine.
•  Enhances urine  output  by  increasing cardiac output. No  other  significant  metabolic or endocrine effects.
•  Side  effects  include  dysrhythmias  (less  than dopamine),  tachycardia,  headaches,  anxiety, tremors, changes in BP. •
•  Used to increase cardiac output without need to effect peripheral resistance (cardiogenic or septic shock) or desire to have metabolic/endocrine effects of dopamine. Also used to assess for myocardial ischaemia (stress test).

Discussion

Though it may be tempting to unload a massive amount of pharmacological knowledge onto such a question, the candidate is reminded that these days we only have 10 minutes per question.

Features

Dobutamine

Dopamine

Class

Synthetic catecholamine

Endogenous catecholamine

Pharmacokinetics

IV infusion 5-15mcg/kg/min Rapidly metabolised by COMT; 
Half-life ~ 5 minutes
No active metabolites

Half-life 2-3minutes
Metabolised by MAO and COMT

Receptor activity

Predominantly a beta-1 receptor agonist; other isomer is a partial alpha-1 agonist

Predominantly beta-1 receptor agonist at low doses, with more alpha-effects  as dose escalates
D-1 receptor agonist at low doses

Mechanism

increases heart rate and contractility by increasing the intracellular levels of cAMP, thus increasing the availablility of intracellular calcium.

increases heart rate and contractility by increasing the intracellular levels of cAMP, thus increasing the availablility of intracellular calcium.

Adverse effects

Increased inotoropy
Increased chronotropy
Peripheral vasodilation (beta-2 effect of one of the enantimers)
Arrhythmia
Hypotension
Increased cardiac metabolic demand, thus potentially exacerbating ischaemia

Arrhythmogenic at the high doses required for treatment of severe sepsis

Increased cardiac oxygen demand due to increased contractility and heart rate may cause ischaemic phenomena

No evidence for any renal protective effects

References

Question 3 - 2001, Paper 2

List the relevant pharmacology of the following drugs when used in ICU to aid the dressing of severe burns: (a) tramadol,  (b) celecoxib and (c) ketamine.

College Answer

Tramadol is a synthetic non-narcotic analgesic with opioid like effects. It acts centrally to bind with mu receptors and also blocks noradrenaline and serotonin uptake. It is rapidly absorbed orally with high bioavailability. It is cleared by hepatic metabolism and may produce dizziness, somnolence, nausea, constipation, sweating and pruritus similar to opioids, but causes significantly less respiratory depression than morphine.

After an IMI dose, peak effect is achieved in 45 minutes and lasts 4-5 hours. Convulsions and rare anaphylactoid reactions have been described with its use. Overdosage may produce respiratory failure and seizures. Its role in this setting is unclear as yet because of low potency but it may be useful as an adjunct.

Celecoxib is a COX-2 inhibitor and as such has anti-inflammatory, analgesic and anti-pyretic properties. In the absence of COX-1 inhibition, it should have no/little effect on gastrointestinal mucosa or platelet function. Disruption of renal blood flow autoregulation in hypovolaemia and shock is still possible.

NSAIDS have been used in burns to reduce the inflammatory response, but have an uncertain role in dressings due to slow onset (1 hour), low potency, oral preparation and untoward renal effect. Duration of action is 6 – 15 hours. They should not be used in patients with sulfonamide allergy or aspirin/NSAID associated asthma.

Ketamine is a general anaesthetic agent related to the hallucinogen phencyclidine which can be given IV or IM. Despite the tendency to emergence delirium it is a useful agent in this setting because of intense analgesia with maintenance of reflexes and minimal respiratory depression. Duration of action is 2 – 4 hours and it undergoes extensive hepatic metabolism. Dreams and hallucination can be reduced by the concomitant administration of a benzodiazapine.

Discussion

Features

Tramadol

Celecoxib

Ketamine

Class/mechanism

Partial opioid receptor agonist;
Serotonin receptor agonist

NSAID (selctive for COX-2)

Anaesthetic;
NMDA receptor antagonist

Advantages

Synergistic with other analgesics

Less respiratory depression than with other opiates

Low toxicity
Synergy with opioids

Dissociative sedation
Opioid-sparing effect

Disadvantages

Weak opioid effect
Lowers seziure threshold
Slow onset
Interacts with SSRIs

May cause renal impairment
May cause platelet dysfunction
Slow onset
Weak analgesic

Confusion, delirium
Emergeance phenomena
Requires speialist staff to be present

Some mixture of ketamine, tramadol and dexmedetomidine may be the best option.

References

 

Zor, Fatih, et al. "Pain relief during dressing changes of major adult burns: ideal analgesic combination with ketamine." Burns 36.4 (2010): 501-505.

 

Norman, Aidan T., and Keith C. Judkins. "Pain in the patient with burns." Continuing Education in Anaesthesia, Critical Care & Pain 4.2 (2004): 57-61.

 

Power, Camillus Kevin. "Burns Injury Pain Management-the evidence or not!."Official publication of the National Academy of Burns-India (2009).

Question 7 - 2001, Paper 2

What drug withdrawal states are relevant to ICU practice?   Outline the principles of their management.

College Answer

Drug withdrawal states in ICU patients may be more common than is generally appreciated. They include –

•    Alcohol

•    Tobacco (nicotine)

•    Narcotic (heroin, morphine)

•    Benzodiazepines

•    Caffeine

•    Other street drugs (cocaine etc)

Principles of their management include –

•    prevention (avoid prolonged high dose narcotics, benzodiazepines

•    detection/diagnosis (be alert for signs eg agitation, tachycardia, fever)

•    sedation (may be necessary to control systemic effects)

•    replacement/substitution (eg nicotine patch)

•    support (airway and respiration, fluid replacement)

•    simple measures such as but firm communication, reality orientation, visible clock and presence of a relative contribute to reassurance of the patient.

Discussion

The following withdrawal syndromes seem relevant:

Withdrawal Syndromes organised by Neurotransmitter  System
Receptor Drugs Clinical withdrawal syndrome Management options
GABAA Alcohol
Barbiturates
Benzodiazepines
Organic solvents
CNS excitation (agitation, tremor, hallucinations, seizures) 
Autonomic stimulation (tachycardia, hypertension, hyperthermia, diaphoresis)
Benzodiazepines
Dexmedetomidine
GABAB GHB
Baclofen
Dyskinesia, seizures, hypertension, hallucinations, psychosis, and coma. Benzodiazepines
Opioid Opiates CNS excitation (agitation, tremor, hallucinations)
Diarrhoea, mydriasis, nausea.
Autonomic stimulation (tachycardia, hypertension, hyperthermia, diaphoresis)
Clonidine
Dexmedetomidine
Adenosine Caffeine Head-ache (cerebral vasodilation), fatigue, and hypersomnia (motor inhibition) -
Nicotinic acetylcholine
receptor
Nicotine Agitation, insomnia, poor concentration, poor gut motility, poor feed tolerance. Varenicline?
Noradrenenaline Amphetamines Agitation, dysphoria, somnolence -
Dopamine Cocaine Anhedonia, irritability, exhaustion -
Cannabis   Agitation, insomnia, poor gut motility Mirtazapine ?

The college presents an excellent summary of the generic principles of managing drug withdrawal:

  •    prevention
  •    detection/diagnosis
  •    sedation
  •    replacement

In greater detail:

Prevention

In this context, "prevention" is not some sort of grassroots social work movement to gets the kids off their street drugs, but rather the push towards intelligent use of opiates and benzodiazepines in the ICU. Rationalising the infusions should prevent the development of iatrogenic withdrawal syndromes. Fortunately, the ICU environment typically does not favour true psychological addiction, as the pleasurable context of drug use is not present.

Detection

In this context, detection describes vigilant monitoring for drug withdrawal:

  • History (i.e. discussing drug use with the family)
  • Examination (looking for features suggestive of drug use, eg. track marks)
  • Biochemistry (eg. the pre-intubation urine drug screen)
  • Index of suspicion (keeping drug withdrawal in the list of differentials when assessing a patient with tachycardia, delirium, fever, or failure to wake)

Supportive management

The supportive management of drug withdrawal aims to reduce the harm from the physiological and psychological consequences of withdrawal:

  • Sedation (for comfort)
  • Analgesia (to combat post-opioid hyperalgesia)
  • Control of physiological derangements (eg. clonidine to block the sympathetic storm of opiate withdrawal)
  • Protection of the CNS from seizures (i.e. in benzodiazepine and alcohol withdrawal)

Replacement and substitution

The aim is to replace the drug of addiction with a less harmful substance which offers submaximal receptor stimulation, so that the symptoms of withdrawl are ameliorated and the harm of pursuing the addiction is reduced. Examples of this include methadone and varenicline.

References

Jenkins, Donald H. "Substance abuse and withdrawal in the intensive care unit: Contemporary issues." Surgical Clinics of North America 80.3 (2000): 1033-1053.
 
Kosten, Thomas R., and Patrick G. O'Connor. "Management of drug and alcohol withdrawal." New England Journal of Medicine 348.18 (2003): 1786-1795.
 
Tetrault, Jeanette M., and Patrick G. O'Connor. "Substance abuse and withdrawal in the critical care setting." Critical care clinics 24.4 (2008): 767-788.

Question 5 - 2002, Paper 1

Outline the role of decontamination of the digestive tract in the management of patients who present with a drug overdose.

College Answer

Balance between potential severity of poisoning, time from ingestion and risk to the patient of interventions considered.  Most overdoses do not develop significant toxicity but reasonable to use technique with low morbidity and reasonable efficacy in all except clearly non-toxic ingestions (eg. single dose activated charcoal [1g/kg]).  Induced emesis with ipecac induces risks without evidence of decreased absorption.   Gastric lavage is associated with reasonable decrease in absorption if performed  early (e.g. < 1 hour), though it is associated  with increased  risks (including  visceralinjury and aspiration); it may have additional benefit if combined with activated charcoal.  Repeat doses of charcoal are usually not of additional benefit except perhaps where a large amount of toxic substance adsorbed by charcoal was ingested (especially slow release preparations).   Whole bowel irrigation (using polyethylene glycol e.g. golytely) may have specific benefit with slow release preparations or agents that are poorly absorbed by activated charcoal.  Rarely endoscopy or surgical removal is indicated. 

Discussion

This question closely resembles section (b) from Question 1 of the second paper of 2004. However, here it is presented on its own, as a 10-mark question, and so some extra thought should be spent on it.

In brief, decontamination can be critically evaluated in the following manner:

Rationale for decontamination

  • In any overdose, especially early, there is some proportion of the ingested drug which still has not absorbed.
  • This unabsorbed drug could potentially be cleared from the gut
  • This would result in a reduced total dose of the drug
  • The reduced total dose should also result in a reduced total toxicity
  • Ergo, the removal of undissolved drugs should reduce the toxicity of the overdose

Techniques of decontamination and their indications

  • Activated charcoal, single or multiple doses
  • Induced emesis (abandoned)
  • Gastric lavage (largely abandoned; only indicated within the first hour)
  • Whole bowel irrigation (only indicated for iron and slow release enteric coated tablets)
  • Surface decontamination for skin-absorbed toxins

Situations which merit the use of gut decontamination

  • The overdose is recent (within the last hour)
  • There is reason to believe a large number of undissolved tablets is still present in the stomach or gut
  • There is no adequate antidote to the drug, and the overdose is lifethreatening

Criticsm of gut decontamination techniques

  • Possibility of aspiration is ever-present, particularly if the airway is unprotected
  • Likelihood of effect diminishes with time.
  • Even charcoal may have serious complications, eg. bowel obstruction
  • Many of the early studies which lauded the effectiveness of gut decontamination techniques such as emesis or lavage were focused on the effectiveness of the emetic in achieving emesis, or in the lavage recovery of some abstract marker substance. No studies focused on patient outcome. Patient outcomes do not seem affected by decontamination techniques.
  • The removal of a proportion of ingested drug may have no effect on the course of the overdose, in terms of outcome. One may think of this in terms of the difference between absorbing 100g of paracetamol vs. only absorbing 75g. In either case, your liver is screwed.

References

The website of the American Academ of Clinical Toxicology has several position statements which might be useful to the fellowship candidate:

Ipecac Syrup

Single-Dose Activated Charcoal

Multi-Dose Activated Charcoal

Cathartics

Whole Bowel Irrigation

Gastric Lavage

Urine Alkalization

Gaudreault, Pierre. "Activated charcoal revisited." Clinical Pediatric Emergency Medicine 6.2 (2005): 76-80.

Andersen, A. Harrestrup. "Experimental Studies on the Pharmacology of Activated Charcoal. III. Adsorption from Gastro‐Intestinal Contents." Acta Pharmacologica et Toxicologica 4.3‐4 (1948): 275-284.

Krenzelok, Edward P. "New developments in the therapy of intoxications." Toxicology letters 127.1 (2002): 299-305.

Eddleston, Michael, et al. "Multiple-dose activated charcoal in acute self-poisoning: a randomised controlled trial." The Lancet 371.9612 (2008): 579-587.

Isbister, Geoffrey K., and Venkata V. Pavan Kumar. "Indications for single-dose activated charcoal administration in acute overdose." Current opinion in critical care 17.4 (2011): 351-357.

Chyka, P. A., and D. Seger. "Position statement: single-dose activated charcoal. American Academy of Clinical Toxicology; European Association of Poisons Centres and Clinical Toxicologists." Journal of toxicology. Clinical toxicology 35.7 (1996): 721-741.

Daly, F. F. S., M. Little, and L. Murray. "A risk assessment based approach to the management of acute poisoning." Emergency medicine journal 23.5 (2006): 396-399.

Olmedo, Ruben, et al. "Is surgical decontamination definitive treatment of “body-packers”?." The American journal of emergency medicine 19.7 (2001): 593-596.

 

Question 9 - 2002, Paper 2

Compare and contrast the pharmacology of lignocaine, magnesium  and amiodarone when used in the treatment of ventricular tachycardia.

College Answer

•    Lignocaine: Class I (membrane stabilising) antiarrhythmic agent. Sodium channel blockades results in decreased action potential duration and shortened refractory period. Rapidly distributed to all body tissues. Approximately 65% protein bound; elimination half-life 1.6 hours (80% metabolised in liver). Adverse effects: lightheaded, hypotension, cardiovascular collapse, heart block, confusion and convulsions. Dosage 1 to 1.5mg/kg with subsequent boluses (up to 3 mg/kg total), followed by infusion (1-4 mg/min, at decreasing dose, up to
24 hours). 

•    Magnesium (as sulphate or chloride): second most abundant intracellular cation. Depresses neuronal activation. Widely distributed, duration of action about 30 minutes. Filtered by kidneys, but most is reabsorbed. Adverse effects include: nausea, flushing, CNS depression, coma, and heart block. Dose 5 mmol bolus (which may be repeated), followed by infusion
of 20 mmol over 4 hours.


•    Amiodarone: Class III antiarrhythmic. Prolongs action potential duration, and prolongs refractory period of atrial, nodal and ventricular tissues. Highly protein bound with very high apparent volume of distribution (6 L/kg); accumulates in adipose tissue and highly perfused organs. Half-life (with chronic dosing) is 14 to 59 days, mainly excreted via the liver and bile. Adverse effects: hypotension/circulatory collapse, bradycardia, sinus arrest, nausea and flushing. Torsades de pointes can be induced. Hyper- or hypo-thyroidism can be
induced. Multiple other potential organ dysfunctions with more chronic use (including some potentially fatal). Dosage 5 mg/kg which can be repeated, and followed by an infusion (15 mg/kg/hr).

Discussion

This question is identical to Question 12 from the first paper of 2006.

References

Question 7 - 2003, Paper 1

Compare and  contrast the pharmacology  of propofol, midazolam  and  thiopentone when used by infusion for the treatment of raised intra-cranial pressure.

College Answer

Pharmacology includes pharmaceutics (including preparation), pharmacokinetics (including distribution, elimination and  biotransformation) and  pharmacodynamics (including dose, mechanism of action, effect of various disease states, adverse effects and interactions).  Effects of all drugs are augmented by other CNS depressant drugs.

Propofol is formulated as a white isotonic aqueous emulsion (containing soya oil and egg lecithin) at a concentration of 10 mg/mL.  It supports the growth of bacteria if accidentally introduced so syringes/bottles should be changed every 12 hours.   It is widely and rapidly distributed (98% protein bound), with an initial half life of redistribution of 2 to 8 minutes, but a terminal elimination of 3 to 20 hours (which may influence waking time after prolonged infusion [many days]: context sensitive half-time).  Inactive metabolites are renally excreted.  Usual ICU sedation dose is infusion of 1 to 3 mg/kg/hr (higher infusion rates in Intensive Care have been associated with rhabdomyosysis). Mechanism of action is not clear. Lipid formulation of approx. 1 kcal/mL should be taken into account, and high triglyceride levels may be seen in susceptible patients. Hypotension and depression of cardiac output may occur (more so when bolus doses are used).  Compatible with
5% dextrose but not with many other solutions or drugs. Time to wake after cessation of infusion is short (minutes to hours) depending on duration of infusion (context sensitive half time).   Rapid awakening may increase likelihood of convulsions in those susceptible. Relatively expensive.

Midazolam is formulated as a colourless isotonic but acidic solution (pH 3.3) as 1 or 5 mg/mL. Onset usually seen within minutes (97% protein bound), and usual elimination half-life quoted at 1 to 3 hours, but often seen 6 times longer in critically ill especially elderly or renal failure Metabolised by P450-3A to active metabolite 1-OH-methyl midazolam, and then renally excreted. Usual ICU dosage 0.03 to 0.2 mg/kg/hr.  Mechanism of action is via activation of benzodiazepine receptor, which augments the inhibitory effect of the GABA receptor.  Cardiorespiratory depression is expected.  Rapid cessation may lead to withdrawal.  Compatible with many solutions and drugs (except hartmanns), infusions should be discarded after 24 hours.  Time to wake after cessation of infusion is intermediate (hours to days) depending on duration of infusion and presence of renal or hepatic dysfunction.

Thiopentone is prepared from a powder and dissolved in water giving an alkaline solution with a final concentration of 25 mg/mL.  Onset of action within minutes (80% protein bound, with very large volume of distribution), usual elimination half life quoted at 3 to 8 hours (but presumably longer after prolonged infusion).  Metabolised predominantly in the liver to (?) inactive metabolites which are renally excreted.  Usual ICU dosage is 25 to 100 mg/hour (0.5 to 1.5 mg/kg/hr), with boluses of 25 to 100 mg as required.  Very effective CNS depressant (including resultant isoelectric EEG and fixed dilated pupils).   Incompatible with many solutions (especially if acidic), and infusions should be discarded after 24 hours.  Time to awakening after cessation of infusion is delayed (up to many days), depending on duration of infusion and the presence of hepatic dysfunction.  Barbiturates may precipitate acute porphyria in susceptible patients by enhancing porphyrin synthesis.

Discussion

Features

Propofol

Midazolam

Thiopentone

Class

General anaesthetic phenol

Benzodiazepine

barbiturate

Pharmacokinetics

Half-life 2-3minutes
Metabolised by the liver; inactive metabolites.

Extensive distribution into fat

Half-life 90 minutes;
Metabolised by the liver;
Active metabolites are renally excreted
Extensive distribution into fat

Half life ~ 9 hours; rapid redistribution decreases the duration of effect.

Extensive distribution into fat

Receptor activity

Thought to be a GABA agonist;
Increases inhibitory neurotransmission

Allosteric modulator of GABA neurotransmission; increases the rate of opening of GABA channels

Allosteric modulator of GABA neurotransmission; increases the duration of opening of GABA channels

Advantages in head injury

Short half life

Decreases cerebral metabolic demand, thus decreasing ICP

Decreases seziure activity

Decreases cerebral metabolic demand, thus decreasing ICP

Decreases seziure activity

Decreases cerebral metabolic demand, thus decreasing ICP

Decreases seziure activity (very potent antiepileptic)

Well tolerated in hemodynamic instability

Disadvantages in head injury

Hypotension
Hyperlipidaemia 
Propofol infusion syndrome

Slow waking; context-sensitive half-life becomes prolonged

Sudden withdrawal may precipitate seziures

Prolonged effect in renal or hepatic impairment

Very slow waking; as drug redistributes from tissue compartment

May precipitate acute porphyria

References

Question 7 - 2003, Paper 2

Outline the diagnostic features, complications and treatment of patients with an overdose of sodium valproate (valproic acid).

College Answer

Sodium valproate is becoming more widely used (seizures, bipolar disorders, migraine), and is often prescribed as a slow release preparation.  Overdose results in a progressive onset of lethargy and CNS depression, with many potential associated features (including hypotension, hypothermia, vomiting,  diarrhoea,  agitation  and  tremors).     Complications  include  cerebral  oedema  (with prolonged coma), encephalopathy (elevated ammonia), hepatotoxicity (rarely fulminant), and electrolyte disorders (with hypernatraemia, hypocalcaemia, increased osmolality and elevated anion gap metabolic acidosis).  Treatment is generally supportive but gastrointestinal decontamination is essential (including multiple dose activated charcoal &/or whole bowel irrigation if  sustained release preparations, and increasing valproic acid levels).  Carnitine supplementation may attenuate hepatotoxicity and hyper-ammonaemia.

Discussion

This is hard, because there are no characteristic clinical features in this overdose. There is non-specific lethargy which progresses to drowsiness and coma. Then, the LFTs come back deranged, and when you do the ammonium level it is through then roof, which makes you think.

Pathophysiology

  • Sodium valproate is a simple branched-chain carboxylic acid, the antiepileptic properties of which were discovered quite by accident (it was the solvent used to dissolve water-insoluble bismuth salts during a series of preclinical animal experiments in 1962)
  • Mitochondrial β-oxidation of valproate  involves "the carnitine shuttle", which leads to the depletion of carnitine.
  • In the absence of carnitine, the liver resorts to an altertative metabolic pathway, which produces 4-en-valproic acid, a hepatotoxin.
  • Hyperammonaemia develops, as valproate promotes the transport of glutamine through the mitochondrial membrane, and ammonia is released as a result of the mitochondrial metabolism of glutamine into glutamate.
  • The hyperammonaemia then gives rise to cerebral oedema, some 72 hours post ingestion.

Diagnostic features

  • hypotension
  • hypothermia
  • CNS depression
  • tremor

Complications of valproate overdose

  • Lactic acidosis
  • hyperammonaemia and encephalopathy
  • acute hepatic failure
  • pancreatitis
  • cerebral oedema
  • hypernatremia
  • Hypocalcemia
  • Hypocarnitinemia, if you actually test for carnitine
  • Bone marrow suppression

Drug levels

  • 50-100mg/L = therapeutic
  • 100-450mg/L = mild toxicity (drowsy, confused)
  • 450-1000mg/L = severe toxicity (usually comatose)
  • Over 1000mg/L = indication for dialysis

Treatment

  • Supportive management (ventilation, vasopressors, etc)
  • gastrointestinal decontamination with charcoal or whole bowel lavage
  • L-Carnitine supplementation: the loading dose is 100 mg/kg IV over 30 minutes (maximum 6 g) followed by 15 mg/kg IV over 10–30 minutes every 4 hours until clinical improvement occurs. At least some of the acute ammonia-induced encephalopathy seems to be due to a carnitine deficiency, as valproate metabolism depletes the stores of carnitine. Replacement of carnitine seems to be the unquestioned dogma in valproate overdose. Lheureux et al (2005) examined the evidence behind this practice, and found that usefulness in overdose probably does not justify routine supplementation.
  • Valproate is 90% protein bound and therefore poorly cleared by dialysis, but ammonia is, and therefore haemodialysis is indicated to prevent cerebral oedema. Moreover, the protein binding is saturable, and in gross overdose you can still bring the valproate levels down significantly with high intensity haemodialysis. Bellomo et al (2009) found that clinical improvement was more rapid with this strategy than with supportive care alone.

One report linked below is an account of a truly massive (25g) valproate overdose, which did not require anything but supportive management, and which was not accompanied by any sort of massive organ system failure.

References

Isbister, Geoffrey K., et al. "Valproate overdose: a comparative cohort study of self poisonings." British Journal of clinical pharmacology 55.4 (2003): 398-404.

Lakhani, Mayur, and M. E. McMurdo. "Survival after severe self poisoning with sodium valproate." Postgraduate medical journal 62.727 (1986): 409-410.

Löscher, Wolfgang. "The discovery of valproate." Valproate. Birkhäuser Basel, 1999. 1-3.

Licari, Elisa, et al. "Life-threatening sodium valproate overdose: A comparison of two approaches to treatment*." Critical care medicine 37.12 (2009): 3161-3164.

Lheureux, Philippe ER, et al. "Science review: Carnitine in the treatment of valproic acid-induced toxicity–what is the evidence?." Critical Care 9.5 (2005): 431.

Question 9 - 2003, Paper 2

Compare and contrast the pharmacology of ceftriaxone, gentamicin and meropenem.

College Answer

Ceftriaxone: vial with yellow water soluble powder for reconstitution; only administered parenterally, 33-66% excreted unchanged in urine, no active metabolites, 85-95% protein bound, elimination half life 6-9 hours (> 36 hours with severely impaired renal function), usual dosage 0.5 to 2g IV 12 or 24 hourly; 3rd generation cephalosporin antibiotic, inhibits cell wall synthesis, covers most gram negative rods (except Pseudomonas), and Gram positive cocci (except Methicillin Resistant, and group D streptococci); adverse reactions uncommon, but include overgrowth of non- susceptible organisms, and occasional haematologic, renal and hepatic adverse effects.

Gentamicin: ampoule with 80 mg/2 mL; only administered parenterally, excreted almost entirely by glomerular filtration, elimination half life 2-3 hours, no active metabolites, usual dosage 1 mg/kg tds or up to 5 mg/kg as daily dose, careful monitoring of blood levels required, especially if renal impairment (trough level not > 2 mcg/mL); aminoglycoside antibiotic, inhibits protein synthesis, covers most gram negative rods (including pseudomonas, but variability from hospital to hospital); serious adverse reactions include oto- and renal toxicity, potentiated by other oto- and nephro- toxins, prolongation of neuromuscular blockade may occur, other reactions uncommon.

Meropenem: vial with water soluble powder for reconstitution; only administered parenterally, 70% excreted unchanged in urine (requiring reduction of dosage if significant renal impairment), plasma binding 2%, elimination half life 1 hour, no active metabolites, usual dosage 500mg to 2g every 8hours; carbapenem antibiotic, inhibits cell wall synthesis, active against a broad spectrum of aerobic and anaerobic bacteria (including Gram positive cocci and Gram negative rods, but excluding MRSA, Enterococcus faecium, Sternotrophomonas and many Pseudomonas); serious adverse reactions are rare, but include overgrowth of non-susceptible organisms, and occasional haematologic, gastrointestinal and hepatic adverse effects.

Discussion

This question is identical to Question 3 from the second paper of 2006.

References

Question 10 - 2003, Paper 2

Outline the diagnostic features, complications and treatment of patients with malignant hyperpyrexia.

College Answer

Malignant hyperpyrexia is a rare genetic disorder, usually autosomal dominant inheritance, with mutations of the calcium channel (ryanodine) found in the sarcoplasmic reticulum of skeletal muscle.  When triggered by drugs (esp. suxamethonium and volatile anaesthetic agents), usually within 1 hour, uncontrolled calcium efflux results in tetany, and markedly increased skeletal muscle metabolism. 

 Diagnostic features include susceptible patient (may be unknown), exposed to triggering agent, with signs of increased metabolic rate (early tachycardia, increased muscle tone, increased oxygen consumption, increased CO2 production [e.g. ETCO2], and later marked hyperthermia).    Complications include rhabdomyolyis, shock, disseminated intravascular coagulation, and a mixed metabolic (lactic) and respiratory acidosis.  

The mainstay of treatment is the removal of triggering agents and administration of the specific antidote (dantrolene 20 mg/vial, diluted to 60 mL with water, dosage e.g. 2 mg/kg every 5 minutes up to 10 mg/kg, repeated every
10 to 15 hours, and continued for three days).  Other treatment is supportive initially with active cooling, and detection and treatment of the potential complications listed above. Confirmation of diagnosis (muscle biopsy) and family screening may be necessary.

Discussion

The European Malignant Hyperthermia Group has published some nice guidelines in 2010, which offer an excellent overview of this topic.

General features

  • Follows suxamethonium or volatile agent administration
  • Develops during anaesthesia
  • Body temperature rises by 1 degree every 10 minutes

Clinical features

  • Hyperthermia
  • Jaw rigidity persists after sux has worn off
  • Tachycardia and tachypnoea
  • Increased EtCO2
  • Increased O2 consumption
  • Profuse sweating
  • Hyperkalemia
  • Cyanosis
  • Generalised rigidity, increased muscle tone
  • Prolonged bleeding

Complications

  • DIC
  • Rhabdomyolysis
  • Hypotension
  • Lactic and respiratory acidosis

Management

  • Abort the procedure
  • Stop the anaesthetic
  • Give 100% FiO2 and hyperventilate
  • Start active cooling
  • Administer dantrolene: 20mg as a rapid infusion
  • Keep giving dantrolene until features of resolution begin to manifest
  • Give steroids; eg. 2g of methylprednisolone
  • Maintain high urine output to avoid renal damage from rhabodomyolysis
  • Correct coagulopathy

References

Hopkins, P. M. "Malignant hyperthermia: advances in clinical management and diagnosis." British journal of Anaesthesia 85.1 (2000): 118-128.

 

Glahn, K. P. E., et al. "Recognizing and managing a malignant hyperthermia crisis: guidelines from the European Malignant Hyperthermia Group." British journal of anaesthesia 105.4 (2010): 417-420.

Question 9 - 2004, Paper 1

Compare and contrast the pharmacology of noradrenaline, vasopressin and phenylephrine

College Answer

Noradrenaline is the catecholamine released by postganglionic adrenergic nerves. Direct agonist acting on alpha (vasoconstrictor: arterial and venous) and beta-1 (contractility, pro-arrhythmic) adrenergic receptors. Not absorbed enterally. Rapidly metabolised by COMT and MAO, resulting short (minutes) duration of effect (usually administered as intravenous infusion into central vein at rate of 0.5 to 100 mcg/min). Used clinically to increase blood pressure (usually in the setting of vasodilatory shock).

Vasopressin is a hormone/neurotranmitter with a complex series of effects. Direct action on a number of receptors (V1 (vascular: vasoconstriction), V2 (renal: anti-diuresis), V3 (pituitary), OTR (oxytocin receptor subtypes) and P2 (purinergic). Not absorbed enterally. Rapidly inactivated by trypsin and peptidases, resulting in short (minutes) duration of effect (longer on kidneys as very low concentration  are  required).  Used  clinically  as  treatment  for  diabetes  insipidus  (IM,  IV  or intranasal), and more recently by intravenous infusion (via central vein at rates of 0.01 to 0.1
U/min) to increase blood pressure (usually in the setting of vasodilatory shock) or as a large intravenous bolus providing potent vasoconstriction during cardiac arrest (40 units). Potentiates the action of other vasoconstrictor agents.

Phenylephrine is a synthetic alpha-1 adrenoreceptor agonist, similar in structure to adrenaline. Not administered enterally, biotransformation not well described but duration of action longer than naturally  occurring  catecholamines  (still  minutes).  Used  clinically  for  vasoconstrictor  effects, usually administered intravenously either in small bolus doses or occasionally as an intravenous infusion (via a central vein at rates of 40 to 180 mcg/min). Refractory hypotension may respond to agents with combined alpha-1 & alpha-2 activity (e.g. noradrenaline). Can be administered topically for alpha-adrenergic effect.

Discussion

Of these "compare and contrast" questions, vasopressin noradrenaline adrenaline phenylephrine dobutamine levosimendan and dopamine have all been asked about.

In fact, even this current mixture has cropped up in Question 18 of the second paper from 2005.

References

Question 1c - 2004, Paper 2

You are called to see a 16-year-old girl in the Emergency Department.  She was brought in by ambulance after being found unconscious by her parents.  She was last seen alive and well 12 hours ago.  Several empty bottles of tablets were found beside her.

(c)        What “antidotes” are available for patients after drug overdose?

College Answer

(c)        What “antidotes” are available for patients after drug overdose?

Many antidotes are available but obviously their relevance depends on the clinical scenario and the specifics of the drugs ingested. Specific antidotes for commonly used agents (eg. naloxone for opioids, flumazenil for benzodiazepines, beta-agonists for beta-blockers, Ca for calcium channel blockers, protamine for heparin, atropine for organophosphates, and physostigmine for anticholinergics). Less commonly used specific antidotes include:
digibind for digoxin, and desferrioxamine for iron. Other indirectly acting antidotes include: Fresh Frozen Plasma and Vitamin K for warfarin, N-acetyl cysteine for paracetamol, glucagon for beta- and calcium channel blockers, glucose for insulin, ethanol for methanol, sodium bicarbonate for tricyclic antidepressants and praladoxime for organophosphates.

Discussion

c)

This question closely resembles Question 28.1 from the second paper of 2009, as well as Question 14.2 from the first paper of 2008 and Question 2 from the first paper of 2007. There are so many lists of antidotes available that I see no point in repeating this answer endlessly.

References

The website of the American Academ of Clinical Toxicology has several position statements which might be useful to the fellowship candidate:

 

Ipecac Syrup

Single-Dose Activated Charcoal

Multi-Dose Activated Charcoal

Cathartics

Whole Bowel Irrigation

Gastric Lavage

Urine Alkalization

Question 1d - 2004, Paper 2

You are called to see a 16-year-old girl in the Emergency Department.  She was brought in by ambulance after being found unconscious by her parents.  She was last seen alive and well 12 hours ago.  Several empty bottles of tablets were found beside her.

(d)       Discuss her ongoing (definitive) management.

College Answer

(d)       Discuss her ongoing (definitive) management.

Definitive management of this girl includes specifics related to the drugs involved (eg. antidotes listed above for paracetamol or tricyclic antidepressants; continuation or otherwise of decontamination techniques) or the presence of any intercurrent diseases (eg. rhabdomyolysis). General supportive care would include attention to pressure areas, nutrition, thromboprophylaxis, and nosocomial infections. Specific care would be directed
to parents/relatives, and psychiatric assessment is required early to facilitate appropriate psychiatric management.

Discussion

d)

The specific management of this overdose victim would depend completely on the drug overdosed upon. All one can say is motherhood statements about supportive management, be it ventilation, sedation, administration of various antidotes, dialysis, vasopressor support, correction of acid-base abnormalities, and councelling of the parents.

In short:

Risk assessment

  • Taking into account:
    • Agent
    • Dose taken
    • Time since ingestion
    • Clinical features
    • Patient factors (eg. chronic renal impairment)
  • What is the point, one might ask? Taken directly from the EMJ article:
    • Early recognition of trivial poisonings allows patient and family to be reassured and unnecessary treatment abandoned
    • Psychosocial assessment can occur earlier and it is likely that length of stay in hospital will be shortened
    • Potentially serious poisonings can be detected early
    • Balanced decisions about gastrointestinal decontamination can be made
    • Appropriate specialised procedures or antidotes can be organised
    • Early communication with the ICU can take place 

Screening investigations:

  • Urine drug screen
  • ECG
  • Paracetamol level
  • CXR ( did they aspirate?)
  • Specific drug levels
  • CK and troponin
  • ABG
  • Serum osmolality

Decontamination

  • Gastric lavage (almost always inappropriate)
  • Whole bowel irrigation (only for iron and slow-release tablets)
  • Activated charcoal

Enhanced elimination

Specific antidotes

Supportive ICU management

A) - If in doubt, keep them intubated.

B) -  Keep them ventilated with a mandatory mode initially; ensure that the minute volume is enough to help them compensate for the acidosis they were experiencing. Classically, the patients with salicylate overdose end up dying suddenly if they are ventilated slowly, and the ensuing respiratory acidosis improves the lipid solubility and CNS penetration of their serum salicylate. Specific strategies may apply in certain circumstances, particularly in the case of paraquat toxicity (where oxygen has a known deleterious effect)

C) - haemodynamic support as required - this may range from ECMO to beta blockade and nitroprusside

D) - nothing specific can be said except the use of benzodiazepines is encouraged in the literature, both as a means of seizure prophylaxis and as a means of controlling a potential impending withdrawal syndrome. Practically, long-acting benzodiazepines are not desirable, as they obscure the neurological findings.

E) - Normal electrolyte concentrations protect the patient from such badness as torsade

F) - Forced diuresis may not be indicated for virtually any intoxication apart from perhaps cyclophosphamie, or in the case of rhabdomyolysis. However, maintaining a good urine output promotes renal clearance of drugs which benefit from it.

G) - There is rarely a firm contraindication to nutrition

H) - There is rarely a requirement for transfusion, but exchange transfusion is a possible solution to severe methaemoglobinaemia.

I) - antibiotics are rarely required; extremes of temperature may require cooling or heating.

References

The website of the American Academy of Clinical Toxicology has several position statements which might be useful to the fellowship candidate:

 

Ipecac Syrup

Single-Dose Activated Charcoal

Multi-Dose Activated Charcoal

Cathartics

Whole Bowel Irrigation

Gastric Lavage

Urine Alkalization

Question 1a - 2004, Paper 2

You are called to see a 16-year-old girl in the Emergency Department.  She was brought in by ambulance after being found unconscious by her parents.  She was last seen alive and well 12 hours ago.  Several empty bottles of tablets were found beside her.

(a)        What is your initial management?

College Answer

(a)        What is your initial management?

Initial management is to and assess vital signs (airway, breathing and circulation), institute appropriate monitoring (ECG, pulse oximeter) and institute whatever immediate supportive management is required. Early supportive management of the airway and breathing may require endotracheal intubation (eg. significant hypoxia, GCS < 9, not protecting airway, respiratory acidosis), and circulation will normally require intravenous fluids and/or vasopressors (ie. intravenous ± central venous access). History of presentation (including nature of tablets found and other medications she would have access to), past history of medical problems (including treatment and allergies) and time course of presentation are essential (from whoever can provide the most information). Examination allows search for toxidromes (pupils, sweating, heat rate etc), focal neurological signs (which may suggest an alternate diagnosis) and any complications of unconsciousness including aspiration, pressure areas etc.) Early investigations would include blood gases (oxygenation, ventilation, acidosis), electrolytes (especially K), blood glucose and paracetamol levels (treatable problem). Other specific investigations may be indicated (eg CK, Creatinine, phosphate if concerned about rhabdomyolysis; osmolality for osmolar gap etc.). It would be reasonable to consider a head CT if there were concerns about the neurological state. Decontamination
and antidotes are considered in subsequent parts of this question.

Discussion

a)

  • Attention to the ABCS, with management of life-threatening problems simultanous with a rapid focused examination and a brief history
  • Airway:
    • assess the need for immediate intubation
    • given that the patient is unconscious, intubation will likely be required
  • Breathing/ventilation
    • maintain oxygenation with a reservoir mask, or by chemanical ventilation as indicated
  • Circulatory support
    • assess the need for fluid resuscitation and vasopressor support;
    • gain multiple points of intravenous access and commence cardiovascular monitoring.
    • Invasive hemodynamic monitoring may be required
  • Supportive management
    • Check BSL and maintain normoglycaemia
    • Check ABG and assess the need to correct her acid-base status
  • Specific management
    • This will be dictated by the history, physical examination, and the findings of investigations.

References

The website of the American Academ of Clinical Toxicology has several position statements which might be useful to the fellowship candidate:

 

Ipecac Syrup

Single-Dose Activated Charcoal

Multi-Dose Activated Charcoal

Cathartics

Whole Bowel Irrigation

Gastric Lavage

Urine Alkalization

Question 11 - 2004, Paper 2

Compare and contrast the pharmacology of ketamine, morphine and dexmetatomidine when used for analgesia in the critically ill.

College Answer

Ketamine is a non barbiturate general anaesthetic, produces a state of “dissociative anaesthesia” with profound analgesia. Pharmaceutics: racemic mixture , clear liquid in ampoule with 200mg in 2mL. Pharmacokinetics: initial rapid redistribution (T 1/2 10 to 15 minutes) representing anaesthetic action, followed by beta phase half life of about 2.5 hrs. 2-
50% protein bound. Volume of distribution 1.8 L/kg, 90% excreted by urine (mainly after extensive hepatic metabolism to less active metabolites, only 4% unchanged). Can be administered IV, IM or SC. Pharmacodynamics: onset of action within 30 seconds and duration of analgesia approximately 30 minutes (profound analgesia of shorter duration). Relative preservation of respiratory reflexes (except at higher dosages), can increase BP and ICP, and result in involuntary movements and emergent reactions. Anaesthetic doses 1-2 mg/kg, but analgesia can be obtained with lower doses (eg. 10-20 mg; 0.1-0.3 mg/kg) or by low dose infusion (eg. 0.1 mg/kg/hr). Value if need short periods of profound analgesia.

Morphine is an opioid analgesic which activates predominantly mu opioid receptors. Pharmaceutics: clear liquid in ampoule with 10 (or 15) mg in 1 mL. Pharmacokinetics:
initial rapid redistribution, followed by more prolonged elimination phase half life of about
2 hrs. 35% protein bound. Volume of distribution 3.3 L/kg, 90% excreted by urine (mainly after hepatic metabolism to active metabolite morphine-6-glucuronide which has a longer half life). Can be administered IV, IM or SC. Pharmacodynamics: rapid onset of action when injected intravenously and duration of analgesia dose (up to hours). Effects significantly prolonged with hepatic or renal dysfunction. Adverse effects include hypotension, sedation, and significant depression of respiratory and gastrointestinal
function, and rarely “biliary” spasm. Antagonist exists: naloxone. Dosage: 10 mg IM, or 1-2 mg boluses (eg. with PCA) and infusion of 1-5 mg/hr.

Dexmedetomidine is a relatively selective alpha-2 adrenoreceptor agonist (providing its sedative and analgesic effects). Pharmaceutics: expensive, clear liquid in ampoule with
200mcg in 2 mL. Pharmacokinetics: initial rapid redistribution (six minutes), followed by more prolonged elimination phase half life of about 2 hrs. 94% protein bound. Volume of distribution 1.5 L/kg. Near complete hepatic metabolism to inactive metabolites which are then excreted in the urine. Administration only by IV infusion (load of 1 mcg/kg, followed by 0.2 to 0.7 mcg/kg/hr). Effects may be prolonged with hepatic or renal dysfunction. Predominant adverse effects include hypotension, bradycardia (including sinus arrest) and dry mouth. Predominant use is for profound sedation for short periods (eg. 24 hours).

Discussion

First of all, let me waggle a shaming finger at the misspelling of dexmedetomidine. It is, of course, possible that some underpaid typist was charged with writing up the college paper from dictated notes, if the examiners in 2004 were loath to soil their carefully manicured hands with the crudeness of keyboard work. Nevermind. At least it is correct in the answer.

Features

Ketamine

Morphine

Dexmedetomidine

Class

NMDA receptor antagonist

Opiate

Central Alpha-2 agonist

Pharmacokinetics

Multiple possible routes of administration

Half life 0.5-2hrs

Hepatic metabolism into weakly active metabolites, which are cleared renally

Multiple possible routes of administration

Half-life 4-9 hours, however has a rapid initial redistribution.

Hepatic metabolicm into active metabolites, which are cleared renally

IV administration

Half-life 20 minutes

Hepatic metabolism into inactive metabolites

Receptor activity

NMDA receptor antagonist

opiate receptor agonist

Central alpha-2 receptor agonist

Mechanism

Competitive antagonism of glutamate neurotransmission at the NMDA receptor

Activates opioid receptors, which activatenumerous intracellular signalling pathways including decreasing the intracellular cAMP levels and closing voltage-sentitive calcium channels, overall leading to decreased neuronal excitability

Acts on presynaptic alpha-2 receptors to decrease the release of synaptic noradrenaline in the central nervous system, thereby decreasing sympathetic outflow.

Benefits in analgesia

Reduces opiate requirements

Effective co-analgesic

Effective for procedural analgesia and sedation

Strong analgesic

Decreases sympathetic responses to pain

Decreases sedation requiremens

Improves tolerance of uncomfortable features of ICU stay (eg. ETT, central lines)

Reduces opiate requirements

Effective co-analgesic

Effective for procedural analgesia and sedation

Additional benefit of sedation without respiratory depression

Adverse effects

Confusion
Delirium
Bronchorrhoea
Sialorrhoea
Sympathetic stimulation

Confusion
Delirium
Hypotension
Euphoria/Dysphoria
Respiratory depression
Constipation

Bradycardia
Hypotension

References

Question 1b - 2004, Paper 2

You are called to see a 16-year-old girl in the Emergency Department.  She was brought in by ambulance after being found unconscious by her parents.  She was last seen alive and well 12 hours ago.  Several empty bottles of tablets were found beside her.

(b)       What is the role of decontamination of the digestive tract?

College Answer

(b)       What is the role of decontamination of the digestive tract?

The role of decontamination of the digestive tract is controversial. This does not refer to Selective Decontamination of the Digestive tract (SDD) which is a form of antimicrobial prophylaxis. The induction of emesis is not favoured. The routine use of gastric lavage and/or activated charcoal has lost favour in the majority of overdose situations because of the limited evidence of benefit, and the possibility of harm (eg. aspiration or trauma). There are some situations where either or both of these techniques should be considered: early presentation (eg. < 1 hour) or presence of a drug which would delay gastric emptying, and presence of toxic drug in high quantities (eg. lethal dose) especially if in a slow release form. Administration of charcoal does not absorb small highly ionised chemicals (eg. metals, electrolytes, acids and alkali). Additional techniques such as repeated activated charcoal (and/or cathartics eg. sorbitol) or whole bowel irrigation (eg. with polyethylene glycol balanced electrolyte solution) may be considered (especially with slow release preparations). Rarely is surgical removal required.

Discussion

b)

Rationale for decontamination

  • In any overdose, especially early, there is some proportion of the ingested drug which still has not absorbed.
  • This unabsorbed drug could potentially be cleared from the gut
  • This would result in a reduced total dose of the drug
  • The reduced total dose should also result in a reduced total toxicity
  • Ergo, the removal of undissolved drugs should reduce the toxicity of the overdose

Techniques of decontamination and their indications

  • Activated charcoal, single or multiple doses
  • Induced emesis (abandoned)
  • Gastric lavage (largely abandoned; only indicated within the first hour)
  • Whole bowel irrigation (only indicated for iron and slow release enteric coated tablets)
  • Surface decontamination for skin-absorbed toxins

Situations which merit the use of gut decontamination

  • The overdose is recent (within the last hour)
  • There is reason to believe a large number of undissolved tablets is still present in the stomach or gut
  • There is no adequate antidote to the drug, and the overdose is lifethreatening

Criticsm of gut decontamination techniques

  • Possibility of aspiration is ever-present, particularly if the airway is unprotected
  • Likelihood of effect diminishes with time.
  • Even charcoal may have serious complications, eg. bowel obstruction
  • Many of the early studies which lauded the effectiveness of gut decontamination techniques such as emesis or lavage were focused on the effectiveness of the emetic in achieving emesis, or in the lavage recovery of some abstract marker substance. No studies focused on patient outcome. Patient outcomes do not seem affected by decontamination techniques.
  • The removal of a proportion of ingested drug may have no effect on the course of the overdose, in terms of outcome. One may think of this in terms of the difference between absorbing 100g of paracetamol vs. only absorbing 75g. In either case, your liver is screwed.

References

The website of the American Academ of Clinical Toxicology has several position statements which might be useful to the fellowship candidate:

Ipecac Syrup

Single-Dose Activated Charcoal

Multi-Dose Activated Charcoal

Cathartics

Whole Bowel Irrigation

Gastric Lavage

Urine Alkalization

Question 16 - 2005, Paper 2

List  potential   adverse  drug reactions,  and  outline  how  they  may  impact  on  your management of the critically ill patient.

College Answer

This question was best answered using a systematic approach. Many candidates did not address the impact of adverse reactions on management, in particular how to prevent or minimise their occurrence. Reasonable list should include effects of drug alone and drug on drug.

Expected reactions (ie extensions of known pharmacologic effects) are many and should include pharmaceutic (eg. compatibility issues), pharmacokinetic (eg. absorption, enzyme induction), pharmacodynamic (eg. innocent bystander organs, competition). Unexpected reactions include idiosyncratic (haematological, hepatic, dermatological), and allergic (mild through to anaphylaxis and anaphylactoid). Management requires detailed drug history: drugs administered (over the counter as well as prescription drugs), alcohol intake, previous drug reactions, conditions that make adverse effects more likely (eg. respiratory depression and sleep apnoea, or severe airways disease). Examination: to look for conditions that may make reactions more likely. Careful prescribing (ie only using drugs when indicated) with attention to potential interactions (including physical incompatibilities etc), and appropriate monitoring (eg. drug levels, organ function).

Twenty-three out of forty-one candidates passed this question.

Discussion

This is a question which would benefit from a systematic approach. An alternative to the college system is the one offered in Table 1 from the  Lancet article by Edwards et al (2000). Apparently, that is the official classification used in "the industry".

Classical pharmacological classification of adverse drug reactions

Dose-related reactions

  • This can include adverse effects at either normal dose or overdose.
  • These may include expected extesions of the therapeutic effect of the drug, eg. bleeding in heparin.
  • Toxic effects eg. serotonin syndrome
  • Side effects are included, eg. anticholinergic effects of tricyclics 

Non-dose-related reactions

  • This refers to drug effects which are totally unrelated to the dose (i.e. any exposure is enough to trigger such a reaction).
  • Allergic reactions
  • Anaphylaxis
  • Idiosyncratic reactions, eg. purpura or drug-induced SLE

Dose and time related reactions

  • This refers to drug effects which occur due to dose accumulation, or with prolonged use
  • Adrenal suppression with corticosteroids is one example.

Time related reactions

  • This refers to drug effects which occur due to prolonged use in a drug which doesnt tend to accumulate.
  • An example might be tardive dyskinesia afte decades of using typical antipsychotics

Withdrawal reactions

  • This refers to the undesired effects of ceasing the drug
  • Classical examples might include opiate withdrawal and rebound hypertension after stopping clonidine.

Unexpected failure of therapy

  • This category has been added to describe an undesirable reduction in the drug's efficacy (or, the undesirable increase thereof)
  • Examples may include increased clearance by dialysis and plasmapheresis, drug interactions alterinc metabolism, and the effects of critical illness on protein bindind and elemination.

Management of adverse drug reactions

Immediate management:

  • ABCs
  • Identification and withdrawal of the offending agent
  • Immediate IM adrenaline (500mcg) for anaphylaxis
  • Hydrocortisone and antihistamines for allergic reactions

Investigation

  • need for thorough drug history
  • search for evidence of previous drug reactions
  • thorough history of allergies
  • search for predisposition to adverse effects
  • Assessment of drug interactions
  • Investigations (such as plasma concentration measurements, biopsies, and allergy tests)
  • Organ system function assessment (EUC, LFTs, TFTs, FBC for neutropenia, etc)
  • Rechallenge with the drug should be considered

Prevention

  • awareness of impaired clearance mechanisms due to organ pathology
  • careful prescribing
  • attention to drug interactions
  • rational management of polypharmacy
  • monitoring of drug levels
  • staff education regarding safe prescribing and administration
  • pharmacist participation in ICU rounds

References

Leape, Lucian L., et al. "Pharmacist participation on physician rounds and adverse drug events in the intensive care unit." Jama 282.3 (1999): 267-270.

Edwards, I. Ralph, and Jeffrey K. Aronson. "Adverse drug reactions: definitions, diagnosis, and management." The Lancet 356.9237 (2000): 1255-1259.

Cullen, David J., et al. "Preventable adverse drug events in hospitalized patients: a comparative study of intensive care and general care units." Critical care medicine25.8 (1997): 1289-1297.

 

Bates, David W., et al. "Incidence of adverse drug events and potential adverse drug events: implications for prevention." Jama 274.1 (1995): 29-34.

Question 18 - 2005, Paper 2

Compare and contrast the pharmacology of  noradrenaline, vasopressin and phenylephrine when used as vasopressors in the critically ill.

College Answer

Noradrenaline is the catecholamine released by postganglionic adrenergic nerves. Direct agonist acting on alpha (vasoconstrictor: arterial and venous) and beta-1 (contractility, pro-arrhythmic) adrenergic receptors. Not absorbed enterally. Rapidly metabolised by COMT and MAO, resulting short (minutes) duration of effect (usually administered as intravenous infusion into central vein at rate of 0.5 to 100 mcg/min). Used clinically to increase blood pressure (usually in the setting of vasodilatory shock).

Vasopressin is a hormone/neurotranmitter with a complex series of effects. Direct action on a number of receptors (V1 (vascular: vasoconstriction), V2 (renal: anti-diuresis), V3 (pituitary), OTR (oxytocin receptor subtypes) and P2 (purinergic). Not absorbed enterally. Rapidly inactivated by trypsin and peptidases, resulting in short (minutes) duration of effect (longer on kidneys as very low concentration are required). Used clinically as treatment for diabetes insipidus (IM, IV or

intranasal), and more recently by intravenous infusion (via central vein at rates of 0.01 to 0.1

U/min) to increase blood pressure (usually in the setting of vasodilatory shock) or as a large intravenous bolus providing potent vasoconstriction during cardiac arrest (40 units). Potentiates the action of other vasoconstrictor agents.

Phenylephrine is a synthetic alpha-1 adrenoreceptor agonist, similar in structure to adrenaline. Not administered enterally, biotransformation not well described (not metabolised by COMT) but duration of action longer than naturally occurring catecholamines (still minutes). Used clinically for vasoconstrictor effects, usually administered intravenously either in small bolus doses or occasionally as an intravenous infusion (via a central vein at rates of 40 to 180 mcg/min).

Refractory hypotension may respond to agents with combined alpha-1 & alpha-2 activity (e.g. noradrenaline).

Discussion

Great masses of text in homage to noradrenaline and vasopressin are available elsewhere. Phenylephrine is less common.

A Comparison of Selected Vasopressors

Features

Noradrenaline

Phenylephrine

Vasopressin

Class

Endogenous catecholamine

Synthetic catecholamine

Endocrine nonapeptide

Pharmacokinetics

Half-life 2-3minutes
Metabolised by MAO and COMT

Half-life 5-10 minutes

0.002 units /kg/min;

or, 2-2.4 units/hr

Receptor activity

Predominantly alpha-1 agonist activity;
Some beta-1 and beta-2 effects at high doses

Affinity for receptors decreases in acidosis

Strongly selective for alpha-1 receptors

Affinity for receptors decreases in acidosis

Acts on V1 receptors (for vasopressor activity) and on V2 receptors (for antidiuretic activity).
Some crossover with oxytocin with respect to uterine contraction.
Affinity for receptors is unchanged by acidosis

Mechanism

Increases intracellular IP3, which in turn increases the availablility of intracellualr calcium to smooth muscle contractile proteins

Increases intracellular IP3, which in turn increases the availablility of intracellualr calcium to smooth muscle contractile proteins

V1 effect is by  Gq-protein coupled receptors, which also increases intracellular IP3.
V2 effect is via Gs-protein coupled receptors, and cAMP.

Clinical effects

Arterial and venous vasoconstriction
Reflex bradycardia 
Increased afterload and preload

Arterial and venous vasoconstriction
Reflex bradycardia 
Increased afterload and preload

Arterial and venous vasoconstriction
Reflex bradycardia 
Increased afterload and preload 
Increased resoprtion of water in the cortical collecting duct

References

For those (like me) who are unfamiliar with phenylephrine, there is an excelent monograph on its properties published by Schering-Plough (it is the Briefing Document for NDAC Meeting in December 14, 2007)

Question 2 - 2006, Paper 1

Outline the clinical features, mechanism of toxicity and therapy of cyanide poisoning.

College Answer

Clinical features:

Symptoms of toxicity range from non-specific symptoms such as headache and nausea to depressed consciousness, seizures and cardiopulmonary arrest. Laboratory features include lactic acidosis and unexpectedly high venous oxygen saturation (with low a-v oxygen difference)

Mechanism of toxicity:

Cyanide blocks mitochondrial cytochrome oxidase resulting in cytotoxic hypoxia and lactic

acidosis.

Therapy:

As cyanide is highly toxic and can penetrate intact skin or be inhaled. Consequently decontamination is essential and mouth-to-mouth resuscitation should not be performed. In cases of ingestion gastric lavage may reduce absorption.

There are various antidotes based on three principles:

1. Conversion of haemoglobin to methaemoglobin (Amyl nitrite or sodium nitrite are used for this purpose). Methaemoglobin has a higher affinity for cyanide than does cytochrome oxidase and therefore promotes its dissociation from cytochrome oxidase. Since methaemoglobin does not carry oxygen, excessive methaemoglobinaemia can lead to anoxia. Methaemoglobin should be measured during treatment; a desirable level is between 20% and 30%.

2. Direct binding to EDTA or the vitamin B12 precursor hydroxocobalamin. A high dose (5 grams)

of hydroxocobalamin is required but has minimal toxicity (in contrast to other treatments).

3. Thiosulfate (administered as sodium thiosulphate) reacts with cyanide forming the relatively non- toxic thiocyanate, which is excreted in the urine. This action is slow and provides little effect in the acute phase.

Discussion

This one is among my favourites.

Clinical features:

  • bradycardia
  • tachypnoea
  • severe metabolic acidosis - predominantly due to lactate
  • high central venous oxygen saturation (low OER)
  • acute renal failure
  • acute hepatic dysfunction
  • acute heart failure and pulmonary oedema
  • circulatory failure, shock
  • coma and seizures
  • Diagnosis of cyanide toxicity rests on historical features which are strongly suggestive (eg. inhalation of smoke in a plastic-based fire) as well as severe lactic acidosis, and in the absence of carbon monoxide poisoning. This might be enough to merit some doses of the (reasonably safe) empirical antidote therapy. The gold standard of diagnosis is the serum cyanide level, which may take too long.

Dose - response  relationship

  • In terms of blood levels:
    • 8-20 µmol/L = mild symptoms
    • 20-38 µmol/L = tachycardia, vasodilation
    • 38-95 µmol/L = decreased level of consciousness
    • 95 µmol/L and above = almost uniformly fatal

Mechanism of toxicity:

  • The best discussion of this mechanism (brief enough for revision work) can be found in the Chest case study about the unresponsive biochemistry professor in the bath tub (Mutlu et al, 2002)
  • Lactic acidosis develops due to the uncoupling of oxidative phosphorylation: cyanide interferes with the  electron transport chain by binding to the ferric Fe3+ ion of cytochrome oxidase. The mechanism of lactic acidosis due to cyanide toxicity is discussed elsewhere.
  • Neurotoxicity occurs at modest doses; initially there is CNS stimulation (dizziness, confusion, restlessness, and anxiety) which is followed by stupor, opisthotonus, convulsions, fixed dilated pupils and unresponsive coma. This is due to the cyanide-stimulated release of excitatory neurotrasmitters, such as NMDA and glutamate.
  • Oxidative damage to lipid bilayers due to free radical generation tends to break the blood-brain barrier and causes a vasodilated SIRS-like state of cardiovascular collapse (but this tends to happen only with very large doses)
  • The development of pulmonary oedema, pulmonary vasoconstriction and coronary artery spasm are blamed on "biogenic amines", vasoactive substances which are supposedly liberated from cyanide-affected endothelia. There is not a lot to back this up in the literature.

Management of cyanide toxicity:

Supportive management

  • A) intubation to support the airway of the comatose patient
  • B) 100% FiO2 has been recommended, but may have no effect (the oxygen content of blood is not the issue)
  • C) Circulatory support with vasopressors and inotropes (cardiac output must be maintained if parenteral rescue agents are to ever get to the tissues)
  • D) Sedation and analgesia should be offered, keeping in mind that normal mechanisms of renal clearance and hepatic metabolism are likely to be grossly impaired
  • E) The electrolytes may be grossly deranged. Specifically, there will be severe acidosis, which may call for sodium bicarbonate purely because the serum bicarbonate is trending towards zero, and you don't want to run out of buffer.
  • F) Renal failure is likely, and serious thought should be given to early dialysis
  • G) A PPI should be started, because these poeple tend to suffer extensive sloughing of their gastric lining, with ensuing gastritis
  • H) In the presence of an excess cyanhaemoglobin, one may consider exchange transfusion - but this is rarely a major contributor to the lethality of a cyanide overdose. usually, oxygen-carrying capacity of the blood is not an issue.

Antidotes:

  • Decontamination may to be effective (however most cyanides are rapidly absorbed).
    • Cyanide has a short half-life (~ 2 hours), but in massive overdose the decontamination of plasma by dialysis may be feasible and has contributed to the survival of at least one historical victim (Wesson et al, 1985).
  • Hydroxycobalamin
    • Hydoxycobalamin binds cyanide and forms cyanocobalamin
    • This is the antidote of choice
    • Advantages include a lack of toxicity for non-poisoned victims (thus, it may be given empirically)
    • The onset of action is rapid
    • It may be given in the pre-hospital setting and requires no monitoring.
    • The side efects are relatively minor; perhaps the most striking is the tendency for the body fluids to turn a vivid red-orange color.
    • dicobalt edetate may be an alternative cobalt-based binder, but hydroxycobalamin is more widely available, and much less toxic. LITFL mentions that dicobalt edetate causes "seizures, chest pain and dyspnoea, head and neck swelling, hypotension, urticaria and vomiting"
  • Sodium thiosulfate
    • Sulfur donors in general act by offering a sulfur ion to the endogenous rhodanese enzyme which converts cyanide to thiocyanate
    • Like hydroxycobalamin, this is a reasonably safe option - there are few side effects.
  • Induction of methaemoglobinaemia
    • Methaemoglobin binds free cyanide and forms cyanmethaemoglobin.
    • Various drugs are available for this. Sodium nitrite and amyl nitrite are the most frequently quoted. Methylene blue is also available, but is not without its side-effects.
    • Hall and Rumack, writing in the mid-1980s, recommended a sniff of a freshly cracked amyl nitrite inhaler as the first-line rescue therapy, presumably because back in those days everybody had a few of those in their back pocket at all times.

References

Hall, Alan H., and Barry H. Rumack. "Clinical toxicology of cyanide." Annals of Emergency Medicine 15.9 (1986): 1067-1074.

Beasley, D. M. G., and W. I. Glass. "Cyanide poisoning: pathophysiology and treatment recommendations." Occupational medicine 48.7 (1998): 427-431.

Mutlu, Gökhan M., et al. "An unresponsive biochemistry professor in the bathtub." CHEST Journal 122.3 (2002): 1073-1076.

Cummings, T. F. "The treatment of cyanide poisoning." Occupational Medicine 54.2 (2004): 82-85.

Wesson, D. E., et al. "Treatment of acute cyanide intoxication with hemodialysis." American journal of nephrology 5.2 (1985): 121-126.

 

Question 12 - 2006, Paper 1

Compare and  contrast the pharmacology  of lignocaine,  magnesium  and  amiodarone when used in the treatment of ventricular tachycardia.

College Answer

Lignocaine:

Class I (membrane stabilising) antiarrhythmic agent. Sodium channel blockades results in decreased action potential duration and shortened refractory period. Rapidly distributed to all body tissues. Approximately 65% protein bound; elimination half-life 1.6 hours (80% metabolised in liver). Adverse effects: lightheaded, hypotension, cardiovascular collapse, heart block, confusion and convulsions.

Dosage used in the treatment of ventricular tachycardia: 1 to 1.5mg/kg with subsequent boluses (up to 3 mg/kg total), followed by infusion (1-4 mg/min, at decreasing dose, up to 24 hours).

Magnesium (as sulphate or chloride):

Second most abundant intracellular cation. Depresses neuronal activation. Widely distributed, duration of action about 30 minutes. Filtered by kidneys, but most is reabsorbed.

Adverse effects include: nausea, flushing, CNS depression, coma, and heart block.

Dose used in the treatment of ventricular tachycardia: 5 mmol bolus (which may be repeated), followed by infusion of 20 mmol over 4 hours.

Amiodarone:

Class III antiarrhythmic. Prolongs action potential duration, and prolongs refractory period of atrial, nodal and ventricular tissues. Highly protein bound with very high apparent volume of distribution (6 L/kg); accumulates in adipose tissue and highly perfused organs. Half-life (with chronic dosing) is 14 to 59 days, mainly excreted via the liver and bile.

Adverse effects: hypotension/circulatory collapse, bradycardia, sinus arrest, nausea and flushing. Torsades de pointes can be induced. Hyper- or hypo-thyroidism can be induced. Multiple other potential organ dysfunctions with more chronic use (including some potentially fatal).

Dosage used in the treatment of ventricular tachycardia: 5 mg/kg (or 300 mg in adults) which can be repeated, and followed by an infusion (15 mg/kg/hr).

Discussion

Again, the college answer outlines all the important stuff.

The table below reconfigures this into a more eye-pleasing form.

Features

Lignocaine

Magnesium

Amiodarone

Class

Class 1a antiarrhytmic

Divalent cation

Class 3 antiarrhytmic (though it has effects of all 4 classes)

Administration / dosage

IV
initial bolus 1-1.5mg/kg.

Then, 4 mg/kg for the first hour, then tapering infusion to 1mg/kg for 24 hrs

IV

10-20mmol/L given over 15-60 minutes,

or 5 mmol boluses followed by 20mmol infusion

IV

150-300mg, followed by an infusion of 900mg over 24 hrs

Pharmacokinetics

Rapid hepatic metabolism into inactive metabolites.
Half life 1-2hrs

Rapid distribution; some proportion becomes intracellular; the rest is renally excreted.

Rapid distribution, with a vas volume of dsitribution; becomes bound to tissue proteins.
Half-life 15-60 days

Mechanism

Inhibits voltage-gated sodium channels, decreasing the duration of action potentials and decreasing the  duration of repolarisation

Mechanism uncertain; appears to act as an antagonist to the entry of calcium into depolarising cells.
Depresses the activity of excitable tissues.

Beta-blockade
Voltage-gated soidum channel blockade
Potassium channel blockade
Calcium channel blockade
Prolongs the duration of both action potential and refractory period.

Adverse effects

Neurological disturbances eg. paraesthesia, seizures

Muscle weakness, decreased reflexes, hypotension

Prolongation of QT interval, risk of Torsades.
Array of chronic toxicities included pulmonary fibrosis and hypothyroidism or hyperthyroidism

References

Question 3 - 2006, Paper 2

Compare and contrast the pharmacology of ceftriaxone, gentamicin and meropenem.

College Answer

Ceftriaxone: vial with yellow water soluble powder for reconstitution; only administered parenterally, 33-66% excreted unchanged in urine, no active metabolites, 85-95% protein bound, elimination half life 6-9 hours (> 36 hours with severely impaired renal function), usual dosage 0.5 to 2g IV 12 or 24 hourly; 3rd generation cephalosporin antibiotic, inhibits cell wall synthesis, covers most gram negative rods (except Pseudomonas), and Gram positive cocci (except Methicillin Resistant, and group D streptococci); adverse reactions uncommon, but include overgrowth of non-susceptible organisms, and occasional haematologic, renal and hepatic adverse effects.

Gentamicin: ampoule with 80 mg/2 mL; only administered parenterally, excreted almost entirely by glomerular filtration, elimination half life 2-3 hours, no active metabolites, usual dosage 1 mg/kg tds or up to 5 mg/kg as daily dose, careful monitoring of blood levels required, especially if renal impairment (trough level not > 2 mcg/mL); aminoglycoside antibiotic, inhibits protein synthesis, covers most gram negative rods (including pseudomonas, but variability from hospital to hospital); serious adverse reactions include oto- and renal toxicity, potentiated by other oto- and nephro-toxins, prolongation of neuromuscular blockade may occur, other reactions uncommon.

Meropenem: vial with water soluble powder for reconstitution; only administered parenterally, 70% excreted unchanged in urine (requiring reduction of dosage if significant renal impairment), plasma binding 2%, elimination half life 1 hour, no active metabolites, usual dosage 500mg to 2g every 8 hours; carbapenem antibiotic, inhibits cell wall synthesis, active against a broad spectrum of aerobic and anaerobic bacteria (including Gram positive cocci and Gram negative rods, but excluding MRSA, Enterococcus faecium, Stenotrophomonas and many Pseudomonas); serious adverse reactions are rare, but include overgrowth of non- susceptible organisms, and occasional haematologic, gastrointestinal and hepatic adverse effects.

Discussion

I refuse to believe that I would have lost marks in this question by not commenting on the colour of ceftriaxone powder.

Here is a tabulated answer with slightly less detail than the college provides.

 

Ceftriaxone

Gentamicin

Meropenem

Class

3rd gen cephalosporin

Aminoglycoside

Carbapenem

Administration

IV – 1g; up to 3g/day

IV; daily dosing 5-7mg/kg

IV 1-2g – up to 6g/day

Pharmacokinetics

Cleared both renally (unchanged) and via biliary excretion (33%)

Cleared renally; rapidly and unchanged

Cleared renally

Mechanism of action

Inhibition of bacterial cell wall synthesis by binding to penicllin-binding proteins

Inhibition of bacterial protein synthesis by binding to the 30S subutnit of the bacterial ribosome

Inhibition of bacterial cell wall synthesis by binding to penicllin-binding proteins

 

Indications

Infections by susceptible gram-positive organisms;
Some gram-negative cover

Infections by susceptible gram-negative organisms;
Little role in treatment of gram-positive infections (with the exception of enterococcal endocarditis)

Infection by suceptible gram-positive and gram negative aerobic and anaerobic organisms; broad cover

Precautions

Seizures in overdose
Bone marrow suppression in chronic high-dose use
Rapid development of microbial resistance

Nephrotoxicity
Ototoxicity
Accumulation in renal failure increases risk of toxicity
Potentiates neuromuscular blockade

Seizures in overdose

References

Question 14 - 2006, Paper 2

Compare and contrast the clinical features and management of a patient  following beta blocker overdose with those of a patient  following calcium-channel blocker overdose.

College Answer

Beta-blockers

Ca channel blockers

General

Clinical features depend
on type of drug and amount ingested. Most symptoms within 4 hrs of ingestion

Varying CVS effects

CVS

Hypotension,
bradycardia, AV block,

Hypotension, bradycardia

CCF

Other systems

Bronchospasm,
hypoglycemia, hyperkalemia, stupor, coma, seizures

Hyperglycemia, nausea
and vomiting reported. Seizures uncommon.

Treatment

Charcoal, fluids, Beta
agonists, vasopressors, atropine, pacing, glucagon.

Charcoal, IV fluids, IV
CaCl2, glucagon, hyperinsulinemai- euglycemia therapy, pacing, charcoal hemoperfusion in the case of verapamil

Discussion

Though satisfactory, the college answer lacks qualities which help the studying candidate generate some memory of the differences between these drugs. The table below builds on the college answer by highlighting in bold the key differences between these overdoses.

 

Calcium channel blocker overdose

Beta-blocker overdose

Clinical features

Bradycardia
Hypotension
Heart block
Hyperglycaemia
Constipation/ileus

Bradycardia
Hypotension
Heart block
HYPOglycaemia
Bronchospasm

Management:

Antidote

Ionised calcium (eg. calcium chloride)
Glucagon
Insulin-dextrose
Inotropes and vasopressors

Glucagon
Insulin-dextrose
Inotropes and vasopressors

Decontamination

Activated charcoal

Activated charcoal

Enhancement of clearance

Hemoperfusion for verapimil

Hemoperfusion for metoprolol

References

DeWitt, Christopher R., and Javier C. Waksman. "Pharmacology, pathophysiology and management of calcium channel blocker and β-blocker toxicity." Toxicological reviews 23.4 (2004): 223-238.

Question 2 - 2007, Paper 1

a) List an antidote (1drug specific to the agent) in the event of an
overdose with  each of the agents listed below in the table.

Benzodiazepines

Beta blockers

Cyanide

Digoxin

Heparin

Iron

Methanol, ethylene glycol

Methaemoglobinemia

Organophosphate

Opiates

Lead

Paracetamol

b) Which of the agents in the above list are not adsorbed by activated charcoal?

College Answer

Benzodiazepines

Flumazenil

Beta blockers

Glucagon, adrenaline

Cyanide

Na thiosulfate, hydroxocobalamin;

Digoxin

Fab,

Heparin

Protamine

Iron

Desferrioxamine

Methanol, ethylene glycol

ethanol

Methaemoglobinemia

Ascorbic acid, methylene blue

Organophosphate

Atropine, pralidoxime

Opiates

Naloxone

Lead

Dimercaprol

Paracetamol

N-Acetylcysteine

b) Which of the agents in the above list are not adsorbed by activated charcoal?

Lead, alcohols, Fe, cyanide

Discussion

This question closely resembles Question 28.1 from the second paper of 2009, as well as Question 14.2 from the first paper of 2008. It has slightly different drugs in its table, but otherwise it is essentially the same.

However, it does ask about the charcoal.

Thus:

The following drugs are NOT treatable by charcoal:

  • cyanide
  • heparin
  • iron
  • methanol/ethylene glycol
  • methaemoglobin
  • lead

More on this can be found in a brief summary of ICU toxicology.

References

Question 14.1 - 2008, Paper 1

A 58 year old farmer with a history of depression was found collapsed in his shed. On arrival at the Emergency Department, his GCS was 10 (E2, V3, M5), respiratory rate was 23, and mouth ulceration was noted with a green coloured substance staining his lips, hands and clothes.

His arterial blood gas and biochemistry on admission were as follows:

FiO2

0.5

pH

7.29

PCO2 (mmHg)

35 (4.6 kPa)

PaO2

68 (9.0 kPa)

HCO3 (mmol/L)

16

(24-28)

Base Excess (mmol/L)

-9

(-2.0 to
+2.0)

Sodium (mmol/L)

140

(135-145)

Potassium (mmol/L)

4.3

(3.5-5.0)

Chloride (mmol/L)

111

(95-105)

Glucose (mmol/L)

7.2

(4.0-6.0)

Lactate (mmol/L)

5.2

<2.5
mmol/L

Haemoglobin (g/L)

162

(130-160)

Creatinine 
micromole/L)

230

(60-120)

a. What is the likely diagnosis?

b. How can you confirm this?

c. List 4 important principles of management specific to this condition.

College Answer

a. What is the likely diagnosis?
Paraquat ingestion

b. How can you confirm this?
Serum paraquat levels
History of exposure

c. List 4 important principles of management specific to this condition.
1)  Risk assessment based on estimate of quantity of Paraquat ingested

2)  Gastrointestinal decontamination with diatomaceous earths, activated charcoal or sodium resonium
3)  Monitoring for organ dysfunction (respiratory, CVS, renal, GIT, adrenal, hepatic, CNS)
4)   Avoid high FiO2           

Discussion

Though the most likely diagnosis is an overdose of some sort of horrible herbicide (and past history suggests the college likes their paraquat questions), one should still go though the motions of analysing a blood gas from basic principles.

Firstly, what we have here is a hypoxia with a widened A-a gradient.

The PAO2 should be (0.5 x 713) - (35 x 1.25), or 311mmHg - so the gradient is a whopping 246.

Next, we have a metabolic acidosis (the BE is -9)

This disorder is inadequately compensated by ventilation. No matter which equation you use, the CO2 should be lower. If you apply the "7.xx" rule, the CO2 shold be the last two digits of the pH - 29. If you apply Winter's Formula, the CO2 should be around 32. Thus, a mild respiratory acidosis also exists.

The anion gap is only slightly raised, 17.3 (140+4.3 - 111 - 16)

The delta ratio is therefore 0.66 (5.3 / 8) -if we take the normal anion gap to be 12.

The metabolic acidosis is therefore a mixed disorder.

The serum osmolality and urea are not provided, so we cannot calculate an osmolar gap.

Anyway... The gas exchange defect suggests pulmonary oedema, the bloods suggest renal failure, and the history screams herbicide. Paraquat selectively attacks the alveoli and causes renal necrosis. Ergo, its a case of paraquat poisoning. Another plausible explanation is early stages of ethylene glycol toxicity. Antifreeze is green - stained with fluoresceine so you can find radiator cracks more easily - and this could account for the hapless farmer's lips and hands. But it does not cause mucosal ulceration, and if the college really wanted the candidates to explore ethylene glycol as the main differential they would probably have provided them with a serum osmolality level.

Anyway. Diagnosis of paraquat toxicity consists of a suspicious history, confirmed by formal paraquat levels.

Management consists of supportive care of multi-organ system failure, and decontamination byFuller's Earth, which is essentially calcium montmorillonite, or bentonite - a absorbent aluminium phyllosilicate, formed from the weathering of volcanic ash.

Dialysis is probably going to be useless, as paraquat is rapidly eliminated and by the time you get the circuit set up most of it will have gone already. The alveolar and renal damage will have been done by then, so you have nothing to gain (other than a more rapid control of the acid-base disturbance).

Hyperoxia is to be avoided, as it has been demonstrated to exacerbate the oxidative toxicity of paraquat.

References

Gawarammana, Indika B., and Nicholas A. Buckley. "Medical management of paraquat ingestion." British journal of clinical pharmacology 72.5 (2011): 745-757.

Clark, D. G. "Inhibition of the absorption of paraquat from the gastrointestinal tract by adsorbents." British journal of industrial medicine 28.2 (1971): 186-188.

Kehrer, James P., Wanda M. Haschek, and Hanspeter Witschi. "The influence of hyperoxia on the acute toxicity of paraquat and diquat." Drug and chemical toxicology 2.4 (1979): 397-408.

Dinis-Oliveira, R. J., et al. "Paraquat poisonings: mechanisms of lung toxicity, clinical features, and treatment." Critical reviews in toxicology 38.1 (2008): 13-71.

Question 14.2 - 2008, Paper 1

List an antidote (1 drug specific to the agent) in the event of an overdose with each of the agents listed below in the table.

Agent

Antidote

Benzodiazepines

 

Beta blockers

 

Cyanide

 

Digoxin

 

Iron

 

Methanol, Ethylene glycol

 

Methemoglobinemia

 

Organophosphate

 

Opiates

 

Paracetamol

 

College Answer

Agent

Antidote

Benzodiazepines

Flumazenil

Beta blockers

Glucagon, adrenaline

Cyanide

Na thiosulfate, hydroxocobalamin,

Digoxin

Fab,

Iron

Desferrioxamine

Methanol, Ethylene glycol

Ethanol, 4 -methylpyruvate

Methemoglobinemia

Ascorbic acid, methylene blue

Organophosphate

Atropine, pralidoxime

Opiates

Naloxone

Paracetamol

N-Acetylcysteine

Discussion

This question closely resembles Question 28.1 from the second paper of 2009.

It has fewer drugs in its table, but otherwise it is essentially the same.

References

Question 17 - 2008, Paper 1

Compare the use of propofol and dexmedetomidine when used for sedation in the mechanically ventilated patient  with specific reference to
A) pharmacodynamics
B) indications
C) complications
for each of the drugs. (You may tabulate your answer).

College Answer

Propofol 
Pharamcodynamics
•    GABA receptor action, though different from benzodiazepine receptor
•    Hydrophobic with high lipid solubility that allows it to cross blood brain barrier rapidly. Lipid solubility allows rapid redistribution to tissues so duration of action is only a few minutes.

B)         Indications
•    Sedation in ICU for ventilation
•    Sedation for procedures such as ETT, endoscopy, TOE etc
•    Sedation for transport
•    Effective anticonvulsant.

C)  Complications

•    Cardiovascular: hypotension from preload reduction due to dilation of venous capacitance vessels & mild myocardial depression.
•    Hyperlipidaemia possible: monitor triglyceride levels. Adjust TPN accordingly
•    Propofol infusion syndrome: dysrhythmias, heart failure, metabolic acidosis, hyperkalaemia, rhabdomyolysis. Beware of high doses ( > 80 microg/kg/min) and/or higher concentrations ( 2% vs 1% ).

Dexmedetomidine

A)
Selective alpha-2 agonist with both sedative and analgesic properties.

B) Indications
Patients are sedated when undisturbed but they arouse easily with minimal stimulation, allowing frequent neurologic examinations. Useful in the agitated, ventilated patient.

•    Analgesic sparing in post operative patients.
•    Results in less delirium compared to benzodiazepines.

C) Complications
•    Cardiovascular: bradycardia & hypotension. ( Vasoconstriction & hypertension have been reported with higher doses )
•    Not well studied for long term administration to critically ill, mechanically ventilated patients.Licensed for use in Australia for 24 hours only, though utilised in trials for up to 120 hours.

Discussion

Agian, the college has effectively and concisely summarised everything in their answer.
But, they didn't tabulate it, even though they invited the candidates to do so.

I have tabulated it for them.  

Feature

Propofol

Dexmedetomidine

Pharmacodynamics

GABA agonist; 
General anaesthetic

Centrally acting alpha-2 receptor agonist;
Sedative

Indications

Sedation (rapid onset)
Anaesthesia
Rapid induction of anaesthesia (eg. for intubation)
Decrease of cerebral metabolic demand

Sedation (slower in onset)
Analgesia: synergy with opiates
Management of agitation in both intubated and non-intubated patients

Less delirium than with benzodiazepines

Complications

Loss of airway reflexes
Hypotension
Hyperlipidemia
Propofol infusion syndrome

Bradycardia
Hypotension

More adverse effects than with benzodiazepines

 

References

Arain, Shahbaz R., and Thomas J. Ebert. "The efficacy, side effects, and recovery characteristics of dexmedetomidine versus propofol when used for intraoperative sedation." Anesthesia & Analgesia 95.2 (2002): 461-466.

 

Venn, R. M., and R. M. Grounds. "Comparison between dexmedetomidine and propofol for sedation in the intensive care unit: patient and clinician perceptions†."British journal of anaesthesia 87.5 (2001): 684-690.

 

Jakob, Stephan M., et al. "Dexmedetomidine vs midazolam or propofol for sedation during prolonged mechanical ventilation: two randomized controlled trials." JAMA307.11 (2012): 1151-1160.

Question 13 - 2008, Paper 2

Compare and contrast the pharmacology of dobutamine and levosimendan

College Answer

LEVOSIMENDAN

DOBUTAMINE

Class

Calcium sensitiser

Synthetic catecholamine

Pharmacokinetics

Slower onset of action
(requires loading dose)
t1/2 of parent compound 1 h Metabolised by conjugation and acetylation
Active metabolite with t1/2 
80-96 h (Effects persist once
infusion ceased for up to 7-
10 days)

Onset of action in 1 – 2 min
t1/2 < 3min
Metabolised by methylation and
conjugation 
Excreted in urine and bile
Inactive metabolites

Mechanism of
action

Increases myocyte sensitivity
to calcium by binding to troponin C
Opens ATP-dependant K+ channels in vascular smooth muscle

Selective beta-1 and beta-2
receptor agonist

Therapeutic 
effects

Increased cardiac
contractility 
Coronary, systemic and pulmonary vasodilation Reduced preload and afterload without impairing diastolic relaxation

Increased cardiac contractility
Mild peripheral vasodilation

LEVOSIMENDAN

DOBUTAMINE

Adverse effects

Hypotension 
Headache

Drug-induced myocardial
ischaemia 
Rate/ rhythm disturbances
Hypotension Hypertension in XS dose Tachyphylaxis

Contraindications

Hypersensitivity to
levosimendan 
LV outflow tract obstruction Severe renal or hepatic failure
Severe hypotension and tachycardia
History of torsade de pointes

Hypersensitivity to dobutamine
LV outflow tract obstruction

Discussion

With such a detailed and comprehensive college answer, one can do nothing more than to rewrite the table with less information, so as to render the information it contains more memorable.

Features

Dobutamine

Levosimendan

Class of drug

Synthetic catecholamine

Calcium sensitizer

Administration

IV infusion 5-15mcg/kg/min

IV infusion 0.05-0.2mcg/kg/min

Pharmacokinetics

Rapidly metabolised by COMT; 
Half-life ~ 5 minutes
No active metabolites

Excreted into the small intestine
Slowly eliminated metabolites (half life ~ 80 hours)

 

Mechanism of action

Activates beta-1 adrencoeptors and increases heart rate and contractility by increasing the intracellular levels of cAMP, thus increasing the availablility of intracellular calcium.

Enhances the affinity of contractile proteins (partiularly cardiac troponin C) for calcium, thereby increasing contractility without incurring additional ATP cost

Clinical effects

Increased inotoropy
Increased chronotropy
Peripheral vasodilation (beta-2 effect of one of the enantimers)

Increased inotropy
Increased chronotropy
Increased lusitropy
Pulmonary vasodilation
Peripheral vasodilation (by action on ATP-sensitive potassium channels in vascular smooth muscle)

Adverse effects

Arrhythmia
Hypotension
Increased cardiac metabolic demand, thus potentially exacerbating ischaemia                 

Arrhythmia
Hypotension


 

References

Antila, Saila, Stig Sundberg, and Lasse A. Lehtonen. "Clinical pharmacology of levosimendan." Clinical pharmacokinetics 46.7 (2007): 535-552.

Papp, Zoltán, et al. "Pharmacological mechanisms contributing to the clinical efficacy of levosimendan." Cardiovascular drug reviews 23.1 (2005): 71-98.

 

Question 19.1 - 2009, paper 1

With reference to intoxications, list the relevant physical features of hemodialysis and hemoperfusion filters which make them suitable for use and give one example of a toxin cleared by each of these.

College Answer

The relevant physical features include

  • Haemodialysis 
  • Small molecule < 500 Da
  • Water Soluble
  • Non-protein bound
  • Low volume of distribution

Haemoperfusion 

  • Larger non-protein-bound molecules 1000 to 1500 KDa

Examples

Haemodialysis :  Lithium, metformin
Haemoperfusion: Phenobarbitone, theophylline

Discussion

A more detailed discussion  is available regarding the use of haemoperfusion and haemodialysis in toxicology There's also a revision page about haemoperfusion in a broader context.  Interestingly, the college question asked for the properties of filters which make them suitable for use, rather than the properties of the drugs (which is what the college answer consists of).

If were to actually answer the question, it would look something like this:

Haemodialysis filters:

  • Large surface area of the membrane increases the rate of molecule transport
  • Porosity of the membrane affects the maximum molecular weight of the transported molecules
  • Ultrafiltration rate, as a function of porosity (among other factors) affects the rate of removal for larger molecules
  • Dialysate flow rate affects the rate of clearance for drugs with smaller molecules
  • Features which favour drug clearance are small molecule size, small volume of distribution, and large protein-unbound fraction
  • One example is lithium

Hemoperfusion filters:

  • Large surface area of resin or charcoal filter enhances adsorption by presenting a larger contact surface for the filtered blood
  • Features which favour drug clearance include high affinity for activated charcoal, or the presence of specific antibody-coated resin on the filter
  • One example is theophylline

References

Nenov, Vesselin D., et al. "Current applications of plasmapheresis in clinical toxicology." Nephrology dialysis transplantation 18.suppl 5 (2003): v56-v58.

Holubek, William J., et al. "Use of hemodialysis and hemoperfusion in poisoned patients." Kidney international 74.10 (2008): 1327-1334.

Ghannoum, Marc, et al. "Hemoperfusion for the treatment of poisoning: technology, determinants of poison clearance, and application in clinical practice." Seminars in dialysis. Vol. 27. No. 4. 2014.

Ghannoum, Marc, et al. "Blood purification in toxicology: nephrology’s ugly duckling." Advances in chronic kidney disease 18.3 (2011): 160-166.

Takki, S., et al. "Pharmacokinetic evaluation of hemodialysis in acute drug overdose." Journal of pharmacokinetics and biopharmaceutics 6.5 (1978): 427-442.

Question 20 - 2009, paper 1

Define ideal body weight and what is its significance to dosing of drugs. Briefly, outline the effects morbid obesity (body mass index > 40kg/m2) may have on the pharmacokinetics of medications in critically ill adults.

College Answer

°     Ideal body weight is usually estimated from formulae or approximately:

IBW (kg) males =height cm -100, IBW (kg) females height cm -110

°     Dosing weight is best worked out from ideal body weight.

Lean body weight or dosing weight = ideal body weight + (ABW-IBW) x 0.4

Pharmacokinetics

Distribution

°     markedly affected by ratio of adipose tissue to lean body mass

°     Increased volume of distribution for lipid soluble drugs

°     Accumulation of lipophilic drugs in fat stores

°     May increase dose needed to gain effect

°     Vd of hydrophilic drugs less affected but blood, extracellular fluid, body organ, and connective tissue volume are also increased.

°     Total body water may be increased by resuscitation volume etc

°     Cmax reduced and T1/2 increased

°     Lipid soluble drugs usually dosed on ABW, water soluble drugs dosed on ideal or lean body weight

Metabolism

°     Variable effects. More likely to be affected by critical illness with drug interactions, reduced hepatic blood flow, altered protein binding

Excretion

°     Obese patients with normal renal function have an increased glomerular filtration rate and thus an increased clearance of drugs excreted by the kidney. Co existing disease processes eg diabetes may change this

°     Calculated and measured creatinine clearance correlate poorly in obesity and in critically ill

Thus morbidly obese predisposed to inadequate dosing and increased toxicity. Need to measure serum levels of drugs with low therapeutic index.

Discussion

Ideal body weight:

  • Who says its "ideal"? Well. The definition is "the ideal weight associated with maximum life-expectancy for a given height". 
  • There are numerous equations, all of which tend to agree (or, close enough for government work). One such equation is:
     

       Ideal body weight (kg) = height (cm) - 100
     

    (100 for males, and 105 or 110 for females.)
    Ideally, this technique should tell you what weight a person should  be, and therefore help you estimate their lean body mass so as to dose their drugs appropriately.
  • The bizarre assumption made in this calculation is that inside every morbidly obese person there is a mass of lean tissue which is ideal for their maximal life expectancy, and which is directly proportional to their height. All patients of the same height would end up receiving the same dose if you use this metric, no matter how much they weigh.
  • This is clearly incorrect, as obese individuals tend to have more lean body mass than normal people (as more muscle is required to carry all the extra weight).

Effect on absorption:

  • Gastric emptying may be increased OR decreased (and it is unpredictable).
  • Absorption from the subcutaneous compartment will be slowed due to poor blood flow to subcutaneous fat
  • Intramuscular injection (or intrathecal, or even intravenous for that matter) is made difficult by poor access.

Effect on distribution:

  • Increased volume of distribution for lipid-soluble drugs
  • Increased accumulation of drugs in the fat compartment
  • Blood flow in fat is poor in people of normal weight: it is only about 5% of the total cardiac output.
  • In obese individuals, blood flow to fat is even poorer.
  • Obese individuals are also likely to have a degree of heart failure which further decreases blood flow.
  • This makes their fat a large compartment of potential distribution for lipophilic drugs which fills gradually, and then becomes a slowly emptying reservoir.
  • Body fluid volume is also increased, increasing the volume of distribution of water-soluble drugs
  • Protein binding may be altered (but this is far from clear: most papers seem to say that albumin binding is unchanged)

Effect on metabolism:

  • Hepatic clearance is slowed not only by decreased cardiac output but also by fatty infiltration. But, you never actually know whether metabolic activity is going to be mre or less rapid. remember that lean tissue (and potentially metabolic organ mass) may be increased.
  • However, increased CYP450 (2E1) activity has been observed
  • Increased Phase II conjugation activity may be present

Effect on clearance:

  • Diabetes which co-exists with obesity tends to damage kidneys, slowing the renal clearance. However, glomerular filtration rate may be increased in healthy obese individuals.
  • Biliary clearance may be slowed by bile stasis or existing bile duct disease

Effect on pragmatic drug dosing and monitoring:

  • Obese people have a larger absolute lean body mass (LBM), as well as fat mass. Lean components account for 20-40% of the absolute body weight ( it is purely the support muscle required to drag all that fat around). Exactly how much muscle is hidden in any given obese individual is difficult to est accurately with the aid of standard equations.
  • The net effect of this is that both under-dosing and over-dosing is more likely than with individuals of normal weight, and monitoring of therapeutic levels is important.
  • Pharmacokinetic data in obesity does not exist for most drugs. Generally drug dosing instructions are for total body weight, but this does not take into account obese individuals, or fluid-overloaded ICU patients.
  • In obese individuals, the ideal body weight is likely to underestimate their lead body mass, leading to under-dosing.
  • The total body weight is likely to over-estimate the dose and lead to overdosing.
  • Thus, in such patients, most drug dosing should be tailored to lean body weight (LBW).
  • In the case of most nonlipophilic drugs, IBW is sufficient because Vd does not change.
  • In the case of  strongly hydrophilic drugs, instead of calculating LBW, 20% can be added to the IBW to account for the increase in lean body tissue content.
  • In the case of strongly lipophilic drugs, and in the case of many anaesthetic agents, LBW is the ideal metric.

References

De Baerdemaeker, Luc EC, Eric P. Mortier, and Michel MRF Struys. "Pharmacokinetics in obese patients." Continuing Education in Anaesthesia, Critical Care & Pain 4.5 (2004): 152-155.

 

Cheymol, Georges. "Effects of obesity on pharmacokinetics." Clinical pharmacokinetics 39.3 (2000): 215-231.

Question 18 - 2009, Paper 2

A two year old boy is suspected of ingesting iron tablets.

a) List three clinical signs of iron poisoning.

b) List two investigations which would support the diagnosis of iron poisoning.

c) Which blood gas (a or b or c) would be most consistent with iron poisoning?  Justify your choice of answer.

 

a

b

c


pH

7.1

7.55

7.45

pCO2

34 mmHg (4.5 kPa)

30 mmHg (4.5 kPa)

34 mmHg (4.5 kPa)

pO2

75 mmHg (10 kPa)

90 mmHg (12 kPa)

70 mmHg (9.3 kPa)

BE

-18 mmol/L

+4 (mmol/L)

-0.1 mmo/L

d) List three treatments specific for iron poisoning and their mechanisms  of action.

e)  List one serious long term complication of iron poisoning.

College Answer

a)          List three clinical signs of iron poisoning.

Clinical sign

Cause

Nausea / vomiting/ haematemesis

Acute gastritis, ischaemia

Diarrhoea

Abdominal pain

Melaena

Tachypnoea

Metabolic acidosis

Coma/ seizures

Shock/ hypotension

Myocardial depression

Oliguria

Capillary leak

Jaundice / coagulopathy

Hepatic necrosis

b)         List two investigations which would support the diagnosis of iron poisoning.

Iron Level > 300 microgm/dL, or 63 micromol/L

Abdominal XR:

Shows iron tablets

Blood gas

Metabolic acidosis

Hyperglycaemia

Coagulopathy

Interference with coagulation cascade/
hepatic failure

Deranged liver enzymes

From hepatic necrosis

Raised white cell count

c)         Which blood gas (a or b or c) would be most consistent with iron poisoning?  Justify your choice of answer.

 

a

b

c


pH

7.1

7.55

7.45

pCO2

34 mmHg (4.5 kPa)

30 mmHg (4.5 kPa)

34 mmHg (4.5 kPa)

pO2

75 mmHg (10 kPa)

90 mmHg (12 kPa)

70 mmHg (9.3 kPa)

BE

-18 mmol/L

+4 (mmol/L)

-0.1 mmo/L

Answer: Metabolic acidosis due to uncoupling of oxidative phosphorylation.

d)         List three treatments specific for iron poisoning and their mechanisms  of action.

Desferrioxamine

(Binds intravenous iron to form water soluble
ferrioxamine that is renally excreted)

Whole bowel irrigation

(Polyethylene glycol: works with minimal
complications, aim for clear rectal effluent and absence of tablets on AXR)

Exchange transfusion with plasmapheresis.

Surgical/ endoscopic removal of tablets

(If seen on AXR).

Treat coagulopathy

Treat hyperglycaemia

Aggressive volume resuscitation (as
capillary leak a feature)

Dialysis

But limited efficacy

Gastric Lavage with HCO3

(Controversial)

Note: Charcoal is ineffective.

e) List one serious long term complication of iron poisoning.

1. Bowel obstruction (esp gastric outlet)
2. GI strictures

Discussion

As this question closely resembles Question 8 from the second paper of 2013, I will not elaborate excessively.

a)

Feature Causes
   
Tachypnoea
  • Metabolic acidosis
Shock, circulatory collapse
  • Third space fluid losses
  • Blood and fluid loss from the ulcerated gut
  • Cardiotoxic effects, with cardiogenic shock
  • Vasodilation due to SIRS
Hypoglycaemia
  • Acute hepatotoxicity
Coma
  • Hypoglycaemia
  • Acute cerebral oedema due to liver failure
High anion gap metabolic acidosis
  • Lactic acidosis
  • Ketosis
  • Minor contribution from iron itself (conversion of  Fe3+ to Fe2+ produces a net loss of a cation, and therefore contributes to the decrease in the SID)
Hyperlactatemia
  • Acute hepatotoxicity and liver failure
  • Shock state
  • Direct mitochondrial toxicity
Renal failure
  • Shock state
  • mitochondrial (tubular) toxicity, ATN
Gastric ulceration
  • direct corrosive effect of the drug
Haemorrhage, melaena
  • from ulcerated gut surface

b)

  • Iron levels
  • ABG (demonstrating a mixed metabolic acidosis)

c)

  • Gas (a) most closely resembles lactic acidosis, as the base deficit and acidosis are significant.

d)

Decontamination

  • Activated charcoal has no role to play
  • Whole bowel irrigation - until effluent turns clear - is a good strategy; much of the toxicity is related to gut ulceration, and by diluting the iron in the gut lumen you may be able to ameliorate this direct corrosive effect, even if you don't manage to prevent toxic absorption.
  • Surgical removal of tablets - if a bezoar is clearly visible on the AXR

Enhanced elimination

  • Exhange transfusion: the removal of iron-poisoned blood is ery old-school, but it works (Movassaghi et al, 1969)
  • Haemodialysis can be considered to help remove the iron-desferrioxamine complexes, as they are renally excreted and there may not be enough renal function to remove this product. Otherwise, apart from correcting acidosis there is no role for dialysis.

Specific antidote

Supportive care

  • Intubation will likely be required to protect the airway not only from the decreased level of consciousness but also from the risks of aspiration associated with whole bowel lavage.
  • Mechanical ventilation will likely be with mandatory mode, to decrease the demands on the failing myocardium
  • Circulatory support should consist of simultaneous fluid resuscitation, inotrope and vasopressor infusions
  • Sedation should be rationalised, given that the patient is already in a coma before the sedation is given, and that the liver is doing little metabolically.
  • Correction of acidosis with bicarbonate may be indicated if catecholamine responsiveness is lost.
  • Electrolyte replacement -losses must be anticipated, the leaky gut and bowel lavage will result in potassium and phosphate depletion.
  • Haemodialysis may be required to maintain metabolic normality, as well as to remove ammonia which may accumulate due to the acute hepatocellular necrosis
  • Hypoglycaemia and ketosis will likely develop. The patient will need a dextrose infusion, as hepatic and skeletal muscle glycogen stores will be depleted.
  • Nutrition will likely be parenteral for some time, depending on the extent of gastric ulceration.
  • Coagulopathy will develop due to hepatocellular necrosis. Coagulation factor replacement will be required.

e)

Toxicity manifests in four stages, where the late Stage IV represents gastrointestinal scarring (4-6 weeks since ingestion) - gastric scarring and pyloric stricture are the specific features.

 

References

The Royal Childrens Hospital has a good set of guidelines for irone overdose.

Abhilash, Kundavaram PP, J. Jonathan Arul, and Divya Bala. "Fatal overdose of iron tablets in adults." Indian journal of critical care medicine: peer-reviewed, official publication of Indian Society of Critical Care Medicine 17.5 (2013): 311.

REISSMANN, KURT R., and THOMAS J. COLEMAN. "Acute Intestinal Iron Intoxication II. Metabolic, Respiratory and Circulatory Effects of Absorbed Iron Salts." Blood 10.1 (1955): 46-51.

REISSMANN, KURT R., et al. "Acute Intestinal Iron Intoxication I. Iron Absorption, Serum Iron and Autopsy Findings." Blood 10.1 (1955): 35-45.

 

Question 22 - 2009, Paper 2

Outline the role of vasopressin and its analogues in the critically ill patient.

College Answer

•    Vasopressin - Septic shock – Improves blood pressure, No evidence of mortality benefit- NEJM study, some benefit in the less sick population
•    Terlipressin - Hepatorenal syndrome
•    Diabetes insipidus – Desmopressin improves polyuria, restores serum Na concentrations
•    Pitressin - Variceal bleed
•    Vasopressin - Cardiopulmonary resuscitation
•    Desmopressin - Post cardiopulmonary bypass bleeding – 20 units improves platetet dysfunction related bleed but may cause myocardial ischemia
•    Von Willebrands’s

Discussion

The college answer embarrasses itself by treating "pitressin" as something separate from vasopressin, whereas in fact they are the same damn thing.

Anyway. An extensive homage to vasopressin is available elsewhere. The discussion of its relatives seems to work better as a table.

Vasopressin and its Chemical Relatives

Substance

Physiology

Uses in critical care

Vasopressin
  • An endogenous nonapeptide, with a ring-shaped molecule
  • Product of the posterior pituitary
  • Plays a major role in the homeostatic mechanisms in defence of tonicity, by increasing the absorption of water in the collecting duct (V2 receptors)
  • At higher doses, acts as a vasoconstrictor  by acting on V1 receptors.

Use in septic shock:

  • Used in septic shock as an adjunct to noradrenaline. Theoretically, the use of vasopressin in septic shock overcomes the pituitary vasopressin depletion which tends to occur in such patients.
  • Together with noradrenaline,  vasopressin reduces the amount of noradrenaline required. (VASST
  • In meta-analysis (Neto et al, 2012), there was an associated small improvement in mortality (RR = 0.87) when vasopressin was compared with noradrenaline.

Use in cardiac arrest:

  • In a meta-analysis of 5 randomised controlled trials (Aung et al, 2005) there was no clear advantage of vasopressin over adrenaline in the treatment of cardiac arrest.
Terlipressin
  • Synthetic analogue of vasopressin: a dodecapaptide
  • Structurally similar to vasopressin: the dodecapeptide contains a nonapeptide chain identical to lysine vasopressin (same as the porcine vasopressin molecule)- where the lysine is substituted for arginine,
  • Has its own intrinsic vasoconstrictor activity and also acts as a pro-drug for lysine vasopressin (when it undergoes hepatic metabolism)
  • Selective for the splanchnic circulation

Use in hepatorenal syndrome

  • According to a recent meta-analysis (Israelsen et al, 2015), terlipressin is more effective than placebo, and  equal with noradrenaline in efficacy. It reduces mortality from Type 1 HRS and  is presently the recommended drug of choice for this condition.

Use in variceal haemorrhage

  • Terlipressin reduces bleeding from varices by decreasing portal venous pressure.
  • In terms of mortality and re-bleeding rates, there seem to be no difference between terlipressin, somatotatin and octreotide (Seo et al, 2014).

Use in septic shock

  • When compared to vasopressin, terlipressin infusion appears to be equivalent in efficacy (Morelli et al, 2009)
Desmopressin
  • Synthetic analogue of vasopressin: with a deaminated cysteine molecule, and the substitution of a D-stereoisomer for L-arginine.
  • Duration of action is 6 to 14 hours.
  • Greater selectivity for V2 receptors, with little V1 activity
  • The effect is therefore largely limited to increasing water retention
  • A pleasing side-effect is the improvement in platelet function and the increases in the plasma levels of factor VIII and vWF, as desmopressin causes them to be released from their storage sites in the vascular endothelium (Manucchi, 1997)

Use in diabetes insipidus

Use in haemostasis

  • According to Massimo (2007), the following haemostatic disorders benefit from desmopressin:
    • von Willebrands disease
    • Haemophilia A (mild)
    • Uraemic platelet dysfunction
    • Platelet dysfunction due to antiplatelet agents
    • Bleeding during cardiac surgery
     
     
 

References

Sharshar, Tarek, et al. "Depletion of neurohypophyseal content of vasopressin in septic shock*." Critical care medicine 30.3 (2002): 497-500.
 
Aung, KoKo, and Thwe Htay. "Vasopressin for cardiac arrest: a systematic review and meta-analysis." Archives of internal medicine 165.1 (2005): 17-24.
 
Israelsen, Mads, Aleksander Krag, and Lise Lotte Gluud. "Terlipressin versus other vasoactive drugs for hepatorenal syndrome." The Cochrane Library (2015).
 
 
Casu, Stefania, Annalisa Berzigotti, and Jaime Bosch. "Use of Vasoactive Drugs for Acute Variceal Bleeding." Variceal Hemorrhage. Springer New York, 2014. 135-145.
 
 
 
 
Mannucci, Pier Mannuccio. "Desmopressin (DDAVP) in the treatment of bleeding disorders: the first 20 years." Blood 90.7 (1997): 2515-2521.
 
Kim, R. J., et al. "Vasopressin and desmopressin in central diabetes insipidus: adverse effects and clinical considerations." Pediatric endocrinology reviews: PER 2 (2004): 115-123.
 
Franchini, Massimo. "The use of desmopressin as a hemostatic agent: a concise review." American journal of hematology 82.8 (2007): 731-735.

Question 28.1 - 2009, Paper 2

List an antidote  (one (1) drug specific to the agent) in the event of an overdose with each of the agents listed below in the table.

Agent

Antidote

Benzodiazepines

 

Beta blockers

 

Bupivacaine

 

Cyanide

 

Digoxin

 

Ethylene glycol

 

Isoniazid

 

Methanol,

 

Methemoglobinemia

 

Organophosphate

 

Opiates

 

Lead

 

Valproate

 
 

College Answer

List an antidote  (one (1) drug specific to the agent) in the event of an overdose with each of the agents listed below in the table.

Agent

Antidote

Benzodiazepines

Flumazenil

Beta blockers

Glucagon

Bupivacaine

Intralipid

Cyanide

Cyanocbalamin/ Sodium thiosulphate

Digoxin

Fab

Ethylene glycol

Ethanol, Fomepizole

Isoniazid

Pyridoxine

Methanol,

Ethyl alcohol

Methemoglobinemia

Methylene blue

Organophosphate

Atropine

Opiates

Naloxone

Lead

Dimercaprol, BAL

Valproate

Carnitine

Discussion

This question does not warrant an especially extensive discussion.

Instead, I will link to intersting articles.

In the list provided by the college, there are standard drugs which everyone would know the antidotes for, and non-standard ones which may not be totally familiar to people without a toxicology background.

Pyridoxine is the antidote for isoniazid

Pyridoxine is a co-factor in the synthesis of GABA; isoniazid interferes with this synthesis, and causes seizures in overdose. The supplementation of pyridoxine seems to prevent the worst of isoniazid toxicity (it seems the inhibition of lactate metabolism is not such a big deal).

Carnitine is the antidote for valproate

Or so it is thought. The most disturbing aspects of valproate toxicity are valroate-induced hyperammonaemic encephalopathy and hepatotoxicity. Carnitine deficiency is implicated in both, and seems to be caused by chronic valproate administration more so than acute. The reason for the efficacy of carnitine in valproate overdose seems to stem from its central role in beta-oxidation of long chain fatty acids (which is the metabolic pathway taken by valproate). It appears to hasten the resolution of coma, and it seems to protect the liver from necrosis; the mechanism is thought to be the prevention of accumulation of toxic metabolites of valproate.

(Incidentally, carnitine is also being considered as a rescue therapy for propofol infusion syndrome)

Dimercaprol is the antidote for lead poisoning

And mercury, antimony, gold, chrome, cobalt and nickel poisoning. First developed to treat arsenic poisoning during the Second World War, dimercaprol (or British Anti-Lewisite, BAL) is a chelating agent which competes for heavy metal ions with the thiol groups of enzymes, thus preventing the inactivation of those enzymes. The metal-dimercaprol complex is then renally excreted.

Dimercaprol itself is horribly toxic, and its use in heavy metal poisoning is limited to situations where heavy metal levels are high, toxicity is already severe, and water-soluble analogues of dimercaprol (eg. DMPS and DMSA) are not available.

References

Murakami, K., et al. "Effect of L‐Carnitine Supplementation on Acute Valproate Intoxication." Epilepsia 37.7 (1996): 687-688.

Lheureux, Philippe ER, et al. "Science review: Carnitine in the treatment of valproic acid-induced toxicity–what is the evidence?." Critical care 9.5 (2005): 431.

Kam, P. C. A., and D. Cardone. "Propofol infusion syndrome." Anaesthesia 62.7 (2007): 690-701.

Peters, Rudolph A., Lloyd A. Stocken, and R. H. S. Thompson. "British anti-lewisite (BAL)." Nature 156.Nov. 24 (1945): 616.

There is an indepth entry on dimercaprol in www.inchem.org.

Question 28.2 - 2009, Paper 2

Briefly outline the mechanism of effectiveness of sodium bicarbonate in the management of tricyclic antidepressant overdose.

College Answer

Increased serum pH, TCAs are weak bases and therefore increasing serum pH will increase the proportion of non-ionised drug thus causing a greater proportion of drug to be distributed throughout the body away from the heart.
Increased serum Na also overcomes the Na receptor blockade
Alkalinisation also accelerates recovery of sodium channels by neutralizing the protonation of the drug receptor complex.

Discussion

The indication for the use of bicarbonate in tricyclic overdose is the widening of the QRS interval, rather than the metabolic acidosis (which may or may not accompany TCA poisoning).

Exactly how this works is a topic of some debate. In general, the QRS prolongation in TAC overdose seems to result from voltage-gated sodium channel blockade

Some authers have been able to demonstrate that amitryptilline enjoys greater protein binding in a more alkaline environment, which decreases the fraction of free drug.

Other authors have correctly identified sodium (rather than bicarbonate) as the more important ion in sodium bicarbonate; the administration of hypertonic saline seemed to have greater antiarrhytmic effect than sodium bicarbonate!

The last part of the college answer I could find no evidence for, at least not in the way it was worded. A good paper on the molecular mechanisms of sodium channel blockade by imipramine seems to report that intracellular alkalosis seems to favour the unbinding of imipramine from the voltage-gated sodium channel, which vaguely sounds like the thing that the college said.

In summary, bicarbonate in TCA overdose works in the following ways:

  • Increased protein binding of TCAs in an alkaline bloodstream, thus decreasing the biologically active free fraction.
  • Increased availability of sodium in sodium bicarbonate, as a substrate for the voltage-gated channels.
  • Decreased binding of TCAs to the voltage gated sodium channel
  • Correction of metabolic acidosis
  • Volume expansion because of the dilutional effect on TCA concentration
  • Cellular membrane hypopolarisation results from bicarbonate-induced intracellualr shift of potassium. 

References

Hoffman, J. R., and C. R. McElroy. "Bicarbonate therapy for dysrhythmia and hypotension in tricyclic antidepressant overdose." Western Journal of Medicine134.1 (1981): 60.

 

Kerr, G. W., A. C. McGuffie, and S. Wilkie. "Tricyclic antidepressant overdose: a review." Emergency Medicine Journal 18.4 (2001): 236-241.

 

Brown, T. C., et al. "The use of sodium bicarbonate in the treatment of tricyclic antidepressant-induced arrhythmias." Anaesthesia and intensive care 1.3 (1973): 203-210.

 

McCabe, James L., et al. "Experimental tricyclic antidepressant toxicity: a randomized, controlled comparison of hypertonic saline solution, sodium bicarbonate, and hyperventilation." Annals of emergency medicine 32.3 (1998): 329-333.

 

Bou-Abboud, Elias, and Stanley Nattel. "Molecular mechanisms of the reversal of imipramine-induced sodium channel blockade by alkalinization in human cardiac myocytes." Cardiovascular research 38.2 (1998): 395-404.

Question 20 - 2010, Paper 1

(a)        List the risk factors for and the clinical and laboratory findings of propofol infusion syndrome.

(b)        Outline your management of a patient with suspected propofol infusion      syndrome.

College Answer

(a)        List the risk factors for and the clinical and laboratory findings of propofol infusion syndrome.

Risk Factors 
Large doses (> 4mg/kg/hr for > 48 hours in adults): typically, but not always, large dose, long time
Younger age
Acute neurological injury
Low carbohydrate intake
Catecholamine and/or corticosteroid infusion

Clinical and laboratory findings Unexplained lactic acidosis Increasing inotrope support
(Lipaemic serum, propofol levels / chromatography (if available??))
Brugada-like ECG abnormalities (Coved-type = convex-curved ST elevation in V1-
3) 
(Green urine)
Cardiovascular collapse, reflected in PICCO / PAC / ECHO Rhabdomyolysis, high CK, hyperkalaemia
Arrhythmia / heart block
Renal failure

(b)        Outline your management of a patient with suspected propofol infusion      syndrome.

Management: 
High index of suspicion
Discontinue immediately
Monitor for early warning signs: lactate, CK, Urine myoglobin, ECG Standard cardio-respiratory support
Consider pacing (bradycardia often resistant to high dose CA and pacing)

Adequate carbohydrate intake (6-8mg/kg/min)
Carnitine supplementation: theoretical benefit
Haemodialysis and haemoperfusion, used, unproven benefit
ECMO: 2 case reports, readily reversible pathology

Discussion

Propofol infusion syndrome is discussed elsewhere.

It is well covered in an article by Prof Kam.

Pathophysiology of propofol infusion syndrome

  • This tends to happen after about 48 hours of infusion, at over 4mg/kg/hr.
  • The mechanism is likely the inhibition by propofol of coenzyme Q and Cytochrome C.
  • This results in a failure of the electron transport chain, and thus the failure of ATP production.
  • In the event of such a breakdown of oxidative phosphorylation the metabolism becomes increasingly anaerobic, with massive amounts of lactate being produced. Furthermore, fatty acid metabolism is impaired- the conversion of FFAs to acetyl-CoA is blocked, and thus no ATP is produced by lipolysis.
  • On top of that, unused free fatty acids leak into the bloodstream, contributing to the acidosis directly.

a) Risk factors for propofol infusion syndrome

  • Propofol infusion dose of >4mg/kg/hr for over 48 hrs
  • Traumatic brain injury
  • Catecholamine infusion
  • Corticosteroid infusion
  • Carnitine deficiency
  • Low carbohydrate intake: because energy demand is met by lipolysis if carbohydate intake is low, thus leading to the accumulation of free fatty acids.
  • Children more susceptible than adults - probably because their glycogen store is lower, and they depend on fat metabolism.
  • Congenital weirdness: Medium-chain acyl CoA dehydrogenase (MCAD) deficiency

   Clinical features and laboratory findings in propofol infusion syndrome

  •     Acute bradycardia leading to asystole.
    • A prelude to the bradycardia is a sudden onset RBBB with ST elevation in V1-V3; Kam’s article has the picture of this ECG. 
  •     Arrhythmias    
  •     Heart failure, cardiogenic shock
  •     Metabolic acidosis (HAGMA) with raised lactate (and also due to fatty acids)
  •     Rhabdomyolysis, raised CK and myoglobin
  •     Hyperlipidaemia
  •     Fatty liver and hepatomegaly
  •     Coagulpathy
  •     Raised plasma malonylcarnitine and C5-acylcarnitine

Management of propofol infusion syndrome

Enhanced elimination

  • Stop the propofol infusion!
  • "decontamination" might be impossible, but haemodalysis should be commenced to wash out propofol and its toxic metabolites
  • Plasma exchange may be required (Da Silva et al, 2010)

Specific antidote

  • Carnitine  has been mentioned as one of the potential antidotes to propofol infusion syndrome (Uezono et al, 2005). The authors observed a patient who developed a propofol-infusion-like syndrome in response to intravenous lipid emulsion, while in the context of an acquired carnitine deficiency. This led to the hypothesis that "acute fat burden in the setting of inadequate delivery of carbohydrate and acquired carnitine deficiency may impair fatty acid oxidation, leading to the conditions similar to those seen in mitochondrial beta-oxidation defects."

Supportive care

  • Pacing and atropine may be useless (the bradycardia is refractory)
  • Vasopressors and inotropes are aso usually ineffective
  • ECMO is the only answer if circulatory collapse with bradycardia has developed
  • Nutrition with a satisfactory amount of carbohydrate  to reduce the use of fat for metabolism. The college answer quotes a dose rate (6-8mg/kg/min) but it is unclear where the got this value from.

References

Kam, P. C. A., and D. Cardone. "Propofol infusion syndrome." Anaesthesia62.7 (2007): 690-701.

Marinella, Mark A. "Lactic acidosis associated with propofol." CHEST Journal109.1 (1996): 292-292.

Vasile, Beatrice, et al. "The pathophysiology of propofol infusion syndrome: a simple name for a complex syndrome." Intensive care medicine 29.9 (2003): 1417-1425.

Schenkman KA, Yan S. Propofol impairment of mitochondrial respiration in isolated perfused guinea pig hearts determined by reflectance spectroscopy. Critical Care Medicine 2000; 28: 172–7.

Fodale, Vincenzo, and Enza La Monaca. "Propofol Infusion Syndrome." Drug Safety 31.4 (2008): 293-303.

Da-Silva, Shonola S., et al. "Partial-exchange blood transfusion: an effective method for preventing mortality in a child with propofol infusion syndrome." Pediatrics 125.6 (2010): e1493-e1499.

Uezono, Shoichi, et al. "Acquired carnitine deficiency: a clinical model for propofol infusion syndrome?." The Journal of the American Society of Anesthesiologists 103.4 (2005): 909-909.

Mirrakhimov, Aibek E., et al. "Propofol Infusion Syndrome in Adults: A Clinical Update." Critical care research and practice 2015 (2015).

 

Question 29 - 2010, Paper 2

A 16 year old female is admitted to the ICU following a multiple drug overdose.

a)Outline  the  role  of  activated  charcoal  in  the  management   of  drug overdose.

b) What are the complications of activated charcoal therapy?

c) When is dialysis utilised in toxic syndromes?

d) In   the   context   of   an   overdose,   list   3  drugs   for   which   charcoal haemoperfusion may be useful.

College Answer

a)         Outline  the  role  of  activated  charcoal  in  the  management   of  drug overdose.

•          Single dose activated charcoal is generally preferred method of decontamination but does not improve outcome when applied to unselected patients and should not be regarded as routine.

•          Indicated when likely that toxic agent is still within the GI tract (1st hour for most agents) and potential benefits outweigh risks.

b)         What are the complications of activated charcoal therapy?

•           Vomiting
•          Pulmonary aspiration
•          Direct administration to lung via misplaced NG tube (potentially fatal)
•          Impaired absorption of oral medications / antidotes
•          Corneal abrasions
•          Constipation / bowel obstruction (MDAC)

c)         When is dialysis utilised in toxic syndromes?

•          Best if drug is:
•          Water soluble
•          MW <500
•          Not highly protein bound
•          Eg Lithium, Ethylene glycol, Salicylates, Na Valproate
•          Also good for correcting fluid and electrolyte abnormalities

d)         In   the   context   of   an   overdose,   list   3  drugs   for   which   charcoal haemoperfusion may be useful.

•          Common drugs carbamazepine, theophylline, paraquat

Discussion

a)

Rationale for the use of activated charcoal

  • Activated charcoal is the product of the pyrolysis (i.e. decomposition by heat and in the absence of oxygen ) of organic matter. It is "activated" by a series of processes, among them heating it in steam or CO2 at a temperature of 600 Cº, washing with organic acids and drying with hot air. The activation process produces a highly porous substance with a massive surface area, up to 2000m2/g (LITFL quotes 3000 m2/g)
  • Once activated, charcoal can act as a broad-spectrum gastrointestinal adsorbent (Andersen, 1948)
  • Its highest affinity is for compounds with a molecular weight of 100–1000 Da (Krenzelok, 2002)
  • Many pharmacologically active substances fall within this range.
  • Most lifethreatening overdoses are by ingestion.
  • Many such overdoses may present early.
  • Gastric emptying rate may be affected (slowed) by the toxin itself.
  • Ergo, giving charcoal early may reduce the absorption of the drug.

Single-dose activated charcoal

  • Time is the most important factor determining efficacy.
  • If the poison is not in the stomach, single-dose activated  charcoal will be useless.
  • Otherwise, activated charcoal should probably be given soon after most significant ingestions:
    • The frequency of serious complications is low
    • The worst thing that would usually happen is that it simply does not work
    • After a drug is absorbed, there are few effective techniques to enhance its elimination
    • Efficacy is inversely related to the time elapsed after the ingestion. The longer you deliberate about the usefullness of charcoal, the more useless the charcoal becomes. Stop wasting time and just give it
    • This pragmatic why-not-have-a-go approach was championed by Isbister and Kumar in their 2011 recommendation paper for Current  Opinion in Critical Care.
  • However:
    • At least one RCT did not demonstrate any benefit (Eddleston, 2002)
    • In fact, the ED stay was longer, and there was more vomiting.
    • Several similar studies have confirmed a relative lack of benefit in unselected patients
    • On this basis of this, the AACT/EAPCCT recommendation in 2004 was not to give single dose charcoal unless it is clearly within 1 hour of the overdose, and unless the drug is well known to adsorb onto charcoal. In short, they were against the random use of charcoal for the undifferentiated overdose.
    • This recommendation cannot be generalised to the severely intoxicated ICU population, as the major risk from charcoal is aspiration, and if your airway is protected with a big tube, that risk is minimal.

Multiple doses of activated charcoal

The rationale for multiple-dose charcoal is slightly different. It's not a matter of "just give more of it for more effect".

  • Many drugs are excreted via the bile, and undergo extensive enterohepatic recirculation.
  • Multiple dose choarcoal ensures that the cycle of recirculation is interrupted (i.e. the excreted drug is bound by charcoal instead of beaing reabsorbed).
  • In this manner, it is a method of enhanced elimination.

The following is a list of well-accepted indications for multiple dose activated charcoal (from Pierre Gaudrealt, 2005)

  •  Amitriptyline
  • Carbamazepine
  • Cyclosporine
  • Dapsone
  • Dextropropopxyphene
  • Digitoxin
  • Digoxin
  • Disopyramide
  • Nadolol
  • Phenobarbital
  • Phenylbutazone
  • Phenytoin
  • Piroxicam
  • Propoxyphene
  • Quinine
  • Sotalol
  • Theophylline

Substances for which activated charcoal is known to be ineffective

Drugs which are absorbed too rapidly

  • Ethanol
  • Paraquat

Drugs which do not adsorb on to charcoal

  • Corrosive substances, eg. strong acids and alkalis,
  • Iron
  • Lithium.

b)

Complications of charcoal administration

  • Its gross. Patients complain. However, actual vomiting appears to be rare (Isbister et al, 2011)
  • It may absorb usueful medications as well as the toxin.
  • It may increase the risk of aspiration (but if it does, then not y much)
  • Aspirated, it may be more harmful than sterile gastric contents (but if it is, then not by much). In their answer to Question 29 from the second paper of 2010, the college lists direct administration of charcoal into the lung as a valid concern.
  • It may cause bowel obstruction; this is rare, and usually associated with multiple dose charcoal in patients who are poisoned with an agent which affects gut motility.

c)

Use of dialysis in toxicology:

  • the drug is easily dialysed:
    • small molecule
    • water soluble
    • not extensively protein bound
    • small volume of distribution
  • The drug produces dialysable matabolites, which are toxic (eg. ethylene glycol)
  • The toxicity produces an acid-base disturbance which cannot be addressed by any other means (eg. lactic acidosis in cyanide toxicity)

d)

  • Paraquat
  • Parathion
  • Theophylline
  • Carbamazepine
  • Phenytoin
  • Paracetamol
  • Digoxin
  • Diltiazem
  • Metoprolol
  • Colchicine
  • Promethazine
  • Amanita phalloides mushroom toxin (phalloidin)

References

The website of the American Academ of Clinical Toxicology has several position statements which might be useful to the fellowship candidate:

Ipecac Syrup

Single-Dose Activated Charcoal

Multi-Dose Activated Charcoal

Cathartics

Whole Bowel Irrigation

Gastric Lavage

Urine Alkalization

Gaudreault, Pierre. "Activated charcoal revisited." Clinical Pediatric Emergency Medicine 6.2 (2005): 76-80.

Andersen, A. Harrestrup. "Experimental Studies on the Pharmacology of Activated Charcoal. III. Adsorption from Gastro‐Intestinal Contents." Acta Pharmacologica et Toxicologica 4.3‐4 (1948): 275-284.

Krenzelok, Edward P. "New developments in the therapy of intoxications." Toxicology letters 127.1 (2002): 299-305.

Eddleston, Michael, et al. "Multiple-dose activated charcoal in acute self-poisoning: a randomised controlled trial." The Lancet 371.9612 (2008): 579-587.

Isbister, Geoffrey K., and Venkata V. Pavan Kumar. "Indications for single-dose activated charcoal administration in acute overdose." Current opinion in critical care 17.4 (2011): 351-357.

Chyka, P. A., and D. Seger. "Position statement: single-dose activated charcoal. American Academy of Clinical Toxicology; European Association of Poisons Centres and Clinical Toxicologists." Journal of toxicology. Clinical toxicology 35.7 (1996): 721-741.

Harris, Carson R., and Dean Filandrinos. "Accidental administration of activated charcoal into the lung: aspiration by proxy." Annals of emergency medicine22.9 (1993): 1470-1473.

Chyka, P. A., and D. Seger. "Position statement: single-dose activated charcoal. American Academy of Clinical Toxicology; European Association of Poisons Centres and Clinical Toxicologists." Journal of toxicology. Clinical toxicology 35.7 (1996): 721-741.

Isbister, Geoffrey K., and Venkata V. Pavan Kumar. "Indications for single-dose activated charcoal administration in acute overdose." Current opinion in critical care 17.4 (2011): 351-357.

Harris, Carson R., and Dean Filandrinos. "Accidental administration of activated charcoal into the lung: aspiration by proxy." Annals of emergency medicine22.9 (1993): 1470-1473.

UpToDate has a nice table of drugs which are removed by haemoperfusion.

Nenov, Vesselin D., et al. "Current applications of plasmapheresis in clinical toxicology." Nephrology dialysis transplantation 18.suppl 5 (2003): v56-v58.

Holubek, William J., et al. "Use of hemodialysis and hemoperfusion in poisoned patients." Kidney international 74.10 (2008): 1327-1334.

Ghannoum, Marc, et al. "Hemoperfusion for the treatment of poisoning: technology, determinants of poison clearance, and application in clinical practice." Seminars in dialysis. Vol. 27. No. 4. 2014.

Ghannoum, Marc, et al. "Blood purification in toxicology: nephrology’s ugly duckling." Advances in chronic kidney disease 18.3 (2011): 160-166.

Takki, S., et al. "Pharmacokinetic evaluation of hemodialysis in acute drug overdose." Journal of pharmacokinetics and biopharmaceutics 6.5 (1978): 427-442.

Question 5 - 2011, Paper 1

a) Outline the effect of critical illness on enteral drug absorption


b) List the reasons for altered drug clearance in the critically ill.

College Answer

a) Outline the effect of critical illness on enteral drug absorption

•    Multiple   factors   may  alter  gastrointestinal   mucosal   absorption   including   mucosal oedema, disordered gastrointestinal motility and disordered mucosal blood flow
•    Gastric  emptying  / gut motility  affected  by drugs  (opioids.  Anticholinergics,  antacids, inotropes), enteral nutrition, brain or spinal injury, diabetes
•    Incomplete oral medication disintegration or dissolution
•    Changes in pH
 

b) List the reasons for altered drug clearance in the critically ill.

Liver function
Reduced clearance
With hepatic dysfunction present in more than half the critically ill patients, drug clearance may be reduced because of :
a.   Lower hepatic blood flow
b.   Decreased hepatocellular enzyme activity c.   Lower bile flow
d.   Administration of other drugs competing for enzymes
Increased clearance
Hepatic enzyme induction by certain drugs may increase clearance of others

Renal function
Reduced clearance
Compromised  kidney function  may be secondary  to reduced  perfusion,  intrinsic  damage secondary to ischaemia or drug toxicity and immunologic injury
A decrease in GFR would increase the half-life of medications that are renally cleared and may result in drug or metabolite accumulation
Increased clearance
Increased cardiac output in early sepsis increases GFR and increased drug clearance Burns, use of diuretics and hypertonic saline also result in increased GFR and potentially increase clearance

Protein binding changes
Three major proteins affecting drug protein binding – albumin, alpha 1 acid glycoprotein and lipoproteins
Reduced clearance
Some proteins (eg alpha 1-acid glycoprotein binding morphine) are increased in critically ill resulting in reduced clearance
Increased clearance
Albumin is reduced so there will be a higher concentration of free drug for drugs normally bound to albumin resulting in increased clearance
Protein  binding  affected  by other  factors  including  accumulation  of endogenous  binding
inhibitors, qualitative changes on binding sites, competition for binding by other substances, pH changes

Discussion

a) in short, critical illness decreases drug absorption by the following mechanisms:

  • poor stomach emptying rate
  • poorer gut transit time
  • altered pH of the stomach
  • decreased blood flow to the gut
  • decreased venous fow from the gut
  • intestinal wall oedema

b) critical illness may reduce drug clearance by the following mechanisms:

  • decreased spontaneous degradation
    • hypothermia
  • decreased tissue metabolism
    • decreased tissue blood flow
    • hypothermia
  • decreased plasma metabolism
    • due to poor hepatic synthetic function, many serum enzymes responsible for drug removal are not synthetised in appropriate quantities
  • decreased metabolism in the liver
    • decreased hepatic blood flow
    • cytokine-induced decrease in hepatic metabolism
    • hepatic injury
    • hypothermia leading to diminished enzyme function
    • hepatic enzyme inhibition by other drugs
  • increased metabolism in the liver
    • pyrexia leading to increased metabolic rate
    • enzyme activation by other drugs
  • decreased clearance in the urine
    • decreased renal blood flow
    • decreased glomerular filtration rate
    • poor tubular function, decreased active transport
    • acute renal injury eg. ATN
  • decreased clearance in the bile
    • biliary stasis
    • decreased gut transit leading to recirculation
  • increased clearance due to decreased portein binding
    • thus, increased free fraction, which is exposed to clearance mechanisms

References

Boucher, Bradley A., G. Christopher Wood, and Joseph M. Swanson. "Pharmacokinetic changes in critical illness." Critical care clinics 22.2 (2006): 255-271.

Question 29 - 2012, Paper 1

A 25-year-old man presents to the Emergency Department following suspected snake bite. He has an effective pressure-immobilisation bandage in situ.

  • List appropriate initial investigations specific to this presentation that should be performed in conjunction with clinical assessment
  • List indications for the use of polyvalent antivenom in snake envenomation.
  • Briefly discuss the role of pharmacological pretreatment prior to the administration of snake antivenom?
  • List 3 parameters that would help you determine that adequate monovalent antivenom has been administered to a patient with snake bite envenomation.

College Answer

Indications for the use of polyvalent antivenom in snake envenomation:

  • Unable to identify snake … could be due to no AVDK, or equivocal result.
  • Severe envenomation and can’t wait for SVDK result AND would need several monovalent snake antivenoms to cover the possible local snakes.
  • Unavailability of appropriate antivenom.
    • Rapid evolution of life-threatening clinical state (no time to wait for VDK)
    • Unavailability of appropriate monovalent antivenom
    • Equivocal VDK result
    • In setting that antivenom administration is justified
  • Initial Investigations:
    • •   CK
    • •   Coagulation
    • •   Venom detection (bite site if possible), if any clinical or investigation abnormalities are present
    • • ELFTs … renal failure are a complication of rhabdomyolysis and a direct effect of brown snake bite.
    • •   Full blood count … measure platelets

c) Role of pharmacological pretreatment prior to the administration of snake antivenom:

  • Allergic phenomena are common with snake antivenoms and preparation for anaphylaxis is mandated when administering antivenom
  • No evidence for any pretreatment
    • Steroid, antihistamine, adrenaline- all no good evidence
  • Common practice in many centres though

d) Parameters:
Several possibilities here and many controversies:

  • Empiric dose administered – concordant with guidelines / CSL recommendations (that there is variability in these can be acknowledged, as can dose for children = dose for adults). Observation and assessment then required
  • Rise in fibrinogen/ resolution of coagulopathy. Takes time, role of FFP controversial
  • Resolution of neurotoxicity (if presynaptic effect)- if postsynaptic changes are established this will be unreliable
  • Resolution of nonspecific symptoms could also be mentioned, as could halt in CK rise

Discussion

Investigations for a snake bite victim:

  • CK (rhabdmyolysis)
  • Coags (DIC, or "venom-induced consumption coagulpathy)
  • FBC (DIC, looking for thrombocytopenia and red cell fragmentation)
  • Fibrinogen (DIC)
  • EUC (renal failure)
  • LFTs (hepatic injury)
  • Snake Venom Detection Kit

Indications for polyvalent antidote:

  • Unsure which snake species was involved
  • SVDK not available
  • monovalent antivenom not available
  • the patient has been bitten by multiple different species of unidentified snakes.

Evidence for premedication for antivenom administration:

  • This is no longer recommended in Australia
  • polyvalent antidote tends to have a higher rate of anaphylaxis

How do you know your monovalent antivenom is working?

  • The short answer is, you dont.
  • It takes tme for some of the irreversible features to resolve (eg. it takes time to synthesis the coagulation factors which have been depleted)
  • Giving more antivenom will not improve the situation.

References

Isbister, Geoffrey K., et al. "Snakebite in Australia: A practical approach to diagnosis and treatment." Medical journal of Australia 199.11 (2013): 763-768.

 

Question 10 - 2012, Paper 2

A 45-year-old man is admitted to the Emergency Department after ingesting an unknown quantity of “headache tablets”. His initial complaints are nausea, vomiting, shortness of breath and tinnitus. Fluid resuscitation has been commenced. You are asked to assess him as he is getting more dyspnoeic.

His serum biochemistry and arterial blood gas profile are as follows:

Parameter

Result

Normal Range

Sodium

138 mmol/L

135 – 145

Potassium

3.2 mmol/L*

3.4 – 5.0

Chloride

108 mmol/L

100 – 110

Bicarbonate

10 mmol/L*

22 – 27

FiO2

0.3

pH

7.32*

7.35 – 7.45

PO2

125 mmHg (16.4 kPa)

PCO2

20 mmHg (2.6 kPa)*

35 – 45 (4.6 – 6.0)

Base Excess

-10 mmol/L*

-2 – +2

Salicylate level

105 mg/dL*

3 – 10 mg/dL

Paracetamol level

<20 mg/L (<130 µmol/L)

<20 (<130 µmol/L)

  • Describe the acid-base status
  • What are 4 severe complications of this toxidrome?
  • What coagulopathy may be present in this toxidrome and what is the treatment?
  • What are the treatment options for severe toxicity, and what is their rationale?
 

College Answer

a) 
Acid-base status:

  • Increased anion gap metabolic acidosis
  • Concomitant normal anion gap metabolic acidosis
  • Respiratory alkalosis
  • Decreased delta ratio

b)

  • Hypoglycaemia
  • Pulmonary oedema
  • Cerebral oedema
  • Arrhythmias
  • Hyperpyrexia

c)

Hypoprothrombinaemia

Vitamin K

d)

Forced alkaline diuresis. Renal excretion of salicylates becomes important when the metabolic pathways become saturated. There is a 10-20 fold increase in elimination when the urine pH increased from 5 to 8

Haemodialysis. Most of the drug is protein-bound, and is concentration dependant. The volume of distribution is small, and binding site saturation leads to large levels of free drug, which is easily dialysable

Multiple-dose charcoal. Many aspirin forms are slow release and after ingestion they clump together in the GI tract, forming a large slow release preparation. It is also poorly soluble in the stomach leading to delayed absorption.

Discussion

a)

The change in anion gap is 10, and the drop in bicarbonate is 14, which gives a delta ratio of 0.8, suggesting that there is a mixed high anion gap and normal anion gap metabolic acidosis.

There is indeed a respiratory alkalosis, which is appropriate (the rules of compensation suggest that the CO2 should be about 23).

b)

Salicylate toxicity has a whole list of complications. The college had asked specifically for severe ones. One may conceive of a respiratory alkalosis so dramatic as to warrant this adjective, and the same can be said for just about any other complication of salicylate toxicity, so they are all listed here.

Serum level 30-50mg/dL: Serum level 50-75mg/dL: Serum level >75mg/dL:
  • Tachypnoea
  • Respiratory alkalosis
  • Nausea
  • Vomiting
  • Tinnitus
  • Dizziness
  • Tachypnoea
  • Respiratory alkalosis
  • Fever
  • Sweating
  • Dehydration
  • Agitation
  • Coma
  • Hallucinations
  • Seizures
  • Cardiogenic shock
  • Pulmonary oedema
  • Coagulopathy: It is known that salicylate toxicity can cause a decrease in prothrombin. Vitamin K (if not prothrombinex) is the answer.
  • Oliguria and renal failure.
  • Ketoacidosis 
  • Lactic acidosis (due to uncoupling of oxidative phosphorylation)

c)

It is known that salicylate toxicity can cause a decrease in prothrombin.

Vitamin K (if not prothrombinex) is the answer.

d)

Severe toxicity from salicylates has several treatment options:

Decontamination

  • Multiple dose activated charcoal is recommended by the UpToDate toxicology authors. Aspirin is well adsorbed by charcoal. Three 25g doses separated by two hours is the recommebded regimen.
  • Whole bowel irrigation is relevant in the context of sustained release preparations, and has been useful in animal models.

Direct  and indirect antidotes

  • There is nothing specific. Urinary alkalinisation is generally held to be the nearest thing to a direct antidote.

Enhancement of clearance

  • Alkalinise the urine. This is vital. An alkaline blood environment also prevents the movement of salicylate into the CSF.  Raising the urine pH from 5 to 8 can increase total salicylate excretion by twenty times.
  • Haemodialysis may be required in severe cases, particularly where you cannot give any more bicarbonate (i.e. the patient is already fluid overloaded) or where the overdose is supermassive (levels in excess of 100mg/dL). Even though salicylate is highly protein bound this technique can usually move eough molecules to make a difference. One must also keep in mind the nonlinear kinetics of elimination - the higher the dose, the longer the half-life, and therefore the more prominent the effects of extracorporeal clearance.

Supportive ICU therapies

  • Intubation may be indicated, but must be carried out carefully (see next point)
  • Mechanical (hyper)ventilation  will be required: if the patient ends up being intubated, their minute volume must be maintained at least as high as it was prior to intubation. Respiratory alkalosis keeps the salicylate ions trapped in the blood; if a post-intubation acidosis is allowed to develop the sudden influx of salicylate into the CNS may cause seizures, cerebral oedema and death.
  • Vasopressors and inotropes  may be useful in some cases, but in the majority of cases the patient will be hypotensive because of volume depletion.
  • Supplemental glucose: these people are neuroglycopenic at normal BSL, and so the BSL should be kept at the higher range of normal.
  • Correction of hypokalemia is vital, because hypokalemia promotes K+ reabsorption at the distal tubule (where K+ is exchanged for H+, i.e. its reabsorption is coupled to acid secretion). Ergo, hypokalemia interferes with the attempt to alkalinise urine, and therefore inhibits salicylate clearance.

References

O'Malley, Gerald F. "Emergency department management of the salicylate-poisoned patient." Emergency medicine clinics of North America 25.2 (2007): 333-346.

Pinedo, H. M., L. B. van de Putte, and E. A. Loeliger. "Salicylate-induced consumption coagulopathy." Annals of the rheumatic diseases 32.1 (1973): 66.

Shapiro, Shepard, Milton H. Redish, and Harold A. Campbell. "Studies on Prothrombin: IV. The Prothrombinopenic Effect of Salicylate in Man."Experimental Biology and Medicine 53.2 (1943): 251-254.

Pearlman, Brian L., and Rashi Gambhir. "Salicylate Intoxication." Postgraduate medicine 121.4 (2009).

Question 15.4 - 2012, Paper 2

The following is an image from an abdominal CT scan taken of a 24 year old man who presented with a carbamazepine overdose.

something

What complication has occurred?

College Answer

Gastrointestinal obstruction secondary to multi dose charcoal administration.

Discussion

It is surprisingly difficult to find a CT scan of a charcoal bezoar. One might think that carbamazepine+charcoal+"CT abdo" would be a specific enough search string to find the exact image in the ind of the examiner, as the scenario described here simpoly screams "case report", and in fact that is exactly what you get; except the case report was published seven years after this SAQ came outAljohani et al (2019) describe a 22-year-old patient who had received multiple-dose activated charcoal for carbamazepine intoxication. The CT, shown above, demonstrated "small bowel obstruction to the level of the proximal ileal loops, with a transition point between the dilated proximal loops and the collapsed terminal ileal loops"

References

Aljohani, Turki Khaled, et al. "A rare case of small bowel obstruction secondary to activated charcoal administration." Journal of surgical case reports 2019.2 (2019): rjz033.

Watson, William A., Karl F. Cremer, and James A. Chapman. "Gastrointestinal obstruction associated with multiple-dose activated charcoal." The Journal of emergency medicine 4.5 (1986): 401-407.

Goulbourne, Karita Boyd, and James E. Cisek. "Small-bowel obstruction secondary to activated charcoal and adhesions." Annals of emergency medicine 24.1 (1994): 108-110.

Chan, Justin CY, Chaminda Saranasuriya, and Bruce P. Waxman. "Bezoar causing small bowel obstruction after repeated activated charcoal administration." Medical Journal of Australia 183.10 (2005): 537.

Question 4 - 2013, Paper 1

What key cardiac effects are observed with acute digoxin toxicity? List two rhythm disturbances highly associated.

List three drugs known to enhance digoxin serum level. Provide a mechanism for each.

Other than drugs, what other factors are known to exacerbate digoxin toxicity?

With respect to the use of digoxin specific Fab fragments:

  • Outline your indications for use in suspected acute digoxin toxicity.
  • Total serum digoxin level continues to remain high after the administration of an appropriate dose of digoxin specific Fab fragments. What action would you take and why?

College Answer

a)

Key cardiac features are increased automaticity combined with AV conduction block.

Rhythms suggestive: PAT with variable block 
Accelerated junctional rhythms 
Bidirectional ventricular tachycardia (specific for Digoxin). 
Other (a variety are seen): SA node arrest, premature ventricular contractions, bradycardia, non paroxysmal junctional tachycardia, AV nodal blockade, ventricular tachycardia, ventricular flutter and fibrillation.

Note: Features of digoxin effect (e.g. T wave flattening/ inversion) do not correlate well with toxicity.

b)

Verapamil, Diltiazem, Amiodarone via inhibition of P-glycoprotein (efflux pump that excretes many drugs, including Digoxin, into the intestine or proximal renal tubule) - effectively reducing renal and GI secretion.

Erythromycin, omeprazole via increased Digoxin absorption.

c)

Low potassium, magnesium, pH, high calcium.

d)

Early recognition of toxicity and prompt administration of Fab fragments essential for severe poisoning. The serum Digoxin concentration does not necessarily correlate with toxicity.

Indications Include:

Life threatening arrhythmia with cardiovascular instability 
Evidence of end organ dysfunction 
Hyperkalaemia (> 5.0 – 5.5 mEq/l) 
Ingestion of 10mg or more in total

After Fab administration free Digoxin levels are decreased to zero within minutes. Total Digoxin level will increase markedly since assays measure bound and free. Bound fraction rises due to an increase in Digoxin-Fab complex. These high levels have no correlation with toxicity and the serum level may be unreliable for several days and no action should be taken based on total level after digoxin-specific Fab fragments administration.

Discussion

a)

The features of digoxin toxicity can be divided into cardiac and non-cardiac.

  • Cardiac:
    • Bradycardia
    • AV block
    • Ventricular ectopics
    • Tachyarrhytmia- pretty much any variety which does not involve rapid AV conduction
    • Bidirectional ventricular tachycardia is ridiculously rare, but digoxin seems to be among the few drugs which can actually produce this.
  • Non-cardiac:
    • Nausea/vomiting
    • Abdominal pain
    • Diarrhoea
    • Weakness
    • Confusion
    • Xanthopsia (seeing yellow)
    • Hyperkalemia (in acute overdose)

b)

Drug interactions of digoxin are a massive topic. The ones which result in overdose can be divided into inhibition of clearance (by inhibition of P-glycoprotein ) and increase of absorption.

  • P-glycoprotein inhibitors:
    • Calcium channel blockers like verapimil and diltiazem
    • Spironolactone
    • Quinidine
    • Amiodarone
  • Absorption enhancers
    • Macrolides (by killing gut bacteria which normally digest some of the orally administered digoxin)
    • Proton-pump inhibitors (by increasing the permeability of the gastric mucosa)

c)

Digoxin toxicity is exacerbated by the following factors:

  • Hypokalemia
  • Hypomagnesemia
  • Hypercalcemia
  • Acidosis

d)

Indications for the use of digoxin-specific Fab fragments are strange.

Life-threatening arrhythmia, hyperkalemia and altered mental status are mentioned, but the article in UpToDate recommends that digoxin antibodies be used in every poisoning, because there is no therapy with a comparable efficacy and safety.

"Total serum digoxin level continues to remain high after the administration of an appropriate dose of digoxin specific Fab fragments. What action would you take and why? "

One appropriate action would be to do nothing. The digoxin assay measures the total digoxin, whereas the free digoxin level after Fab may in fact be reduced to nearly zero. One is then confronted with a situation where the measured digoxin level is still very high, but the patient  looks perfectly fine.

In such a situation, one should ignore the total level. I thank Yun from Canberra for pointing out the error in my initial reading of this question. If the clinical features of toxicity have resolved, the total digoxin level is meaningless. If they have not resolved, the patient requires another dose of the specific Fab fragments. If for whatever reason this is inadewuate, one may attempt resin hemoperfusion. However, this is not universally acknowledged as a useful strategy. Fab fragments together with plasmapheresis is another experimental technique.

References

UpToDate has a nice article.

Hauptman, Paul J., and Ralph A. Kelly. "Digitalis." Circulation 99.9 (1999): 1265-1270.

Marcus, Frank I. "Pharmacokinetic interactions between digoxin and other drugs." Journal of the American College of Cardiology 5.5s1 (1985): 82A-90A.

Gabello, M., et al. "Omeprazole induces gastric permeability to digoxin."Digestive diseases and sciences 55.5 (2010): 1255-1263.

Juneja, Deven, et al. "Severe suicidal digoxin toxicity managed with resin hemoperfusion: A case report." Indian journal of critical care medicine: peer-reviewed, official publication of Indian Society of Critical Care Medicine 16.4 (2012): 231.

Hauptman, Paul J., and Ralph A. Kelly. "Digitalis." Circulation 99.9 (1999): 1265-1270.

Question 17 - 2013, Paper 1

List the pharmacological characteristics of phenytoin and outline how these impacts on its clinical use.

College Answer

Pharmacology

Impact on clinical use

Formulation: oral tablets or suspension 
or colourless solution (with a carrier 
agent). Intravenous preparation is 
stable in normal saline for a short time.

Administration: oral or slow intravenous injection 
(over 20 min). Should not be given IM as 
precipitates in tissue, with erratic absorption. 
Carrier agent thought to be responsible for some 
of the adverse effects of parenteral phenytoin 
and fosphenytoin may be preferable where 
available.

Good oral bioavailability (85%). Rate 
of absorption (although not total 
absorption) reduced by enteral 
feeding.

Enteral and intravenous doses are the same

Distribution: approximately 90% 
protein bound (to albumin).

Free drug concentration may be therapeutic in 
the face of low total drug concentration in 
hypoalbuminaemic patients.

Elimination predominantly hepatic
Half life approx 22 hours (7-42 hours)

Several days to reach a new steady state and 
loading dose is required for patients in whom 
rapid attainment of therapeutic concentrations is 
required. Adjustments to maintenance doses 
should only be made every 4-7 days.

Kinetics: first order at low 
concentrations but zero order within 
the therapeutic range.

Changes in maintenance doses should be small.

Mode of action: 
Blocks voltage gated neuronal sodium 
channels.

Clinical use as antiepileptic drug predominantly 
for generalized or partial seizures (particularly 
status epilepticus) 
Also for prevention/treatment of torsades in long 
QT syndrome

Hepatic enzyme inducer

Increased metabolism and lower drug 
concentrations of many drugs including other
anti-epileptics, warfarin, benzodiazepines, 
cyclosporine, theophylline

Other interactions

Phenytoin concentrations may be increased by 
fluconazole, omeprazole

Adverse effects: 
IV administration: heart block, 
hypotension 
Dose related neurotoxicity: slurred
speech, impaired memory, headache, 
nystagmus, impaired coordination,
confusion 
Haematological effects 
Blood dyscrasias 
Megaloblastic anaemia 
Acute hypersensitivity reactions – rare
(anticonvulsant hypersensitivity 
syndrome [AHS} hepatotoxicity, 
DRESS, Stevens Johnson syndrome) 
Less important to mention: rash, acne, 
gingival hyperplasia

Decrease rate of infusion, monitor blood pressure 
and ECG during infusion 
Check phenytoin concentration and reduce dose 
if appropriate 
Responds to folate supplementation 
Stop drug promptly with severe hypersensitivity 
reactions and avoid other aromatic anticonvulsants.

Discussion

Phenytoin
Properties Influence on ICU management
Chemical properties
  • Sodium salt has decent water solubility
  • The injectable solution has a pH of 12, and is 40% propylene glycol (and 10% alcohol)
  • Cannot be given IM: as a depot, the phenytoin precipotates and is absorbed slowly and erratically
  • When given IV, can be very irritating, and needs to be diluted
  • Can only be administered together with saline
Chemical relatives
  • A derivative of hydantoin
  • Other relatives are danrolene and fosphenytoin
  • Cross-hypersensitivity may occur
Administration
  • IV or PO/NG
  • Conveniently available as an oral suspension for NG administration
Absoption
  • Well absorbed orally
  • Bioavailability is ~ 85% (but varies depending on the manufacturer)
  • Food influences absorption
  • Usually no need to adjust dose when changing from IV to oral
  • Need to cease feeds when administering (2hrs before, 2hrs after) - enterally fed patients should be getting IV phenytoin.
Distribution
  • Volume of distribution: 0.5-1.0L/kg
  • Susceptible to removal by plasmapheresis and haemoperfusion
Protein binding
  • Highly protein bound: 90%
  • Not susceptible to haemodialysis
  • Higher free levels are to be expected in patients with low albumin (even though total levels may be normal or low)
Metabolism/elimination
  • Hydroxylated in the liver by a saturable enzyme system
  • Enhances its own elimination through enzyme induction
  • After administration, most of the drug is excreted in the bile as inactive metabolites which are then reabsorbed from the intestine and excreted in the urine
  • Zero-order kinetics after enzymes are saturated, and first-order before.
  • Enterohepatic recirculation means there may be a theoretical benefit to multiple dose activate charcoal administration
  • Small dose increases may significantly increase the half-life and levels once the enzyme system is saturated
Half life
  • mean plasma half-life is 22 hours
  • Optimal for once daily dosing
Mechanism of action
  • "membrane stabiliser"
  • blocks voltage gated sodium channels
  • Most of the antiepileptic effect takes place at the motor cortex, where it inhibits the spread of seizure activity possibly by promoting sodium efflux from neuron
  • Can also be used as a Class 1 antiarrhythmic
Indications
  • Control of generalised tonic-clonic seizures
  • Prevention of post-TBI seizures
  • Cheap and familiar
Contraindications
  • Levitiracetam is an (expensive) alternative
Adverse effects
  • Rash (5% to 10% )
  • Gum hypertrophy
  • Ataxia, nystagmus, slurred speech
  • Confusion
  • Drug-induced lupus
  • Agranulocytosis
  • Aplastic anemia
  • Hepatitis
  • Anticonvulsant hypersensitivity syndrome
  • May consider another agent in a patient with already poor bone marrow function
Interactions
  • Phenytoin is an inducer of the hepatic cytochrome P450 microsomal isoenzymes CYP3A4, CYP2D6, CYP1A2, CYP2C9 and CYP2C19
  • Phenytoin is metabolised primarily by
    CYP2C9 (major) and CYP2C19 (minor), thus several drugs may inhibit or induce the
    metabolism of phenytoin

Common ICU drugs which interfere with phenytoin:

  • Amiodarone (increased level)
  • Erythromycin (increased level)
  • Fluconazole (increased level)
  • Rifampicin (decreased level)
  • Folate supplements (decreased level)
  • Fluoroquinolones (decreased level)
Acute overdose
  • Nystagmus appears at 20 µg/mL
  • Ataxia appears at 30 µg/mL
  • Nystagmus appears at 20 µg/mL
  • The lethal dose in adults is estimated to be 2 to 5g. 
  • In overdose settings, saturation of the hepatic hydroxylation system occurs and zero order kinetics predominate
  • Activated charcoal is recommended in life-threatening overdose
  • Dialysis is of no benefit
  • Haemoperfusion or plasmapheresis may have some benefit

References

Question 8 - 2013, paper 2

A two-year-old boy is suspected of ingesting iron tablets.

a)List the clinical features, and the underlying pathophysiology, of iron poisoning.

b) Briefly outline your management of this child.

College answer

a)


Clinical Feature

Mechanism

Nausea, vomiting, diarrhoea

Direct corrosive effect on GIT

Abdominal pain

Direct corrosive effect on GIT

Gut ischaemia

Disruption of cellular metabolism

Shock

Fluid losses from GIT

3rd space losses and vasodilatation

Anion gap metabolic acidosis

Disruption of cellular metabolism

Acute liver failure

Disruption of cellular metabolism

Shock and hypovolaemia

Jaundice, coma, low BSL, coagulopathy

Hepatic necrosis

Renal failure

Disruption of cellular metabolism

Oliguria

Shock and hypovolaemia

  • b)

    • Management consists of:
      • Resuscitation as indicated with concurrent specific assessment and management of the toxidrome.
      • Resuscitation:
        • ABCs 
        • Priority is early restoration of circulating volume 
        • Boluses of 10-20 ml/kg crystalloid and assess response
    • Assessment for signs and symptoms indicative of iron toxicity.
    • Risk assessment:
      • History of ingestion – type, quantity of tablets and time of ingestion
        • Iron preparations differ in the amount of elemental iron contained.
        • < 20 mg/kg elemental iron is asymptomatic
        • 20 – 60 mg/kg causes GI symptoms
        • > 60 mg/kg causes systemic toxicity
        • > 120 mg/kg is potentially lethal
        • Children rarely ingest more than 60 mg/kg.
      • Specific investigations
        • BSL
        • Serum iron level
        • ABG
        • AXR – useful in confirming ingestion
    • Disposition
      •  Asymptomatic at 6hr and negative AXR may be discharged home
      • Monitoring and treatment in paediatric centre (ward, HDU, ICU depending on severity)
    • Ongoing assessment of response to resuscitation and antidotes.
    • Antidotes
      • Desferrioxamine chelation therapy in cases of systemic toxicity (high serum iron level or metabolic acidosis on ABG)
    • Decontamination
      • Iron not absorbed to activated charcoal
      • Whole bowel irrigation indicated for confirmed ingestions > 60 mg/kg – difficult and potentially hazardous in 2-year-old
      • Surgical or endoscopic removal of tablets if lethal ingestion or WBI not feasible

Discussion

The pediatric aspect of this question does not feature prominently in the answer. The only time it is mentioned is in the discussion of whole bowel irrigation, and how foolish it would be to subject a two-year old to this.

a) is well presented by the college.

A flowchart of the mechanisms of high anion gap metabolic acidosis due to iron poisoning is presented elsewhere.

I will reproduce it here, for convenience.

mechanism of metabolic acidosis due to iron overdose

Feature Causes
   
Tachypnoea
  • Metabolic acidosis
Shock, circulatory collapse
  • Third space fluid losses
  • Blood and fluid loss from the ulcerated gut
  • Cardiotoxic effects, with cardiogenic shock
  • Vasodilation due to SIRS
Hypoglycaemia
  • Acute hepatotoxicity
Coma
  • Hypoglycaemia
  • Acute cerebral oedema due to liver failure
High anion gap metabolic acidosis
  • Lactic acidosis
  • Ketosis
  • Minor contribution from iron itself (conversion of  Fe3+ to Fe2+ produces a net loss of a cation, and therefore contributes to the decrease in the SID)
Hyperlactatemia
  • Acute hepatotoxicity and liver failure
  • Shock state
  • Direct mitochondrial toxicity
Renal failure
  • Shock state
  • mitochondrial (tubular) toxicity, ATN
Gastric ulceration
  • direct corrosive effect of the drug
Haemorrhage, melaena
  • from ulcerated gut surface

Toxicity manifests in four stages:

  • Stage I:  GI toxicity (0-6 h since ingestion): vomiting, haematemesis, abdominal pain and lethargy
  • Stage II: "apparent stabilization" (6-12 h since ingestion) - symptoms subside
  • Stage III:  mitochondrial toxicity and hepatic necrosis (12-48 h since ingestion)- acute liver failure, coagulopathy, acute tubular necrosis, metabolic acidosis and shock.
  • Stage IV: GI scarring (4-6 weeks since ingestion) - gastric scarring and pyloric stricture

b) A systematic approach to an answer would resemble the following:

  • Immediate management:
    • ABCs
    • Circulatory support with fluid resusicitation and inotropes if indicated
  • Diagnostic studies
    • ABG - to assess extent of acidosis
    • AXR - to directly visualise the bezoar
    • Serum iron level

Decontamination

  • Activated charcoal has no role to play
  • Whole bowel irrigation - until effluent turns clear - is a good strategy; much of the toxicity is related to gut ulceration, and by diluting the iron in the gut lumen you may be able to ameliorate this direct corrosive effect, even if you don't manage to prevent toxic absorption.
  • Surgical removal of tablets - if a bezoar is clearly visible on the AXR

Enhanced elimination

  • Exhange transfusion: the removal of iron-poisoned blood is ery old-school, but it works (Movassaghi et al, 1969)
  • Haemodialysis can be considered to help remove the iron-desferrioxamine complexes, as they are renally excreted and there may not be enough renal function to remove this product. Otherwise, apart from correcting acidosis there is no role for dialysis.

Specific antidote

Supportive care

  • Intubation will likely be required to protect the airway not only from the decreased level of consciousness but also from the risks of aspiration associated with whole bowel lavage.
  • Mechanical ventilation will likely be with mandatory mode, to decrease the demands on the failing myocardium
  • Circulatory support should consist of simultaneous fluid resuscitation, inotrope and vasopressor infusions
  • Sedation should be rationalised, given that the patient is already in a coma before the sedation is given, and that the liver is doing little metabolically.
  • Correction of acidosis with bicarbonate may be indicated if catecholamine responsiveness is lost.
  • Electrolyte replacement -losses must be anticipated, the leaky gut and bowel lavage will result in potassium and phosphate depletion.
  • Haemodialysis may be required to maintain metabolic normality, as well as to remove ammonia which may accumulate due to the acute hepatocellular necrosis
  • Hypoglycaemia and ketosis will likely develop. The patient will need a dextrose infusion, as hepatic and skeletal muscle glycogen stores will be depleted.
  • Nutrition will likely be parenteral for some time, depending on the extent of gastric ulceration.
  • Coagulopathy will develop due to hepatocellular necrosis. Coagulation factor replacement will be required.

References

The Royal Childrens Hospital has a good set of guidelines for irone overdose.

Abhilash, Kundavaram PP, J. Jonathan Arul, and Divya Bala. "Fatal overdose of iron tablets in adults." Indian journal of critical care medicine: peer-reviewed, official publication of Indian Society of Critical Care Medicine 17.5 (2013): 311.

REISSMANN, KURT R., and THOMAS J. COLEMAN. "Acute Intestinal Iron Intoxication II. Metabolic, Respiratory and Circulatory Effects of Absorbed Iron Salts." Blood 10.1 (1955): 46-51.

REISSMANN, KURT R., et al. "Acute Intestinal Iron Intoxication I. Iron Absorption, Serum Iron and Autopsy Findings." Blood 10.1 (1955): 35-45.

Question 18.1 - 2013, paper 2

A 62-year-old female is brought into hospital with suspected organophosphate poisoning.

a) List six acute clinical features associated with this condition.

b) List the antidotes indicated in this condition and the rationale for their use.

The following data are taken from this patient:

Parameter

Patient Value

Normal Adult Range

Cholinesterase

0.3 KU/L*

3.4 – 9.0

Cholinesterase mixing

33%*

100%

c)    What does the result of the mixing test indicate?

College Answer

a)

  • Diarrhoea
  • Urination
  • Miosis
  • Bronchospasm
  • Bronchorrhoea
  • Emesis
  • Lachrymation
  • Salivation
  • Fasciculations
  • Tremor
  • Weakness
  • Respiratory muscle weakness
  • Bradycardia (tachycardia may be present)
  • Hypotension
  • Agitation
  • Coma
  • Seizures

b)

  • Atropine to control clinical features of cholinergic excess – anti-muscarinic. Large doses may be required
  • Pralidoxime to reactivate acetyl choline esterase – only effective before irreversible binding or “ageing” takes place

c)

  • This mixing test is suggestive of free organophosphate present in the blood OR inadequate dose of pralidoxime.

Discussion

The first part of the question asks the candidate to produce 6 features of the cholinergic toxidrome. This should be a piece of cake. One recalles the mnemonic SLUDGEM:

  • Salivation
  • Lacrimattion
  • Urination
  • Diarrhoea
  • Gastrointestinal upset
  • Emesis
  • Miosis

The college answer does not lend itself well to being so easily memorised, and has broken at least one anagram engine. However, Yun from Canberra has pointed out that it is taken directly from the Australian Toxicology Handbook. The first six points are DUMBBELS (the muscarinic features), and the rest are nicotinic.  

b)

Atropine and pralidoxime were asked for. The brevity of the college answer cannot be improved upon.

c)

In the mixing test, the patients serum and some random reference serum are both tested for plasma cholinesterase, and then a 50-50 mixture of the two is tested.

If there is enough pralidoxime being given, there will be little free organophosphate in the patient's sample, and the mixed sample will have a plasma cholinesterase level which is exactly between the patients sample and the reference sample.

If there is still free organophosphate present, then it will disable the plasma cholinesterase in the reference sample, and the cholinesterase level of the mixed sample will be surprisingly low.

References

Brian Kloss from LITFL has a superb cartoon to illustrate the horrors of the cholinergic toxidrome.

Sungur, Murat, and Muhammed Güven. "Intensive care management of organophosphate insecticide poisoning." Critical care 5.4 (2001): 211.

Kamanyire, R., and L. Karalliedde. "Organophosphate toxicity and occupational exposure." Occupational Medicine 54.2 (2004): 69-75.

Jr, Bailus Walker, and Joseph Nidiry. "Current concepts: organophosphate toxicity." Inhalation toxicology 14.9 (2002): 975-990.

Question 9 - 2014, Paper 1

With respect to the management of a 35-year-old female presenting with toxicity secondary to deliberate self-harm with paracetamol:
 
a) Outline how paracetamol causes liver dysfunction and how N-acetylcysteine (NAC) works as an antidote in this setting.
 
b) List the criteria for liver transplantation in this patient.
 
c) Outline your management of the patient in the event of clinical deterioration, pending transfer to the regional centre for liver transplantation.

College Answer

 
a)
 Paracetamol is predominantly conjugated into glucuronate and sulphate moeities
 Small percentage is metabolized by cytochrome P450 to a toxic metabolite NAPQI, N-acetyl-p-benzoquinone imine (also known as NABQI).
 Amount of NAPQI will vary according to genetic profile.
 NAPQI is conjugated with glutathione to non-toxic moieties.
 In paracetamol toxic ingestion the phase 2 conjugation enzymes are saturated so a higher fraction is converted to the toxic metabolite.
 Conjugation of NAPQI with glutathione continues until it is depleted.
 Toxic NAPQI accumulates and causes direct damage to hepatocytes.
 NAC is a glutathione surrogate that detoxifies the toxic metabolite of paracetamol
 NAC is converted to glutathione increasing the sulphation of paracetamol which prevents formation of the toxic metabolite blunting the localised inflammatory response in the liver.
 
b)
 Arterial pH < 7.3 or lactate > 3.0 mmol/L after adequate resuscitation
OR
 If all 3 of the following occur within a 24 hour period
 Creatinine > 300 μmol/L
 PT >100 seconds (INR > 6.5)
 Grade 3 – 4 encephalopathy

c)
 General supportive care with specific liver supportive therapy
 Continue NAC
 Ventilate as required
 Normocarbia
 Support the circulation
 Fluids cautiously to avoid worsening cerebral oedema
 Catecholamines / vasopressors
 Early CRRT for renal failure
 Control of acidaemia
 Control of fluid balance
 Avoid fever
 Commence nutrition involving liver specific feeds with low amino acids
 Lactulose 30 mL mg tds with other aperients
 Thiamine loading large dose 300 mg iv tds
 Regular vitamin K 10 mg iv daily
 Avoid FFP unless requiring coverage for invasive procedures
 Surveillance for infection and early antibiotic therapy if required
 Stress-ulcer and DVT prophylaxis
 Avoid hypoglycaemia
 Control ICP
 
Examiners' comments: Candidates who did not pass gave sparse answers without sufficient detail, e.g. answer to part (b) was given as "King's College criteria" without further explanation.

Discussion

a)

Mechanism of paracetamol toxicity is discussed elsewhere. Special attention is also given to the mitochondrial toxicity of paracetamol, which gives rise to lactic acidosis. In brif:

  • Most paracetamol is metabolised by glucouronidation and sulfation
  • Some (~5%) is metabolised by CYP2E1
  • In the course of this, superoxide and NAPQI are generated
  • In the presence of ample glutathione, NAPQI is rapidly detoxified by conjugation
  • In the presence of massive overdose, glutathione is rapidly depleted
  • As NAPQI levels increase, it binds covalently to numerous proteins, causing toxicity
  • Of particular interest is the uncoupling of oxidative phosphorylation, which results in a failure of ATP synthesis, lactic acidosis, and the release of ionised calcium from mitochondrial stores
  • The consequence of this is hepatocellular apoptosis and necrosis.
  • NAC is converted to glutathione, replenishing the reserves.
  • Cysteine, the midproduct of metabolism, also supplies ample sulfate for the sulfation of paracetamol (so less of it goes down the toxic MEOS pathway).
  • There are also theoretical antioxidant benefits

b)

The examiners complained that the trainees merely mentioned the King's College criteria by name. The model answer lists the actual criteria, implying that the trainees are expected to memorise them.  The whole issue of prognostication in acute liver failure is discussed elsewhere, and the abovementioned criteria are only one of the possible ways of prognosticating - presumably, somebody who mentioned the MELD criteria would have also received a few marks.

King's College (O'Grady) Criteria - for paracetamol overdose

  • pH of 7.3 on ABG, following fluid resuscitation, more than 24 hours post ingestion
  • OR
    • PT over 100 seconds (INR 6.5)
    • Creatinine over 300mmol/L
    • Grade 3-4 encephalopathy
    • All of these must be present within a 24 hour timeframe

King's College (O'Grady) Criteria - for NON-paracetamol acute liver failure

  • pH of 7.3 on ABG, following fluid resuscitation
  • OR
    • PT over 100 seconds (INR 6.5)
  • Alternatively,
    • Encephalopathy AND
      • Age over 10, or over 40
      • Bilirubin over 300 mmol/L
      • More than 7 days separate onset of jaundice from onset of encephalopathy
      • Aetiology is seronegative hepatitis, or a drug-induced hepatitis

c)

Management plans should include the following points: 

  1.  Intubate the patient for airway protection, as they will be obtunded (and to control the CO2). The patient with fulminant liver failure is very likely to require intubation at some stage.
  2. Hyperventilation to low-normal PaCO2 (35 mmHg) is a part of HHHH therapy, and is mainly directed to prevent the cerebral vasodilation and intracranial hypertension which is associated with acute liver failure.
  3. Vasopressor support: Maintain haemodynamic stability using noradrenaline preferentially. These patients will vasodilate extensively. It will be important to avoid fluid overload because that tends to impair hepatic venous outflow.
  4. Sedation with short-acting drugs. Use propofol instead of benzodiazepines, and avoid long-acting opiates.
    Consider an ICP monitor. This offers you a means of monitoring the development of massive and undetected ICP fluctuations, presumably because you will react to them with more HHHH therapy. It is surprisingly safe - Rajajee et al (2017) trialled a protocol where all their acute liver patients got an ICP monitor and found that only one of the 24 had an intracranial haemorrhage (which was apparently asymptomatic and had "an excellent outcome"). All patients were given some Factor VIIa no more than 1hr prior to the procedure, and they all used delicate little intraparenchymal monitors rather than EVDs.
    Management of raised ICP for these patients does not differ significantly from what you'd normally do for stroke or TBI. If the ICP is uncontrollable by normal means, various extreme authors have suggested various extreme measures. Ringe et al (1988) found that the dying liver was causing more harm than good, and removed it entirely ("we considered it a potential advantage to tolerate a prolonged anhepatic period before implantation of a functioning allograft than to leave the necrotic liver in situ")
  5. Hypothermia to a temperature of 32-33° - 
    Hypernatremia (to control ICP, with hypertonic saline) to achieve a sodium of 145-155 mmol/L
    Replace phosphate: hypophosphataemia tends to develop in the recovery phase, as hepatocytes reproduce vigorously. As a major intracellular anion, phosphate will be sucked up into the rapidly growing cells. Also you need it to make use of all that glucose you are infusing. In summary, give phosphate.
  6. Haemodiafiltration - continuously - to remove ammonia. This prevents acute cerebral oedema. In hyperacute liver failure cerebral oedema may actually develop before hepatic encephalopathy or other major complications of liver failure. Don't use citrate.
    Fluid resuscitation with crystalloid - keeping in mind that fluid overload is undesirable. EASL guidelines (2017) recommend something buffered with acetate.
  7. Give 10-50% dextrose as infusion. Anticipate hypoglycaemia and increased resting energy expenditure. Caloric requirements are  increased by 18 to 30% compared with normal controls (Schneeweiss, 1992) and there does not seem to be any special increase the the metabolism of any specific macronutrient group. The authors of that 1992 study estimated that normal hepatic glucose release  rate is about 8μmol/kg/min, and so one should aim to duplicate this with exogenous glucose in the anhepatic patient. That ends up being about 6-7g/hr of dextrose for a 70kg patient, or approximately 125-150ml/hr of 5% dextrose.
    Give enteral lactulose for management of hepatic encephalopathy. 
    Ensure PPIs are administered. Don't give them any excuse to have a GI bleed.
    Drain the tense ascites. It behaves like a gravid uterus, from a haemodynamic standpoint. If there is tense ascites, draining it could potentially improve venous return and haemodynamics.
  8. Anticipate coagulopathy. Administer Vitamin K empirically, however little that is expected to have. Apparently the correct dose is 10mg. Pereira et al (2005) gave this dose to their patients and found that 27% had some sort of subclinical Vitamin K deficiency. Hard to say whether this had any positive effect on their INRs, let alone survival
    Consider blood products, but view complete correction as unobtainable.
    Remember: in spite of apparent "numerical" coagulopathy, a hypercoagulable state develops.
  9. Vigilant surveillance for sepsis: they are prone to it, and it makes the encephalopathy worse. The EASL guidelines (2017) recommend daily surveillance cultures.

References

O’Grady, John G., et al. "Early indicators of prognosis in fulminant hepatic failure." Gastroenterology 97.2 (1989): 439-445.

Daly, Frank FS, et al. "Guidelines for the management of paracetamol poisoning in Australia and New Zealand-explanation and elaboration." Medical journal of Australia 188.5 (2008): 296.

Dhiman, Radha K., et al. "Early indicators of prognosis in fulminant hepatic failure: An assessment of the Model for End‐Stage Liver Disease (MELD) and King's College Hospital Criteria." Liver transplantation 13.6 (2007): 814-821.

Yantorno, Silvina E., et al. "MELD is superior to King's college and Clichy's criteria to assess prognosis in fulminant hepatic failure." Liver transplantation13.6 (2007): 822-828.

Question 10.1 - 2014, paper 2

A 58-year-old farmer with a history of depression was found collapsed in his shed. On arrival at the Emergency Department, his Glasgow Coma Scale score was 10 (E2, V3, M5), respiratory rate was 23 breaths per minute, and mouth ulceration was noted with a green coloured substance staining his lips, hands and clothes.

His arterial blood gas and biochemistry on admission were as follows:

Parameter Patient Value Normal Adult Range
FiO2 0.5  
pH 7.29* 7.35 – 7.45
PCO2 35 mmHg (4.6 kPa) 35 – 45 (4.6 – 6.0)
PaO2 68 mmHg (9.0 kPa)
HCO3 16 mmol/L* 24 – 28
Base Excess -9.0 mmol/L* -2.0 – +2.0
Sodium 140 mmol/L 135 – 145
Potassium 4.3 mmol/L 3.5 – 5.0
Chloride 111 mmol/L* 95 – 105
Glucose 7.2 mmol/L* 4.0 – 6.0
Lactate 5.2 mmo/L* < 2.5
Haemoglobin 162 g/L* 130 – 160
Creatinine 230 µmol/L* 60 – 120

a) Characterise the acid-base and blood gas abnormalities.

b) What is the likely diagnosis?

c) List the important principles of management specific to this condition.

College Answer

a) Characterise the acid-base and blood gas abnormalities.

Combined high anion gap and normal anion gap metabolic acidosis with inadequate respiratory compensation (respiratory acidosis)
A-aDO2 = 245

b) What is the likely diagnosis?
Paraquat ingestion

c) List the important principles of management specific to this condition.

Risk assessment based on estimate of quantity of Paraquat ingested
Gastrointestinal decontamination with diatomaceous earths, activated charcoal or sodium resonium

Monitoring for organ dysfunction (respiratory, CVS, renal, GIT, adrenal, hepatic, CNS)

Avoid high FiO2

Discussion

a)

This data set is identical to that of Question 14.1 from the first paper of 2008.

  • Firstly, what we have here is a hypoxia with a widened A-a gradient.
  • The PAO2 should be (0.5 x 713) - (35 x 1.25), or 311mmHg - so the gradient is a whopping 246.
  • Next, we have a metabolic acidosis (the BE is -9)
  • This disorder is inadequately compensated by ventilation. No matter which equation you use, the CO2 should be lower. If you apply the "7.xx" rule, the CO2 shold be the last two digits of the pH - 29. If you apply Winter's Formula, the CO2 should be around 32. Thus, a mild respiratory acidosis also exists.
  • The anion gap is only slightly raised, 17.3 (140+4.3 - 111 - 16)
  • The delta ratio is therefore 0.66 (5.3 / 8) -if we take the normal anion gap to be 12.
  • The metabolic acidosis is therefore a mixed disorder.
  • The serum osmolality and urea are not provided, so we cannot calculate an osmolar gap.

b)

The findings suggest paraquat toxicity:

Mild overdose:

  • Nausea and vomiting
  • Diarrhoea
  • Intestinal hemorrhage
  • Haemoptysis
  • Oliguria
  • Minimal renal dysfunction

Moderate overdose:

  • Renal failure (ATN within 12-24hours)
  • Pulmonary oedema
  • Hepatotoxicity
  • Pulmonary haemorrhage
  • Shock
  • Pulmonary fibrosis

Massive overdose:

  • multi-organ system failure
  • rapidly fatal

The toxicity (at least in moderate doses) emerges in several discrete phases:

  • Phase I: corrosion; mucosal linings ulcerate and swell; there may be haematamesis.This is the first two days.
  • Phase II: organ failure; between the second and fifth days following ingestion, renal failure and hepatocellular necrosis develop. Most patients with severe overdose will die during this phase.
  • Phase III: pulmonary fibrosis; death after many days/weeks of hypoxia.

c)

Management of paraquat overdose follows the following pattern:

Decontamination

  • Fuller's Earth: calcium montmorillonite, or bentonite - a absorbent aluminium phyllosilicate, formed from the weathering of volcanic ash.
  • Activated charcoal may have equal efficacy, and is more widely available
  • Cation exchange resins (eg. resonium) may be of use
  • The “window of opportunity”  is very narrow, only a few hours at most. Absorption from the gut is very rapid.
  • Remove contaminated clothes
  • Wash skin with soap and water to prevent transdermal absorption

Enhancement of elimination

  • Charcoal haemoperfusion works very well, but contributes little to the overall prognosis because the drug is rapidly cleared from the plasma anyway, and the pulmonary reserve is trapped there (it is not available for removal).
  • Dialysis is probably going to be useless, as paraquat is rapidly eliminated and by the time you get the circuit set up most of it will have gone already. The alveolar and renal damage will have been done by then, so you have nothing to gain (other than a more rapid control of the acid-base disturbance).

Specific antidotes

  • None exist. Among previously trialled antioxidants, we can find Vitamin E, Vitamin C, desferrioxamine, N-acetylcysteine, methylene blue, etc. Thus far, nothing satisfying has been found.

Supportive management

  • Intubation to protect the rapidly swelling airway after corrosive ingestion
  • Avoidance of hyperoxia:  it has been demonstrated to exacerbate the oxidative toxicity of paraquat.
  • Circulatory support (there will be shock from myocardial necrosis and third space losses
  • Analgesia and sedation which is almost palliative in its intent - many of these people will die in spite of everything you do.
  • Specifically, propofol seems to have some sort of unique scavenging effect.

References

Gawarammana, Indika B., and Nicholas A. Buckley. "Medical management of paraquat ingestion." British journal of clinical pharmacology 72.5 (2011): 745-757.

Clark, D. G. "Inhibition of the absorption of paraquat from the gastrointestinal tract by adsorbents." British journal of industrial medicine 28.2 (1971): 186-188.

Kehrer, James P., Wanda M. Haschek, and Hanspeter Witschi. "The influence of hyperoxia on the acute toxicity of paraquat and diquat." Drug and chemical toxicology 2.4 (1979): 397-408.

Dinis-Oliveira, R. J., et al. "Paraquat poisonings: mechanisms of lung toxicity, clinical features, and treatment." Critical reviews in toxicology 38.1 (2008): 13-71.

Sirker, A. A., et al. "Acid− base physiology: the ‘traditional’and the ‘modern’approaches." Anaesthesia 57.4 (2002): 348-356

Question 13.1 - 2015, Paper 1

The following data refer to a 28-year-old male who is day 5 in ICU following a severe traumatic
brain  injury.  He  has  no  other  injuries  and  has  been  heavily  sedated  with  infusions  of  fentanyl, midazolam and propofol since admission. Over the last four hours he has become increasingly bradycardic  and  hypotensive,  and  has  not  responded  to  fluid  loading  or  repeated  doses  of atropine.

Venous Biochemistry
Parameter Patient Value Normal Adult Range
Sodium 138 mmol/L 135 – 145
Potassium 5.1 mmol/L* 3.5 – 4.5
Chloride 100 mmol/L 95 – 105
Bicarbonate 11 mmol/L* 22 – 26
Urea 29 mmol/L* 2.9 – 8.2
Creatinine 310 μmol/L* 70 – 120
Calcium (corrected) 1.71 mmol/L* 2.10 – 2.55
Phosphate 2.31 mmol/L* 0.65 – 1.45
Creatine Kinase 25,000 U/L* 0 – 270
Lactate 5.1 mmol/L* < 2.0

Give the most likely diagnosis and the rationale for your answer.
(20% marks)

College Answer

Propofol Infusion Syndrome.
Rationale:  Biochemistry  consistent  with  rhabdomyolysis.  No  other  injuries  to  account  for
it. Refractory bradycardia and  hypotensive suggestive. History of high dose propofol
administration.
(Partial credit given for rhabdomyolysis, raised ICP and coning)

Discussion

Propofol infusion syndrome is not the first thing I would think of when confronted with a trauma patient who has suddenly become hypotensive and bradycardic. And the raised CK does not help (as if rhabdomyolysis is unknown in trauma patients). However, the college threw in the "refractory to atropine" thing, which arouses concern.

Propofol infusion syndrome is discussed elsewhere.

It is well covered in an article by Prof Kam.

Clinical features of propofol infusion syndrome

  •     Acute bradycardia leading to asystole.
    • A prelude to the bradycardia is a sudden onset RBBB with ST elevation in V1-V3; Kam’s article has the picture of this ECG. 
  •     Arrhythmias    
  •     Heart failure, cardiogenic shock
  •     Metabolic acidosis (HAGMA) with raised lactate (and also due to fatty acids)
  •     Rhabdomyolysis
  •     Hyperlipidaemia
  •     Fatty liver and hepatomegaly
  •     Coagulpathy
  •     Raised plasma malonylcarnitine and C5-acylcarnitine

References

Kam, P. C. A., and D. Cardone. "Propofol infusion syndrome." Anaesthesia62.7 (2007): 690-701.

Marinella, Mark A. "Lactic acidosis associated with propofol." CHEST Journal109.1 (1996): 292-292.

Vasile, Beatrice, et al. "The pathophysiology of propofol infusion syndrome: a simple name for a complex syndrome." Intensive care medicine 29.9 (2003): 1417-1425.

Schenkman KA, Yan S. Propofol impairment of mitochondrial respiration in isolated perfused guinea pig hearts determined by reflectance spectroscopy. Critical Care Medicine 2000; 28: 172–7.

Fodale, Vincenzo, and Enza La Monaca. "Propofol Infusion Syndrome." Drug Safety 31.4 (2008): 293-303.

Da-Silva, Shonola S., et al. "Partial-exchange blood transfusion: an effective method for preventing mortality in a child with propofol infusion syndrome." Pediatrics 125.6 (2010): e1493-e1499.

Uezono, Shoichi, et al. "Acquired carnitine deficiency: a clinical model for propofol infusion syndrome?." The Journal of the American Society of Anesthesiologists 103.4 (2005): 909-909.

Mirrakhimov, Aibek E., et al. "Propofol Infusion Syndrome in Adults: A Clinical Update." Critical care research and practice 2015 (2015).

 

Question 21.3 - 2015, Paper 1

A 46-year-old male from a foreign fishing vessel presents unconscious to the Emergency Department. He complained of visual disturbance prior to his deterioration.

The following blood results are obtained:

Parameter Patient Value Normal Adult Range
Sodium 144 mmol/L 135 – 145
Potassium 4.0 mmol/L 3.5 – 5.0
Chloride 102 mmol/L 95 – 110
Bicarbonate 8.2 mmol/L* 22.0 – 30.0
Urea 6.4 mmol/L 3.0 – 7.0
Creatinine 127 μmol/L* 44 – 97
Glucose 5.0 mmol/L 3.5 – 7.8
Calcium (ionised) 1.10 mmol/L 1.03 – 1.23
Lactate 4.1 mmol/L* 0.6 – 2.4
Osmolality 324 mOsm/kg* 275 – 295

a) What is the most likely diagnosis? (10% marks)

b) What is the pathophysiology of the visual disturbance? (20% marks)

c) List three specific treatments you would institute. (15% marks)

College Answer

a)

Methanol toxicity

b)

Methanol - > formaldehyde - > formate which is neurotoxic (especially retina and basal ganglia)

c)

Sodium bicarbonate

ADH inhibition with Ethanol (or fomepizole if available)

Dialysis

Cofactor therapy with either folic or folinic acid

Discussion

So as to be fair to the other no-less-toxic alcohols, here is a table of the common alcohol toxidromes

Disorder Toxin Clinical and Laboratory Abnormalities
Alcoholic ketoacidosis
  • β-hydroxybutyric acid
  • Acetoacetic acid
  • Metabolic acidosis
Methanol intoxication
  • Formic acid
  • Lactic acid
  • Ketones
  • Metabolic acidosis
  • hyperosmolality
  • retinal damage with blindness
  • Basal ganglia (putamen) damage
Ethylene glycol intoxication
  • Glycolic acid
  • Calcium oxalate
  • Cardiovascular collapse
  • Myocardial damage
  • Cerebral damage
  • Renal failure
  • Metabolic acidosis
  • Hypocalcemia
Diethylene glycol intoxication
  • 2-Hydroxyethoxyacetic acid
  • Neurological damage
  • Rrenal failure
  • Metabolic acidosis
Propylene glycol intoxication
  • Lactic acid
  • Metabolic acidosis
Isopropyl alcohol intoxication
  • Isopropanol
  • Coma
  • hypotension
  • No acidosis! Only acetone is the metabolic product

Management of toxic alcohol poisoning:

Decontamination

  • Activated charcoal is useless. Absorption is too rapid.

Enhanced elimination

  • Haemodialysis: toxic alcohols and their metabolites are rapidly cleared in this manner
  • Thiamine enhances metabolism of ethylene glycol to alpha-hydroxy-beta-ketoadipate
  • Pyridoxine enhances metabolism of ethylene glycol to glycine (and ultimately hippuric acid).
  • Folate and leucovorin enhance the clearance of formate
  • Alkalinization of urine with a bicarbonate infusion promotes dissociation of formic acid (it is less toxic in its ionised state) and improves its clearance by ion trapping in the urine

Specific antidotes

  • Alcohol -  the precise use of this substance in overdose is discussed in the chapter on ethylene glycol and its toxic acid metabolytes. 
  • In brief, one should sustain a blood ethanol concentration of 20 to 30 mmol/L (100 to 150 mg/dL) - this equates to a blood alcohol level of 0.1-0.15%.
  • Fomepizole as it is known, is basically a competitive antagonist to alcohol dehydrogenase. It does what ethanol would do, except it does so with great expense, and without ethanol intoxication. The advantage of using it is its lack of CNS effects - if the patient is confused already you do not want to add alcohol into the mix.

Supportive management

  • Boring supportive care is all that is required.
  • Airway control and mechanical ventilation:  the patient may be uncooperative and with a foul manner.
  • Circulatory support  in case of significant haemodynamic collapse
  • Sedation and analgesia with short acting substances

References

Kraut, Jeffrey A., and Ira Kurtz. "Toxic alcohol ingestions: clinical features, diagnosis, and management." Clinical Journal of the American Society of Nephrology 3.1 (2008): 208-225.

Henderson, William R., and Jeffrey Brubacher. "Methanol and ethylene glycol poisoning: a case study and review of current literature." Cjem 4.1 (2002): 34-40.

Question 26 - 2015, Paper 1

a)  Outline the clinical features and laboratory abnormalities likely to be found in a patient with envenomation due to an Australian snake-bite. (50% marks)


b)  Outline the management of a patient with confirmed snake envenomation. (50% marks)

College Answer

a)

Clinical features

Local pain, swelling and bruising. This may be absent

Sudden collapse – associated with hypotension and loss of consciousness, rarely cardiac arrest and seizure (5%)

Non –specific systemic symptoms – nausea, vomiting, diarrhoea, headache, sweating.

Neurotoxicity – descending flaccid paralysis – starting with ptosis, diplopia, blurred vision, and then progressing to bulbar weakness, respiratory and limb muscle paralysis.

Myotoxicity – local and generalised myalgia and muscle tenderness. Haemorrhage – rare – intracranial, gastrointestinal or from cannula sites

Laboratory abnormalities

Venom induced consumptive coagulopathy – characteristic of Australian snake bite – INR >3, APPT >100, fibrinogen < 1, raised D-dimers – can be 100 times assay cut off, Thrombocytopenia <100

CK – 1000 to over 100,000 u/L associated with myotoxicity

Acute renal failure – raised potassium, urea and creatinine.

Fragmented red cells in blood film – microangiopathic haemolytic anaemia.

b)

Management

First aid – Pressure bandage with immobilisation of the limb and the patient, pressure similar to that for a sprained ankle.

Monitor the patient in critical care area with resuscitation facilities – ED, HDU, ICU – neurological state, HR, BP, respiration, bleeding

Resuscitation as appropriate with two large bore cannulas and collect blood for laboratory tests – Coags (INR, APTT, Fibrinogen, D-Dimers), platelets, Urea, creatinine, electrolytes, CK.

Identify the likely snake type; the site of the bite can be swabbed and a venom detection kit (VDK) used or urine but not blood, or consultation with an herpetologist. Administer anti-snake venom (ASV) only if clinical symptoms or signs or lab abnormalities such prolonged INR. Current guidelines are for one vial ASV only and then correct subsequent coagulopathy with FFP

Release pressure bandage only after administration of ASV.

Type of ASV (monovalent or polyvalent) depends on clinical presentation, geography and VDK.

Monitor closely for anaphylactic reaction. Treat with adrenaline. Premedication with adrenaline, steroids or antihistamines not recommended.

Repeat lab investigations at 6, 12 and 24 hours to monitor response such as improvement in coagulopathy (INR).

Supportive treatment such ventilation for muscle paralysis and respiratory failure, dialysis for acute renal failure, inotropes for cardiovascular collapse and FFP for severe coagulopathy and bleeding complications

Discussion

Specific clinical features

  • Local pain, swelling and bruising (eg. brown snake bites)
  • Maybe myonecrosis (from black and tiger snakes)
  • Fang marks
  • Draining lymph nodes may be enlarged and painful
  • Systemic effects
  • Nausea
  • Vomiting
  • Abdominal pain
  • Diaphoresis
  • Diarrhoea
  • Headache
  • Renal impairment

Laboratory findings​ and investigations for a snake bite victim:

  • CK (rhabdmyolysis)
  • Coags (DIC, or "venom-induced consumption coagulpathy)
  • FBC (DIC, looking for thrombocytopenia and red cell fragmentation)
  • Fibrinogen (DIC)
  • EUC (renal failure)
  • LFTs (hepatic injury)
  • Snake Venom Detection Kit

Indications for polyvalent antidote:

  • Unsure which snake species was involved
  • SVDK not available
  • monovalent antivenom not available

Evidence for premedication for antivenom administration:

  • This is no longer recommended in Australia
  • polyvalent antidote tends to have a higher rate of anaphylaxis

How do you know your monovalent antivenom is working?

  • The short answer is, you dont.
  • It takes tme for some of the irreversible features to resolve (eg. it takes time to synthesis the coagulation factors which have been depleted)
  • Giving more antivenom will not improve the situation.

References

Isbister, Geoffrey K., et al. "Snakebite in Australia: A practical approach to diagnosis and treatment." Medical journal of Australia 199.11 (2013): 763-768.

Question 17 - 2015, Paper 2

A 45-year-old male is admitted to the Emergency Department after ingesting an unknown quantity of“headache tablets”. His initial complaints are nausea, vomiting, shortness of breath and tinnitus. Fluid resuscitation has been commenced. You are asked to assess him as he is getting more dyspnoeic.

His serum biochemistry and arterial blood gas profile are as follows:

Parameter

Patient Value

Normal Adult Range

Sodium

138 mmol/L

135 – 145

Potassium

3.2 mmol/L*

3.4 – 5.0

Chloride

108 mmol/L

100 – 110

Bicarbonate

10 mmol/L*

22 – 27

FiO2

0.3

pH

7.32*

7.35 – 7.45

PO2

125 mmHg (16.4 kPa)

PCO2

20 mmHg (2.6 kPa)*

35 – 45 (4.6 – 6.0)

Base Excess

-10 mmol/L*

-2 – +2

Salicylate level

105 mg/dL*

3 – 10

Paracetamol level

< 20 mg/L (< 130 µmol/L)

< 20 (< 130)

a)  Describe the acid-base status.            (20% marks)
b)  What are four severe complications of this toxidrome?        (20% marks)
c)  What coagulopathy may be present in this toxidrome and what is the treatment?    (10% marks)
d)   What are the treatment options for severe toxicity, and what is their rationale?                       (50% marks)

College Answer

a)
Acid-base status:

Increased anion gap metabolic acidosis Concomitant normal anion gap metabolic acidosis Respiratory alkalosis

Decreased delta ratio

b) Hypoglycaemia

Pulmonary oedema Cerebral oedema Arrhythmias Hyperpyrexia

c) Hypoprothrombinaemia Vitamin K

d)

Forced alkaline diuresis. Renal excretion of salicylates becomes important when the metabolic pathways become saturated. There is a 10-20 fold increase in elimination when the urine pH increased from 5 to 8.

Haemodialysis. Most of the drug is protein-bound, and is concentration dependant. The volume of distribution is small, and binding site saturation leads to large levels of free drug, which is easily dialyzable.

Multiple-dose charcoal. Many aspirin forms are slow release and after ingestion they clump together in the GI tract, forming a large slow release preparation. It is also poorly soluble in the stomach leading to delayed absorption.

Additional Examiners’ Comments:

Most candidates understood the acid-base abnormalities but not all were able to provide cogent answers relating to the complications and management. Few were able to describe all the treatment options for severe toxicity with the rationale for these strategies.

Discussion

This question is identical to Question 10 from the second paper of 2012.

b) Complicatons of salicylate overdose:

Serum level 30-50mg/dL: Serum level 50-75mg/dL: Serum level >75mg/dL:
  • Tachypnoea
  • Respiratory alkalosis
  • Nausea
  • Vomiting
  • Tinnitus
  • Dizziness
  • Tachypnoea
  • Respiratory alkalosis
  • Fever
  • Sweating
  • Dehydration
  • Agitation
  • Coma
  • Hallucinations
  • Seizures
  • Cardiogenic shock
  • Coagulopathy, with raised INR.
  • Oliguria
  • Renal failure.
  • Lactic acidosis and ketoacidosis

b) Coagulopathy in salicylate overdose? Its not just platelet inhibition.  According to UpToDate, this is because of hepatotoxicity and interference with the synthesis of vitamin K dependent factors.  Specifically, it is well known that salicylate toxicity can cause a decrease in prothrombin. Vitamin K (if not prothrombinex) is the answer.

c)Management of sever salicylate overdose consists of the following measures:

Severe toxicity from salicylates has several treatment options:

Decontamination

  • Multiple dose activated charcoal is recommended by the UpToDate toxicology authors. Aspirin is well adsorbed by charcoal. Three 25g doses separated by two hours is the recommebded regimen.
  • Whole bowel irrigation is relevant in the context of sustained release preparations, and has been useful in animal models.

Direct  and indirect antidotes

  • There is nothing specific. Urinary alkalinisation is generally held to be the nearest thing to a direct antidote.

Enhancement of clearance

  • Alkalinise the urine. This is vital. An alkaline blood environment also prevents the movement of salicylate into the CSF.  Raising the urine pH from 5 to 8 can increase total salicylate excretion by twenty times.
  • Haemodialysis may be required in severe cases, particularly where you cannot give any more bicarbonate (i.e. the patient is already fluid overloaded) or where the overdose is supermassive (levels in excess of 100mg/dL). Even though salicylate is highly protein bound this technique can usually move eough molecules to make a difference. One must also keep in mind the nonlinear kinetics of elimination - the higher the dose, the longer the half-life, and therefore the more prominent the effects of extracorporeal clearance.

Supportive ICU therapies

  • Intubation may be indicated, but must be carried out carefully (see next point)
  • Mechanical (hyper)ventilation  will be required: if the patient ends up being intubated, their minute volume must be maintained at least as high as it was prior to intubation. Respiratory alkalosis keeps the salicylate ions trapped in the blood; if a post-intubation acidosis is allowed to develop the sudden influx of salicylate into the CNS may cause seizures, cerebral oedema and death.
  • Vasopressors and inotropes  may be useful in some cases, but in the majority of cases the patient will be hypotensive because of volume depletion.
  • Supplemental glucose: these people are neuroglycopenic at normal BSL, and so the BSL should be kept at the higher range of normal.
  • Correction of hypokalemia is vital, because hypokalemia promotes K+ reabsorption at the distal tubule (where K+ is exchanged for H+). Ergo, hypokalemia interferes with the attempt to alkalinise urine.

References

O'Malley, Gerald F. "Emergency department management of the salicylate-poisoned patient." Emergency medicine clinics of North America 25.2 (2007): 333-346.

Pinedo, H. M., L. B. van de Putte, and E. A. Loeliger. "Salicylate-induced consumption coagulopathy." Annals of the rheumatic diseases 32.1 (1973): 66.

Shapiro, Shepard, Milton H. Redish, and Harold A. Campbell. "Studies on Prothrombin: IV. The Prothrombinopenic Effect of Salicylate in Man."Experimental Biology and Medicine 53.2 (1943): 251-254.

Pearlman, Brian L., and Rashi Gambhir. "Salicylate Intoxication." Postgraduate medicine 121.4 (2009).

Question 28 - 2016, Paper 1

Outline the differences in the assessment and management of poisoning from substance ingestion in the following clinical scenarios, compared with a healthy young adult:

a) 2-year-old child. (30% marks)

b) 30-week gestation pregnant female. (35% marks)

c) 75-year-old adult with chronic kidney disease. (35% marks)

College Answer

2-year-old child                                                                                                     

  • Ingested agent likely to be non-pharmaceutical
  • Vast majority of ingestions are benign
  • Other children may be affected (siblings, playmates)
  • Doses ingested likely to be small (2-3 tablets or small handful) and toxic effects mg/kg the same as adults but some agents can be potentially lethal for a toddler if even 1-2 tablets taken (e.g. amphetamines, Ca channel blockers, sulphonylureas) or a mouthful (e.g. organophosphate insecticides, eucalyptus oil, one mothball)
  • Unlikely to obtain accurate dosing history – risk assessment and management based on “worst-case scenario”
  • Need admission to health care facility with resources for paediatric resuscitation
  • Regular check of blood sugar levels
  • Usual toxicology screening tests for adult patient not necessary
  • GI decontamination with activated charcoal is not routine because of increased risks with aspiration – reserved for severe or life-threatening poisoning where supportive care or antidote treatment alone is inadequate
  • If severe intoxication suggesting large, repeated or unusual exposure, consider NAI

30/40 pregnant female                                                                                       

  • Risks to mother and foetus
  • Pregnancy-induced physiological changes impact on drug pharmacokinetics
  • Delayed gastric absorption and GI transit time slows drug absorption and increases period of potential benefit for decontamination
  • Increased blood volume increases VD and decreases drug plasma levels
  • Dilution of plasma proteins increases free drug levels 
  • Hepatic enzyme systems altered by circulating hormones
  • Increased cardiac output increase renal blood flow and GFR
  • Hypovolaemia and respiratory compromise may go unrecognised until at a late stage
  • A few agents pose increased risk to foetus and treatment threshold is lowered (e.g.
  • salicylates, CO, lead, MetHb-inducing agents)
  • Excellence in supportive care for the mother ensures best outcome for foetus
  • Obstetric and neonatal as well as toxicology input needed including decision for emergency delivery of baby.

75-year-old with CKD                                                                                           

  • Limited physiological reserve, deteriorating cognition, multiple co-morbidities and polypharmacy lead to exaggerated and unpredictable response in poisoning
  • More severe clinical course for same dose of same agent taken by healthy young adult
  • Pharmacokinetic changes with ageing and CKD o Delayed GI absorption o Decreased protein binding and increased free drug levels o Reduced liver function with decreased drug metabolism o Reduced renal function and reduced elimination o Baseline CKD likely to be made worse o “Therapeutic” drug doses may be toxic
  • Pharmacodynamic differences from drug effects on impaired organs e.g. poor ability to respond to CVS, respiratory and CNS depressant agents
  • Greater incidence of complications e.g. delirium, pneumonia, thrombo-embolism
  • Longer ICU and hospital stay

Discussion

This is another one of the questions in this paper which had a 0% pass rate. Locally available resources include the following chapters:

The answer would probably work better as a table:

A Comparison of Toxicological Differences
Group Infant/toddler Pregnant woman Elderly
Absorption
  • Cutaneous  absorption is more rapid
  • Intramuscular depot absorption is more rapid
  • Increased absorption by inhalation
  • Delayed absorption
  • More complete absorption (slower gut transit)
  • Slowed gastric emptying rate
  • Slowed gut transit
  • Gastric pH is lower
  • Transcutaneoius absorption is slow
Distribution
  • VD is greater for water soluble drugs
  • VD is smaller for fat-soluble drugs
  • Decreased protein binding
  • The blood brain barrier is immature
  • Increased volume of distribution
  • Decreased protein binding
  • Foetal pH causes "ion trapping"
  • VD changes: fat increases, water decreases
  • Protein binding is decreased
  • Predictive equations become inaccurate
Clearance
  • Rates of drug metabolism are increased
  • Altered hepatic clearance (due to hormones)
  • Increased renal clearance
  • Breastfeeding must be considered
  • Slowed hepatic clearance
  • Slowed renal clearance
Pharmacodynamics
  • Respiratory depression occurs more readily
  • Hypoglycaemia occurs more readily
  • Cardiovascular collapse occurs more precipitously
  • Paradoxical reaction to benzodiazepines and antihistamines
  • Foetal exposure is determined by maternal blood levels.
  • Teratogenicity of drugs must be considered
  • Increased  toxic effects
  • Decreased physiologic reserve
Pattern of poisoning
  • Accidental, or "exploratory"
  • Small in scale
  • Aspiration is more serious than the actual poisoning.
  • There are a few drugs which pose a greater threat to the foetus than the mother:  
    • Carbon monoxide
    • Methaemoglobin-inducing agents
    • Lead
    • Salicylates
    • Valproate
  • Accidental double dosing (due to poor memory)
  • Toxicity of a usually "safe dose"
  • Drug interactions
  • Outdated and discontinued drugs
  • Over the counter drugs
  • Opportunistic ingestion of random substances
Differences in approach
  • Overdose is considered in any child with unexplained obtundation
  • Early airway protection
  • NG charcoal only in the conscious and cooperative (or intubated) child
  • Naloxone 1-2mg IV - i.e. around 0.1mg/kg,
  • Dextrose 50%
  • Almost all of the antidotes to the various toxins are FDA pregnancy-risk category C, 
  • The exceptions are N-acetylcysteine, glucagon and naloxone (category B).
  • Emegency delivery needs to be considered to defend the foetus
  • Assisted clearance (eg. dialysis) may be required
  • ICU stay will be longer
  • Mortality is greater
  • Side-effects from decontamination are greater
       

References

Kearns, Gregory L., et al. "Developmental pharmacology—drug disposition, action, and therapy in infants and children." New England Journal of Medicine 349.12 (2003): 1157-1167.

Barry, J. Dave. "Diagnosis and management of the poisoned child." Pediatric annals 34.12 (2005): 937-946.

Reid, David HS. "Treatment of the poisoned child." Archives of disease in childhood 45.241 (1970): 428.

Henretig, Fred M. "Special considerations in the poisoned pediatric patient." Emergency medicine clinics of North America 12.2 (1994): 549-567.

Calello, Diane P., and Fred M. Henretig. "Pediatric toxicology: specialized approach to the poisoned child." Emergency medicine clinics of North America 32.1 (2014): 29-52.

Shieh-Czaja, Angela, Diane P. Calello, and Kevin C. Osterhoudt. "Sick sisters." Pediatric emergency care 21.6 (2005): 400-402.

Anderson, Gail D. "Pregnancy-induced changes in pharmacokinetics." Clinical pharmacokinetics 44.10 (2005): 989-1008.

Goldfranks Manual of Toxicologic Emergencies: 2007 Edition, Ch. 30: "Reproductive and Perinatal Principles"

Zelner, Irene, et al. "Acute poisoning during pregnancy: observations from the toxicology investigators consortium." Journal of medical toxicology 11.3 (2015): 301-308.

Klein-Schwartz, Wendy, and Gary M. Oderda. "Poisoning in the elderly." Drugs & aging 1.1 (1991): 67-89.

Ticehurst, Stephen, et al. "Elderly patients with deliberate self-poisoning treated in an Australian general hospital." International psychogeriatrics 14.1 (2002): 97-105.

Carlsten, A., Margda Waern, and P. Allebeck. "Suicides by drug poisoning among the elderly in Sweden 1969–1996." Social psychiatry and psychiatric epidemiology 34.11 (1999): 609-614.

Jansen, Paul AF, and Jacobus RBJ Brouwers. "Clinical pharmacology in old persons." Scientifica 2012 (2012).

Sotaniemi, Eero A., et al. "Age and cytochrome P450-linked drug metabolism in humans: an analysis of 226 subjects with equal histopathologic conditions." Clinical pharmacology and therapeutics 61.3 (1997): 331-339.

Mitchell, Rebecca J., et al. "Dementia and intentional and unintentional poisoning in older people: a 10 year review of hospitalization records in New South Wales, Australia." International Psychogeriatrics 27.11 (2015): 1757-1768.

Rogers, Jody J., and Kennon Heard. "Does age matter? Comparing case fatality rates for selected poisonings reported to US poison centers." Clinical toxicology 45.6 (2007): 705-708.

Doak, Martin W., et al. "Self-poisoning in older adults: patterns of drug ingestion and clinical outcomes." Age and ageing 38.4 (2009): 407-411.

Question 8 - 2016, Paper 2

With respect to salicylate toxicity:

a)    List four severe complications.    (20% marks)
b)    List the associated haematological abnormalities.    (10% marks)

c)    List the options for enhancing salicylate removal, and briefly explain the rationale for each option listed.    (50% marks)

d}  Give your interpretation of a declining serum salicylate level.    (20% marks)

College answer

a) List four severe complications:                                                                          

  • Pulmonary oedema
  • Cerebral oedema
  • Arrhythmias
  • Hyperpyrexia
  • Shock and cardiovascular collapse
  •  Acid-base disturbance (high anion gap metabolic acidosis and respiratory alkalosis)
  1. List the associated haematological abnormalities:                                              
    • Hypoprothrombinaemia
    • Thrombocytopaenia
  1. List the options for enhancing salicylate removal, and briefly outline the rational for each option listed:        
    • Haemodialysis. Most of the drug is protein-bound, and is concentration dependant. The volume of distribution is small, and binding site saturation leads to large levels of free drug, which is easily dialyzable
    • Multiple-dose charcoal. Many aspirin forms are slow release and after ingestion they clump together in the GI tract, forming a large slow release preparation. It is also poorly soluble in the stomach leading to delayed absorption.
    • Forced alkaline diuresis. Renal excretion of salicylates becomes important when the metabolic pathways become saturated. There is a 10 – 20 x increase in elimination when the urine pH increased from 5 – 8.  Current role is questionable as haemodialysis is more efficient at removal, with less metabolic disturbance.  Reasonable, as initial therapy whilst waiting for circuit prime and line insertion.
  2. Give your interpretation of a declining serum salicylate level:             
    It may indicate that the drug is moving into the tissues, and not necessarily being eliminated This means that clinical assessment is paramount​

Additional Examiners‟ Comments:

Most candidates were able to give general statements but were unable to give specifics – in particular about how the therapies worked. There was poor understanding of the pharmacokinetics of salicylates and the rationale for the use of haemodialysis. 

Discussion

This question closely resembles Question 10 from the second paper of 2012 and the identical Question 17 from the second paper of 2015, except instead of asking about "what coaguloapthy might be present",

a)

Salicylate toxicity has a whole list of complications:

  • pulmonary oedema
  • cerebral ordema
  • myocardial depression and shock
  • hypoglycaemia
  • seizures
  • haemorrhage from gastric ulceration
  • muscle rigidity leading to respiratory depression

c)

  • Raised PT: The classical coagulopathy which develops (asked about in the SAQs) is a prothrombin deficiency, leading to a prolonged PT and increased INR. According to UpToDate, this is because of hepatotoxicity and interference with the synthesis of vitamin K dependent factors. In addition to this, Question 8 from the second paper of 2016
  • Platelet dysfunction (due to COX enzyme inhibition)
  • Haemolytic anaemia (either by an autouimmune mechanism similar to that of methyldopa, or by oxidative damage as in G6PD - as per Sanford-Driscoll et al, 1986).

c)

Severe toxicity from salicylates has several treatment options:

Decontamination

  • Multiple dose activated charcoal is recommended by the UpToDate toxicology authors. Aspirin is well adsorbed by charcoal. Three 25g doses separated by two hours is the recommebded regimen.
  • Whole bowel irrigation is relevant in the context of sustained release preparations, and has been useful in animal models.

Direct  and indirect antidotes

  • There is nothing specific. Urinary alkalinisation is generally held to be the nearest thing to a direct antidote.

Enhancement of clearance

  • Alkalinise the urine. This is vital. An alkaline blood environment also prevents the movement of salicylate into the CSF.  Raising the urine pH from 5 to 8 can increase total salicylate excretion by twenty times.
  • Haemodialysis may be required in severe cases, particularly where you cannot give any more bicarbonate (i.e. the patient is already fluid overloaded) or where the overdose is supermassive (levels in excess of 100mg/dL). Even though salicylate is highly protein bound this technique can usually move eough molecules to make a difference. One must also keep in mind the nonlinear kinetics of elimination - the higher the dose, the longer the half-life, and therefore the more prominent the effects of extracorporeal clearance.
  • Multiple dose charcoal  as mentioned above

d) A declining salicylate level means nothing. Serial salicylate level measurement is meaningless, because:

  • It is highly protein bound, and the free fraction changes depending on the dose (as binding sites are saturated)- knowing the total level tells you nothing about the bioavailable fraction
  • It is poorly correlated with severity of intoxication (according to A.K.Done, 1960 - even the Done Nomogram has been largely abandoned because of this)
  • Acidosis causes the trapping of salicylate in the CNS, which would not be apparent from serum levels

Salicylate level may be declining because

  • It is clearing renally or by hepatic metabolism
  • Absorption from a bezoar is diminishing
  • The intracellular uptake of salycilate has resulted in decreased serum levels

References

O'Malley, Gerald F. "Emergency department management of the salicylate-poisoned patient." Emergency medicine clinics of North America 25.2 (2007): 333-346.

Pinedo, H. M., L. B. van de Putte, and E. A. Loeliger. "Salicylate-induced consumption coagulopathy." Annals of the rheumatic diseases 32.1 (1973): 66.

Shapiro, Shepard, Milton H. Redish, and Harold A. Campbell. "Studies on Prothrombin: IV. The Prothrombinopenic Effect of Salicylate in Man."Experimental Biology and Medicine 53.2 (1943): 251-254.

Pearlman, Brian L., and Rashi Gambhir. "Salicylate Intoxication." Postgraduate medicine 121.4 (2009).

Rothschild, Bruce M. "Hematologic perturbations associated with salicylate." Clinical Pharmacology & Therapeutics 26.2 (1979): 145-152.

Sanford-Driscoll, Marcia, and Leroy C. Knodel. "Induction of hemolytic anemia by nonsteroidal antiinflammatory drugs." Annals of Pharmacotherapy 20.12 (1986): 925-934.

Mandelli, M., and G. Tognoni. "Monitoring plasma concentrations of salicylate." Clinical pharmacokinetics 5.5 (1980): 424-440.

Done, Alan K. "SALICYLATE INTOXICATION Significance of Measurements of Salicylate in Blood in Cases of Acute Ingestion." Pediatrics 26.5 (1960): 800-807.

Kashani, John, and Richard D. Shih. "Salicylate Overdose." Encyclopedia of Intensive Care Medicine (2012): 2011-2014.

Question 2 - 2017, Paper 1

Please note: The following ECG has been recorded at 25 mm/sec and gain setting of 10 mm/mV.

A 73-year-old female collapsed in the Outpatient Radiology Department where she had been waiting to have a CT coronary angiogram. She had been given 160 mg verapamil to slow her heart rate for the scan.

Her usual medications included sotalol 80 mg twice a day.

On arrival of the Rapid Response Team she was drowsy, cold and peripherally shut down with systolic blood pressure 60 mmHg. Her arterial blood gas results at the scene are below, and her ECG is shown on page 3 (Figure 1).

Parameter

Patient Value

Adult Normal Range

Fi02

0.5

pH

7.05*

7.35 — 7 45

pCO2

40.4 mmHg (5.3 kPa)

35.0 - 45.0 (4.6 - 6.0)

pO2

221 mmHg (29.1 kPa)

SpO2

98%

Bicarbonate

10.5 mmol/L•

22.0-26.0

Base Excess

-17.9 mmol/l-•

-2.0- +2 0

Lactate

8.0 mmol/L*

0.5- 1 6

Sodium

132 rnmol/l_•

135 - 145

Potassium

5.4 mmol/L*

3.5 - 5.0

Chloride

105 mmol/L

95 - 105

Glucose

5.3 mmol/l-

3.5 - 6.0

CHB from LITFL

Give the likely underlying cause for the patient's collapse.  (10% marks)

Interpret the investigations.   (20% marks)

Outline specific therapies for the management of this patient, indicating the doses and mechanisms of action for any pharmacotherapy you have listed. (70% marks)

College answer

a) Cardio-toxicity from a combination of a beta-blocker and calcium channel blocker resulting in 
cardiogenic shock.
Candidates may include a differential diagnosis – MI and cardiogenic shock not unreasonable.


b) Metabolic (lactic) acidosis with inadequate respiratory compensation
 A-aDO2 approx 85 mmHg – raised for 73-year-old

Junctional bradycardia (but much slower than expected). Ventricular escape rhythm 
acceptable. Peri arrest.

c) Specific therapies
Statement on resuscitation (Rapid ABC; iv access; O2, start CPR if indicated, monitor, rapid 
echo).
Multiple agents often required with stepwise approach.
Atropine 1mg stat (can be repeated x 3; often ineffective; muscarinic receptor 
antagonist increases SA node discharge, conduction through the AV node and opposes 
action of Vagus nerve)
Adrenaline or Noradrenaline infusion starting at 10-20 g/min and titrate to a MAP > 65 
mmHg (+ve inotropy, chronotropy, vasoconstriction)
Calcium – Chloride or Gluconate can be given (more calcium in CaCl) – 10mls of 10% 
solution (can be repeated x3 +/- infusion; competitively increases calcium entry into the 
myocardium via non-blocked channels)
Glucagon 5mg stat (can be repeated x3; increases intracellular cAMP and has been 
shown to increase heart rate in BOTH beta-blocker and CCB toxicity).
• 100mls 8.4% NaHCO3 stat (she is already very acidotic)
Hyperinsulinaemia-Euglycaemia – short acting insulin 1 unit/kg with 50mls 50% 
Dextrose bolus, then 0.5 units insulin /kg/hr with 10% dextrose infusion and q1hrly BGLs 
and K+ (high dose insulin = +ve inotrope but mechanism not clearly understood)
Lipid Emulsion – 1ml/kg 20% lipid emulsion bolus (can be repeated x 3 then start 
infusion 0.5mls/kg/min; acts as a “lipid sink” surrounding lipophillic drugs rendering them 
ineffective & maybe fatty energy source for myocardium)

Other Therapies
• Trans-cutaneous pacing
• Trans-venous temporary pacing.
• VA-ECMO

Additional Examiners’ Comments:
Many candidates failed to interpret the ECG, or to discuss the mechanism of therapies. Basic 
knowledge gaps in many answers.

Discussion

The image used in this SAQ is not from the original college paper (those are a sacred and jealously guarded resource). Fortunately, the ECG I found at LITFL is virtually identical. 

In short, the underlying cause of the collapse is simultaneous calcium channel blocker and beta blocker overdose, a variant on the theme of toxic antiarrhythmic polypharmacy

To interpret the investigations:

  • Sightly raised A-a gradient (85 mmHg)
  • Acidaemia
  • The CO2 is contributing to the acidaemia
  • There is metabolic acidosis (SBE is -17.9)
  • No respiratory compensation (the expected CO2 is about 23.75 by Winter's rule, or around 22 if you use SBE to calculate it)
  • The anion gap is (132 - 105 - 10.5) = 16.5,  elevated even if we don't know the albumin
  • The delta ratio, assuming an albumin of 40,  = (16.5-12)/(24-10.5) = 0.33  i.e. the acidosis is mainly a normal anion gap disorder. This makes absolutely no sense with the lactate value of 8.0.  
  • The ECG demonstrates a complete heart block. The college ECG was not much different (they called their a "junctional bradycardia" as there were no P waves).
  • Vital signs and examination findings suggest cardiogenic shock with very poor cardiac output (drowsy, cool extremities, hypotensive etc).
  • The glucose is normal, which is weird - but the competing effects may have balanced each other out (beta-blockers cause hypoglycaemia and CCBs cause hyperglycaemia).

As to management: the college asked to outline specific therapies, i.e. not supportive ones. For the management of combined beta blocker and calcium channel blocker toxicity, this would really consist of the following strategies:

Management Dose and rationale
Decontamination with activated charcoal

1g/kg of charcoal; because it may slow the absorption of some beta-blockers and calcium channel blockers. 

Calcium infusion

0.2mmol/hr.
Calcium is a direct antagonist to calcium channel blockers.
Need to monitor phosphate levels, as they will drop.

High-dose insulin

0.5-2.0 unit/kg/hr, as well as whatever amount of dextrose is required to maintain normoglycaemia. This is becoming the standard of care  (Woodward et al, 2014) Why?

  • CCBs cause a decreased uptake of glucose into the myocardium
  • Activating insulin receptors bypasses the G-protein-coupled beta-receptors and activates the second messenger system (cAMP), promoting contractility 
  • Calcium channel blockers cause hypoinsulinaemia because insulin release is regulated by calcium entry into islet beta cells via L-type channels
Glucagon

May be effective (eg. Doyon et al, 1993) but is not recommended as a first-line agent.

Intralipid

Lipid emulsion should "decontaminate" the bloodstream by making these highly lipophilic drugs less bioavailable (true for most of them, with the exception of atenolol and sotalol). Verapamil toxicity is listed as one of the indications for the use of lipid emulsion in toxicology(Cave and Harvey, 2009).

In addition, generic supportive therapies can be listed, although they may not attract any marks:

  • Intubation for the patient with propanolol-induced coma or seizures
  • Mechanical ventilation to help manage the pulmonary oedema which can develop. It also reduces the whole-body oxygen demands (in presence of severely depressed cardiac function).
  • Vasopressors and inotropes  may be useful in some cases, but there will always the the possibility of disaster. For example, the administration of adrenaline for a β-1 selective blocker overdose. A large dose of adrenaline will be required to achieve a given chronotropic effect because of the presence of a competitive antagonist. This large dose of adrenaline will then go on to have a massive and unopposed α-1 agonist effect, and the patient's head will explode.
  • Milrinone particularly has been used in the past, and can be viewed as a legitimate option which ought to be tried before you go with IABP or ECMO
  • Transvenous pacing may be possible, but the ventricle may not capture; and when it captures it may not respond with vigorous contractions, but rather with a limp sort of twitching. 
  • IABP has been used in cases where nothing you do seems to help, and particularly in case where there has been a calcium channel blocker co-ingestion
  • ECMO may be the only answer to a complete failure of the circulation.
  • Magnesium supplementation to prevent torsade de pointes
  • Phosphate supplementation to enhance inotropy and counteract the effects of the calcium infusion
  • Sodium bicarbonate to reverse the QRS prolongation (and, as the college pointed out, "she is alerady very acidotic"

References

Nawrath, H., et al. "Class I Antiarrhythmic Drug Effects: What Is the Basis for Subgroups Ia, Ib and Ic." Cardiac Arrhythmias: The Management of Atrial Fibrillation (2013): 39.

Vaughan-Williams, E. M. "Classification of antiarrhythmic drugs." Cardiac arrhythmias 449 (1970).

Rosen, Michael R. "The sicilian gambit-a new approach to the classification of antiarrhythmic drugs based on their actions on arrhythmogenic mechanisms." Circulation 84.4 (1991): 1831-1851.

Kowey, Peter R. "Pharmacological effects of antiarrhythmic drugs: Review and update." Archives of internal medicine 158.4 (1998): 325-332.

Siddoway, L. A., P. J. Podrid, and P. R. Kowey. "Pharmacologic principles of antiarrythmic drugs." (1995): 355-368.

Yamreudeewong, Weeranuj, et al. "Potentially significant drug interactions of class III antiarrhythmic drugs." Drug safety 26.6 (2003): 421-438.

Strauss, William E., and Alfred F. Parisi. "Combined Use of Calcium-Channel and Beta-Adrenergic Blockers for the Treatment of Chronic Stable AnginaRationale, Efficacy, and Adverse Effects.Annals of internal medicine 109.7 (1988): 570-581.

Markota, Andrej, et al. "Treatment of near-fatal beta blocker and calcium channel blocker intoxication with hyperinsulinemic euglycemia, intravenous lipid emulsions and high doses of norepinephrine." Signa Vitae 10.1 (2015): 144-150.

Woodward, Christina, Ali Pourmand, and Maryann Mazer-Amirshahi. "High dose insulin therapy, an evidence based approach to beta blocker/calcium channel blocker toxicity." Daru 22.36 (2014): 2008-223.

Henry, Philip D. "Comparative pharmacology of calcium antagonists: nifedipine, verapamil and diltiazem." The American journal of cardiology 46.6 (1980): 1047-1058.

Doyon, Suzanne, and James R. Roberts. "The use of glucagon in a case of calcium channel blocker overdose.Annals of emergency medicine 22.7 (1993): 1229-1233.

Question 29 - 2017, Paper 1

A 65-year old male has been admitted to ICU needing invasive mechanical ventilation following two episodes of generalised tonic-clonic convulsions and vomiting after an episode of suspected self-harm.

He has a history of hypertension, chronic obstructive pulmonary disease (COPD) and depression. His medications include Ramipril,Fluoxetine,Metoprolol,Theophylllne and Fluticasone/Salmeterol inhaler. 

His vital parameters are as follows:

  • Temperature 36°C
  • Blood Pressure 85/46 mmHg
  • SpO97% (Fi02  0.35)
  • ECG: Atrial flutter with ventricular rate of 150 beats/min, normal QRS­ duration and Qtc interval.

His CT brain scan did not reveal any abnormality. Results of his biochemistry are as follows:

Parameter

Patient Value

Adult Normal Range

Sodium

136 mmol/l

135 - 145

Potassium

2.9 mmoVL*

3.5 .5.5

Chloride

105 mmol/L

92 - 107

Bicarbonate

10.9 mmoUL•

22.0 - 28.0

Urea

19.7 mmoll•

2.5 - 6.5

Creatlnine

220 umolJL•

45 - 90

Magnesium

0.55 mmovL·

0 65 - 1.00

Phosphate

0.55 mrnol/L*

0.75 - 1.50

Corrected Calcium

2.67 mrnol/l*

2.15 -2.55

Creatinine Kinase

150 U/L

55 - 170

Blood Glucose

15.2 mmol/l*

3.5 - 6.0

         Lactate

4.9 mmol/L*

< 2.0

Give the most likely diagnosis AND your reasoning.         (40% marks)

Briefly outline your therapeutic strategies for this patient.

 

College answer

a)

• Acute Theophylline Poisoning. The clinical findings of vomiting, seizures, hypotension, 
Atrial Flutter combined with metabolic abnormalities strongly suggests theophylline 
poisoning
• Above biochemical abnormalities may suggest β-agonist toxicity; but cardiac arrhythmias 
and seizures are rare features of β-agonist toxicity
• Biochemical findings and ECG abnormalities do not favour tricyclic anti-depressant or
SSRI overdose

b)
• Check serum theophylline
• Repeated doses of activate charcoal, as means of decontamination. Theophylline is also 
more rapidly cleared from the blood in patients receiving activated charcoal
• Extracorporeal removal such as charcoal hemoperfusion or hemodialysis, as 
theophylline has low volume of distribution without extensive protein binding. High 
efficiency hemodialysis as effective as charcoal hemoperfusion
• Control of seizures with benzodiazepines. Phenytoin should be avoided as it is not 
effective and may worsen mortality
• Correction of electrolyte abnormalities (hypokalemia, hypomagnesemia and 
hypophosphatemia)
• IV Esmolol or amiodarone for cardiac arrhythmia, after correction of electrolyte 
abnormalities
• Hypotension should be treated with IV fluids and/or noradrenaline. IV propranolol or 
esmolol may reverse hypotension as it is caused by β2-adrenergic effects
• Hypercalcemia usually responds to fluid resuscitation
• Hyperglycemia responds to fluids and/or insulin administration

Additional Examiner Comments: 
Several candidates failed to recognise theophylline poisoning. Many candidates failed to read the stem and did not give a rationale for their diagnosis. Management of theophylline toxicity was discussed poorly.

Discussion

Let us interpret these data systematically.

  • Sounds like an overdose
  • Clinical features include:
    • Seizures
    • Nausea and vomiting
    • Hypotension/shock
    • Atrial tachyarrhythmia
  • Biochemistry demonstrates:
    • Hypokalemia
    • Hypomagnesemia
    • Hypophosphataemia
    • Hyperglycaemia
    • Hypercalcemia
    • Lactic acidosis
    • Renal failure

So, sounds like a theophylline overdose. As the collegely rightly pointed out, there is no way this old guy could have cosumed enough salmeterol to make him this sick.

In general, the features of theophylline overdose are as follows:

Symptoms Signs Biochemistry
  • Nausea
  • Vomiting
  • Elevated mood
  • Agitation, anxiety
  • Hallucinations
  • Tachypnoea
  • Tachycardia
  • Hypotension
  • Widened pulse pressure
  • Tremor
  • Seizures
  • Increased muscle tone
  • Fasciculations
  • Hypokalemia
  • Hypomagnesemia
  • Hypophosphataemia
  • Hyperglycaemia
  • Hypercalcemia
  • Lactic acidosis
  • Respiratory alkalosis
  • Rhabdomyolysis

As for the management:

Decontamination

  • Repeated doses of activated charcoal (MDAC)

Enhanced elimination

  • Charcoal haemoperfusion

Antidotes

  • Strangely, SVT does not respond to adenosine. Goldfranks' Manual (2007 edition, p. 557) recommends calcium channel blockers as a more effective antiarrhythmic therapy (a β-blocker would be just as good but the patient will inevitably be somebody with either asthma or COPD). 

Supportive management

A - the patient will likely need intubation at some stage

B - ventilate them with a slightly higher rate to maintain the compensation for metabolic acidosis

C - they will likely be hypotensive with a large overdose; noradrenaline will be required.
      They will also have arrhythmias. The college answer helpfully suggests esmolol or amiodarone.           Esmolol has been used successfully (Seneff et al, 1990) and may paradoxically improve blood               pressure by acting as a β2-antagonist, as well as slowing the rate and improving diastolic filling.

D - Sedation with benzodiazepines seems like a sensible move.
       Likely, the patient will need them anyway for seziure control.
       Other antiepileptics are apparently ineffective.

E - Correct all their electrolyte disturbances

F - Consider dialysis; high efficiency dialysis may even remove some theophylline

G - Regular antiemetics and/or NGT (given how much you are relying on multi-dose charcoal)

References

Barnes, Peter J. "Theophylline.American journal of respiratory and critical care medicine 188.8 (2013): 901-906.

Hendeles, Leslie, et al. "Food-induced “dose-dumping” from a once-a-day theophylline product as a cause of theophylline toxicity." Chest 87.6 (1985): 758-765.

Ehlers, Sally M., Darwin E. Zaske, and Ronald J. Sawchuk. "Massive theophylline overdose: Rapid elimination by charcoal hemoperfusion." Jama240.5 (1978): 474-475.

Hall, Kevin W., et al. "Metabolic abnormalities associated with intentional theophylline overdose." Annals of internal medicine 101.4 (1984): 457-462.

Seneff, Michael, et al. "Acute theophylline toxicity and the use of esmolol to reverse cardiovascular instability." Annals of emergency medicine 19.6 (1990): 671-673.

MILTON, L. McPHERSON, et al. "Theophylline-lnduced Hypercalcemia."Annals of internal medicine 105 (1986): 52-54.

Question 1 - 2017, Paper 2

You have received a phone call from a junior colleague at a remote location. A previously well 32-year-old male has presented with nausea and hypotension following a confirmed bite on his leg from a brown snake. A retrieval team will arrive in approximately three hours; until then your colleague is the only medical officer available.

a) Outline the telephone advice you would give them. Include guidance on what complications they might expect to arise and how to manage them. (80% marks)

b) Several days after arrival in your Intensive Care Unit (ICU) the patient develops oliguric renal failure. List the possible causes. (20% marks)

College answer

a)                                                                                                                                  

  • Ensure patient is in an appropriate monitored area
  • Give face mask oxygen, obtain iv access. Fluid resuscitation if hypotensive.
  • Apply pressure bandage over the bite site and aim to cover entire leg. Splint limb and keep immobile.
  • Patient has features of systemic envenomation and should therefore receive appropriate antivenom, one vial is adequate dose. No requirement for premedication with adrenaline or steroids.
  • Ideally take baseline blood tests, including coagulation studies U&E, FBE, CK, LFTs.
  • Given the circumstances it would be reasonable to either release the pressure bandage after antivenom administration or keep it in place until the patient has been retrieved (Note to examiners – some mention of what to do with the PB expected, although either option acceptable)
  • Discussion with National Poisons Information Centre

Complications include:

  • Anaphylaxis to antivenom – manage by stopping infusion, airway management as indicated and fluid resuscitation. May require adrenaline – use with caution due to concern of raised blood pressure and potential coagulopathy.
  • Coagulopathy: - likely very high INR, undetectable fibrinogen
  • If no active bleeding does not require specific management other than antivenom. If severe or life-threatening bleeding, reasonable to give FFP after antivenom.
  • May develop severe hypotension or cardiac arrest. Manage according to basic ALS principles
  • Neurotoxicity and cardiotoxicity rare and mild with brown snake envenomation

        b)                                                                                                                                       

  • Potential causes of renal failure.
  • Thrombotic microangiopathy secondary to consumptive coagulopathy
  • Rhabdomyolysis
  • ATN secondary to prolonged hypotension/arrest.
  • Secondary sepsis
  • Transfusion mismatch

Examiners Comments:

 Many candidates ignored the setting of a remote location completely, and gave a management plan that was applicable to a tertiary centre (e.g., TEG and ROTEM; "intubate" without reference to the skill of the junior doctor, etc.).

 Some candidates appeared unaware of even the most basic aspects of snake bite management e.g., pressure immobilization, VDK, monovalent versus polyvalent etc.

 Many candidates used an ABCDE template which prioritized airway and breathing above the first-aid of snake bite; also, it resulted in not covering the coagulopathy aspects well enough.

The answer for the renal failure again seemed templated (pre-renal, renal, post-renal) and lacked context - there were very few references to the snake bite and antivenom as possible causes of renal failure

Discussion

The venom itself is a mixture of presynaptic and postsynaptic neurotoxins and procoagulants. There is nothing myotoxic or nephrotoxic in the venom. Acute kidney injury is seen anyway because of thrombotic microangiopathy, which is a side-effect of the procoagulant venom. 

Brown snake venom produces the following stereotypical effects:

  • Venom-induced consumpation coagulopathy (VICC): all of the clotting factors are depleted, fibrinogen drops to 0 and INR increases dramatically. Apparently this takes about 24 hours to resolve near-completely. Giving clotting factors may shorten this time- Brown et al (2009) observed that people were generally giving 4 units of FFP and 8 units of cryoprecipitate.
  • Haemorrhage from trivial injuries: for example, Allen et al (2012) found that 32% of the victims end up having haemorrhage from cannula sites.
  • Myotoxicity: this is usually a feature of envenoming by the king brown snake, Pseudechis australis  (Ponraj et al, 1996). Normal brown snake bites should not cause rhabdomyolysis or myoglobinuria; whereas the king brown snake venom can cause local myonecrosis at the site of the bite. How to tell whether your snake is royalty?  Apparently it is difficult even for snake afficionados. Apart from being a bit wider, the distinctions rest in subtle things like paired subcaudal scales on one and singles on the other. It would be unreasonable to expect the "junior colleague" from Question 1 to be able to confidently identify the reptilian enemy, and so it would be reasonable to instruct them that they may expect rhabdomyolysis.
  • Mild neurotoxicity: This is a possible consequence of the presynaptic and postsynaptic effects of the brown snake venom, but it is very rare. In the review by Allen et al (2012), only 1% of the patients (2 victims) had neurotoxicity: one developed ptosis, and the other had weird migratory cranial nerve signs including diplopia and bulbar weakness. Given that coagulopathy is a major problem here, any sudden onset neurological signs would probably need to be interpreted as an intracranial haemorrhage. You'd scan the head before putting things down to neurotoxicity.
  • Cardiovascular consequences: The VICC tends to create cardiovascular collapse with decreased cardiac output and severe hypotension (which in some human cases has concluded with cardiac arrest in the prehospital setting).  Tibballs et al (1992) were able to demonstrate this in a bunch of dogs they envenomed for science. The culprit appears to be the prothrombin-activating component of the venom, as all cardiovascular badness was prevented completely by premedicating the dogs with heparin. 
  • Thrombotic microangiopathy,  which appears to be unrelated to the VICC.  The microscopic clots which form everywhere in the process of VICC might be expected to have a cheesegrater-like effect on the endothlium of small vessels (like in TTP-HUS) but in fact the DIC has usually resolved by the time this micorangiopathy takes place. Isbister et al (2007) found that microangiopathic haemolytic anaemia tends to develop in about 13% of the victims, with the nadir of severe thrombocytopenia (platelet count less than <20 × 109/L) occurring around 4-5 days after the bite. The authors likened the effect to that of HUS-indicung E.coli, commenting that "it is conceivable that the venom (or a toxin in the venom) induces similar endothelial damage and initiates the thrombotic microangiopathy".

Specific management steps should include:

  • Pressure bandage
  • Splint limb
  • Urgent antivenom
    • Polyvalent or monovalent, depending on whether the species has been confidently identified
  • FFP and cryoprecipitate to help correct coagulopathy more rapidly (if available)

Distant back-of-Bourke management should consist of:

  • Immobilisation
  • Airway support
  • Basic blood tests
  • Vascular access 
  • Organisation of retrieval
  • Liason with Poisons Centre

ICU-level management should consist of the following supportive steps:

  • A - assess the need for airway protection; intubate the patient if needed or if appropriate skills are available
  • B - there may be hypoxia; perform a CXR to assess pulmonary haemorrhage or pulmonary oedema 
  • C - haemodynamic instability is likely and hydration probably has merit if myotoxicity is going to develop - fluid resuscitation should be vigorous.
  • D - analgesia is probably going to be required
  • E - electrolyte derangement may be present due to prehospital exposure (dehydration, this is 'Straya) and rhabdomyolysis
  • F - Renal replacement therapy may be indicated as acute kidney injury develops
  • H - Nonessential invasive procedures should be delayed until after the coagulopathy subsides
  • I - Antibiotics are not indicated, but don't forget the ADT booster (because it is possible that everybody else did forget)

Though the examiners complained bitterly about templated answers being used to mask the candidates' unfamiliarity with snake bites, one cannot help but note that in the absence of specific venom nephrotoxins the patient's renal failure could be due to any of the normal things which cause renal failure. And these things are typically categorised as pre-renal, post-renal and intra-renal. With the exception of VICC-induced microangiopathy, the college list of differentials is certainly no different to a normal list of causes for renal failure in critical illness, featuring such favourites as "sepsis" and "ATN secondary to prolonged hypotension/arrest". In response, here is a classically organised list of plausible-sounding reasons for renal failure in a patient with a brown snake bite:

Causes of Acute Renal Failure
Following a Brown Snake Bite

Pre-renal

Intra-renal

Post-renal

  • Hypovolemia:
    • Haemorrhage
    • Dehydration in the outback
  • Redistribution of fluid
    • Sepsis
    • Aseptic SIRS, eg. anaphylaxis due to antivenom
  • Decreased cardiac output
    • Cardiac failure due to VICC
  • Renal microvascular obstruction
    • Thrombotic microangiopathy
  • Acute Tubular Necrosis
    • Vascular insufficiency (pre-renal)
    • Drug-related
    • Myoglobin (rhabdomyolysis)
    • Haem (haemolysis)
    • Sepsis
  • Upper tract obstruction
    • Renal haemorrhage due to coagulopathy
  • Bladder outlet obstruction
    • Clots due to haematuria (traumatic IDC insertion, coagulopathy etc)

References

Isbister, Geoffrey K., et al. "Snakebite in Australia: a practical approach to diagnosis and treatment." Med J Aust 199 (2013): 763-768.

Bücherl, Wolfgang, Eleanor E. Buckley, and Venancio Deulofeu, eds. Venomous Animals and Their Venoms: Venomous Vertebrates. Vol. 1. Elsevier, 2013.

Russell, Findlay E., and Harold W. Puffer. "Pharmacology of snake venoms." Clinical toxicology 3.3 (1970): 433-444.

Daltry, Jennifer C., Wolfgang Wüster, and Roger S. Thorpe. "Diet and snake venom evolution." Nature 379.6565 (1996): 537-540.

Allen, George E., et al. "Clinical effects and antivenom dosing in brown snake (Pseudonaja spp.) envenoming—Australian snakebite project (ASP-14)." PLoS One 7.12 (2012): e53188.

Brown, Simon GA, et al. "Clotting factor replacement and recovery from snake venom-induced consumptive coagulopathy." Intensive care medicine 35.9 (2009): 1532-1538.

Isbister, Geoffrey K., et al. "Thrombotic microangiopathy from Australian brown snake (Pseudonaja) envenoming." Internal medicine journal 37.8 (2007): 523-528.

Tibballs, J., et al. "The cardiovascular and haematological effects of purified prothrombin activator from the common brown snake (Pseudonaja textilis) and their antagonism with heparin." Anaesthesia and intensive care 20.1 (1992): 28-32.

Ponraj, Durairaj, and Ponnambalam Gopalakrishnakone. "Establishment of an animal model for myoglobinuria by use of a myotoxin from Pseudechis australis (king brown snake) venom in mice." Laboratory animal science 46.4 (1996): 393-398.

White, Julian. "Factor replacement for Australian snakebite coagulopathy: a re-evaluation?." (2009): Intensive Care Med (2009) 35:1503–1504

Question 7 - 2017, Paper 2

In the setting of haemodynamic collapse secondary to drug overdose, give the pharmacological antidote/s for each of the agents listed below. For each antidote cited, give the rationale/mechanism of action.  

a)       Digoxin.

b)      Tricyclic  anti-depressants.

c)       Beta blockers.

d)       Lignocaine.

College answer

Detail in template more than required for full marks:                                                      

 

Digoxin

Digoxin

Fab

Fragments

(Digibind)

  • Digibind has a much higher affinity (high affinity (109– 1010 L/mol) for digoxin than the Na+/K+ ATPase digoxin receptor site
  • Binds to digoxin in the extracellular spaces preventing digoxin binding to the Na+/K+ ATPase
  • Creates a concentration gradient that extracts digoxin from the intracellular space
  • Bound digoxin is then renally eliminated with digibind
  • If potential for cardiac arrest due to digoxin – antidote of choice

TCA

Sodium bicarbonate

  • Alkalinising solution – leading to increased pH.  
  • Favours the neutral or non-ionised form of TCA making it less available to bind to sodium channels.  
  • Cardiac muscle more inotrope responsive

Sodium load 

  • Increased extracellular Na concentration increasing the electrochemical gradient across cardiac cell membranes, potentially attenuating the TCA-induced blockade of rapid sodium channels

Beta

Blockers

Glucagon

  • Activates adenylate cyclase in cardiac muscle cells at a site independent from B-adrenergic agents, causing increase in cAMP leading to increased intracellular calcium augmenting contractility.
  • Large doses required and tachyphylaxis occurs

High Dose insulin  +/- glucose therapy

Several theories of effect:

  • Insulin release from B-islet cells is impaired following overdose (especially Ca blocker)
  • Overdose appears to disrupt fatty acid metabolism and create relative insulin resistance in myocardium.
  • State of CHO dependence in stressed myocardium and insulin resistance can be overcome with high dose insulin therapy

Atropine

• Anti-cholinergic agent

Lignocaine

Lipid emulsion therapy

  • Has been used in poisonings involving other lipophilic medications 
  • Thought to act as a lipid “sink”: increasing plasma concentration of lipid – shift of lipophilic medications from tissue to plasma.
  • Also providing myocardium with an energy source. Case reports of effect in b blockers and ca channel blockers
  • Used as an adjunct to other therapies.

Discussion

This question begs for a tabulated answer. The college table is comprehensive and difficult to improve upon. One's only recourse would be either to make the answer more succinct, or (more likely) to add more unnecessary detail ("more than required for full marks").   

Drug Antidote Rationale/mechanism

Digoxin

Digoxin-specific Fab fragments
  • Digoxin-Fab is a monovalent immunoglobulin
  • Its molecular weight is 46,000 Da 
  • Its volume of distribution is about 0.4L/kg, i.e. must also distribute at least to some extent into the interstitial fluid.
  • The circulating Fab acts as a digoxin sink, increasing the gradient for free digoxin to enter the circulation; this increases the renal clearance of digoxin by 20-30% (Chan and Buckley, 2014).
  • It is removed by both renal clearance and hepatic metabolism, but it's mainly renal: the digoxin-antibody complexes are filtered through the glomeruli  (which is surprising, consider their size) and reabsorbed in the proximal tubules while the digoxin is excreted. In renal failure, its half life (19 hours) is increased to 130 hours.
  • It has a 100 – 1000 times higher affinity for digoxin than does Na+/K+ ATPase.
  •  Each vial of DigiFab (38 – 40 mg of Fab) binds approximately 500 mcg of digoxin
Tricyclics Sodium bicarbonate
  • Increased protein binding of TCAs in an alkaline bloodstream, thus decreasing the biologically active free fraction. .
  • Increased availability of sodium in sodium bicarbonate, as a substrate for the voltage-gated channels (though the administration of hypertonic saline seemed to have greater antiarrhytmic effect than sodium bicarbonate!)
  • Decreased binding of TCAs to the voltage gated sodium channel - apparently this binding is affected by subtle changes in pH, and this receptor family has greater affinity for TCAs at acidic pH. 
  • Correction of metabolic acidosis may play a brutally stupid non-toxicological role by improving the affitnity of catecholamine receptors for their ligands.
  • Volume expansion which probably leads to better haemodynamic performance.
  • Cellular membrane hypopolarisation results from bicarbonate-induced intracellualr shift of potassium.
β-blockers High dose insulin with euglycaemia
  • Insulin is a potent positive inotrope in high doses;  this is apparently because of its effects on various calcium-handling pathways, particulalry those mediated by PI3K (Engebretsen et al, 2011).
  • It assists myocardial uptake of carbohydrates, which is the preferred fuel substrate of the heart under stressed conditions (whereas normally free fatty acids are preferred).
  • It improves the response to catecholamines
  • Insulin produces vasodilation, which improves local microcirculation (due to enhancement of endothelial
    nitric oxide synthase activity) - apparently this can "achieve perfused capillary density similar to that of exercising muscle". 
  • The dose is approximately 0.5-1 unit/kg/hr, but can be titrated up to 10 unit/kg/hr
Glucagon
  • Glucagon activates adenylate cyclase, which leads to increased levels of cyclic AMP in the myocytes
  • This is the same mechanism of action as the activation og the G-protein-coupled β-receptor
  • The net effect is that the blocked receptor is bypassed. Weirdly, bypassing it in other ways (eg. by giving a phosphodiesterase inhibitor) does not seem to have a satisfactory effect, particularly in terms of chronotropy.
  • This drug is fairly unwieldy to use, as the dose is a continuous i.v. infusion at a rate of 2–5 mg/hr (maximum: 10 mg/hr);  one patient will require up to 50 mg of glucagon over 24 hours 
  • There is tachyphylaxis, reports of treatment failure (Shepherd, 2006) and it may not work for all the β-blockers (eg. it may not be effective for propanolol)
Atropine
  • Its an antimuscarinic drug, which should increase the sinus node rate
  • However, this will do little to help the cardiac contractility and the slowed AV node conduction
  • Peterson et al (1984) found it "inconsistent in reversing the bradycardia and hypotension"
Lignocaine Lipid emulsion
  • Ciechanowicz et al (2012) lists several mechanisms:
  • Lipid emulsion acts as a "lipid sink", binding circulating (highly lipohilic) molecules of lignocaine to reduce their bioavailability to the cardiac and CNS voltage-gated sodium channels (eg. free fraction of bupivacaine  is decreased by two thirds)
  • Decreased circulating free drug fraction increases the mobilisation out of tissues, and increases the availability of lignocaine to organs of clearance.
  • Triglycerides also act directly on cardiac calcium channels to increase myocardial calcium concentration
  • Free fatty acid availability may have some sort of metabolic benefit for the fat-hungry myocardium
  • The dose of 20% lipid emulsion is 1.5 mL/kg over 1 minute; followed by an infusion of
    15 mL/kg/h. 
  • It was first discovered by Weinberg (1998) who was trying to kill rats with bupivacaine (surely there must be an easier way...) - those pre-treated with ;ipid emulsion had their LD50 increased by 50%

References

Hauptman, Paul J., and Ralph A. Kelly. "Digitalis." Circulation 99.9 (1999): 1265-1270.

Hoffman, J. R., and C. R. McElroy. "Bicarbonate therapy for dysrhythmia and hypotension in tricyclic antidepressant overdose." Western Journal of Medicine134.1 (1981): 60.

Woodward, Christina, Ali Pourmand, and Maryann Mazer-Amirshahi. "High dose insulin therapy, an evidence based approach to beta blocker/calcium channel blocker toxicity." Daru 22.36 (2014): 2008-223.

Donald, M. J., and S. Derbyshire. "Lignocaine toxicity; a complication of local anaesthesia administered in the community." Emergency medicine journal 21.2 (2004): 249-250.

Chan, B. S. H., and N. A. Buckley. "Digoxin-specific antibody fragments in the treatment of digoxin toxicity." Clinical Toxicology 52.8 (2014): 824-836.

Shepherd, Greene. "Treatment of poisoning caused by β-adrenergic and calcium-channel blockers." American Journal of Health-System Pharmacy 63.19 (2006): 1828-1835.

Engebretsen, Kristin M., et al. "High-dose insulin therapy in beta-blocker and calcium channel-blocker poisoning." Clinical toxicology (2011).

Peterson, Charles D., J. Steven Leeder, and Steve Sterner. "Glucagon therapy for β-blocker overdose." Drug intelligence & clinical pharmacy 18.5 (1984): 394-398.

Ciechanowicz, Sarah, and Vinod Patil. "Lipid emulsion for local anesthetic systemic toxicity." Anesthesiology research and practice 2012 (2012).

Weinberg, Guy L., et al. "Pretreatment or resuscitation with a lipid infusion shifts the dose-response to bupivacaine-induced asystole in rats." The Journal of the American Society of Anesthesiologists 88.4 (1998): 1071-1075.

Question 11 - 2018, Paper 1

Discuss the pathophysiology, clinical features and the management of a patient who presents with acute crystal methamphetamine ("ICE") intoxication

College answer

Pathophysiology 

  •  Methamphetamine lacks direct adrenergic effects, but is instead an indirect neurotransmitter by displacing adrenaline, noradrenaline, dopamine, and serotonin into the cytosol, leading to a surge of adrenergic stimulation.  
  •  Serotonergic activation contributes to alterations in mood as well as deranged responses to hunger and thirst.  

Clinical features 

  • Systemic / vital signs 
    • Hypertension 
    • Tachycardia 
    • Tachypnea 
    • Hyperthermia 
  •  CNS 
    • Severely agitated delirium / psychosis 
    • Seizures 
    • Coma 
  • CVS 
    • Stress-induced cardiomyopathy 
    • Accelerated Atherosclerosis 
  •  Metabolic 
    • Metabolic acidosis 
    • Hyperkalemia/Hypernatraemia 
    • Other electrolyte disturbances 
  •  Oliguric renal failure 
  •  Skin – track marks, cellulitis, abscess 

Candidates should have demonstrated an understanding of the multisystem nature of the condition (e.g. listing of several affected systems) in order to score well for this section. 
 

Management 

  • Mainly supportive management 
    • Management of severe agitation with high risk of self-harm or harm to others – pharmacological and non-pharmacological management 
      • Sedation with benzodiazepines/consider dexmetomidine or clonidine 
      • Low threshold to intubate 
        • Avoid succinylcholine 
    •  Aggressive cooling for hyperthermia with combination of techniques – surface cooling, intravenous cooling, antipyretics 
      • Control of autonomic disturbance (tachycardia, hypertension) 
      • Autonomic disturbance (tachycardia, hypertension) – combined alpha + beta blocker
        (avoid pure beta blockade due to risk of malignant hypertension) 

 Examiners Comments: 
 A number of candidates only mentioned generic details in their answer instead of specific issues related to the condition. Knowledge of the pathophysiology was poor. 

 

Discussion

Pathophysiology:

  • Administration is rarely by the oral route - usually smoked snorted or injected
  • Large volume of distribution
  • Highly lipophilic drug, penetrates well into the CNS
  • Resistant to metabolism, long half-life (19-34hrs)
  • Pharmacologic effect is by several mechanism:
    • Blockade of monoamine reuptake transporters
    • Displacement of monoamines from presynaptic vesicles
    • Displacement of monoamines from neuronal cytosol by changing cytosolic pH

Clinical features:

  • Respiratory
    • Tachypnoea, increased minute volume
    • Irregular respiratory pattern
  • Circulatory
    • Tachycardia
    • Hypertension (with severe overdose, hypotension)
    • ECG changes suggestive of coronary ischaemia
    • Raised troponin
    • Flushing, brisk capillary refill
  • Neurological
    • Agitation, anxiety
    • Hallucinations
    • Psychosis
    • Seizures
    • Hyperthermia
    • Mydriasis
    • Piloerection
    • Hyper-reflexia
  • Fluid, electrolyte and endocrine-related
    • Diaphoresis
    • Increased insensate fluid loss though tachypnoea and diaphoresis
    • Hyperkalemia
    • Metabolic acidosis
  • Renal
    • Rhabdomyolysis-induced myoglobinuria
    • Concomitant acute pre-renal failure due to dehydration

Management: 

  • Decontamination
    • Activated charcoal is only indicated for orally ingested drug, within 1-2 hrs
    • Laparotomy is often required for "body stuffers"
  • Control of agitation
    • Benzodiazepines (oral or IV diazepam, or IM midazolam)
    • Propofol  for the intubated amphetamine overdose patient. 
    • Haloperidol appears safe in small doses (under 10mg) but in higher doses may lower the seizure threshold
    • Dexmedetomidine is a safe novel agent  Richards et al (2015) were able to dig up one case series and a few case reports to support its use. 
  • Control of hypertension
    • For hypertension, first control agitation.
    • Additional drugs could include alpha-antagonist drugs such as phentolamine, or vasodilators such as GTN or sodium nitroprusside.
    • β-blockers are controversial (there might be an "unopposed alpha effect" );  Richards et al (2015) recommend the use of nonselective β-blockers such as labetalol.
  • Seizure management
    • Benzodiazepines would be first-line.
    • Phenytoin should be avoided
  • Temperature management
    • Maintain normothermia
    • Active cooling may need to take place
    • Local guidelines recommend intubation and active cooling with paralysis if the temperature exceeds 39.5°C
    • Antipyretics such as paracetamol are not effective
  • Fluid and electrolyte correction
    • Investigate for hyponatremia (i.e. from polydipsia)
    • Investigate for consequences of rhabdomyolysis
    • Correct hyperkalemia
    • CRRT may be required because of AKI, rather than to remove the drug.

References

Li, Wenlong, and Naren Gunja. "Illicit drug overdose: Prevalence and acute management.Australian family physician 42.7 (2013): 481.

Vasan, Sarayu, and Garth J. Olango. "Toxicity, Amphetamine." (2017).

Richards, John, and Erik Laurin. "Toxicity, methamphetamine." (2017).

Darke, Shane, Sharlene Kaye, and Johan Duflou. "Rates, characteristics and circumstances of methamphetamine‐related death in Australia: a national 7‐year study.Addiction112.12 (2017): 2191-2201.

Albertson, Timothy E., Robert W. Derlet, and Brent E. Van Hoozen. "Methamphetamine and the expanding complications of amphetamines." Western Journal of Medicine 170.4 (1999): 214.

King, Andrew, Mirjana Dimovska, and Luke Bisoski. "Sympathomimetic Toxidromes and Other Pharmacological Causes of Acute Hypertension.Current hypertension reports20.1 (2018): 8.

Laitselart, Philippe, et al. "Severe Sympathomimetic Toxidrome in a French Soldier: How Caffeine Overdose Can Lead to Severe Consequences.Military Medicine (2017).

Richards, John R., et al. "Treatment of toxicity from amphetamines, related derivatives, and analogues: a systematic clinical review." Drug & Alcohol Dependence 150 (2015): 1-13.

Jenner, L., et al. "Management of patients with psychostimulant toxicity: guidelines for emergency departments." Canberra, Australian Government Department of Health and Ageing (2006).

Question 13 - 2018, Paper 1

Compare and contrast Serotonin Syndrome with Neuroleptic Malignant Syndrome 

College answer

Serotonin syndrome (SS)

Neuroleptic malignant syndrome (NMS)

Precipitants &

Risk factors

Serotonergic Agents such as TCAs, SSRIs, SNRIs, MAOIs, triptans, nefazodone, buspirone, mirtazapine, carbamazepine, tramadol, linezolid, MDMA (ecstasy), dextromethorphan, St. John's wort, lithium, methadone, cocaine, levodopa, reserpine, and amphetamines. *naming a few drugs/classes adequate

Usually concurrent use of multiple agents

Dopamine Antagonists such as antipsychotics and antiemetics. Also, abrupt withdrawal of dopamine agonists, for instance, those used in the management of Parkinson's disease, may produce signs and symptoms correlating with NMS. NMS does not necessarily correspond with high doses of antipsychotics, as it can occur with lower doses  

Concurrent use of serotonergic agents 

Use of illicit drugs, especially when used in patients concurrently taking a serotonin enhancing drug. 

Use of first- &/or second-generation antipsychotics. Use of higher doses of first- &/or second-generation

antipsychotics

Rapid escalation of dosing, switching among agents, higher potency agents, and long-acting depot formulations

Incidence 

Rare

0.02–2.4% in patients being treated with neuroleptics

Time of onset following inciting agent

 Usually < 24 hours of initiation or change in a medication

Usually 1-3 days (can be later) of exposure to a dopamine antagonist or withdrawal of a dopamine agonist

Autonomic

features

Tachypnoea

Hyperthermia (> 40°C)

Tachycardia

Hypertension

Diaphoresis

Hypersalivation

Tachypnoea

Hyperthermia (> 40°C)

Tachycardia

Hypertension

Diaphoresis

Hypersalivation

Neuromuscular

Increased tone, worse in the lower

extremities than upper extremities

Hyperreflexia

Clonus (unless masked by increased muscle tone) Dilated pupils

Classically agitation then coma

'Lead-pipe' rigidity globally  Rapid, increasing signs of extrapyramidal symptoms

Hyporeflexia

Normal pupils

Classically alert then coma

Treatment

Discontinue serotonergic agents

Benzodiazepines

Cyproheptadine

Supportive management 

Discontinue dopaminergic agents

Cooling

Fluids

Benzodiazipines

Dopamine agonists e.g. Bromocriptine or amantidine

Dantrolene

Supportive management  

Examiners Comments: 
 
Marks were allocated to descriptions of Precipitants and Risk factors, Clinical Features/Diagnosis and Management – the specific headings in the Table were not required. 
 
Many candidates lacked the basic knowledge to pass the question, and many did not complete it. Many confused Neuroleptic Malignant Syndrome with Malignant Hyperthermi
a. 
 

Discussion

As a "compare and contrast" question, this one would benefit from  a tabulated answer. The college table is of a sufficiently high quality that any attempt to "improve" on it would only lead to a messier more confusing answer model. As such, it would be completely consistent with the spirit of this revision resource.

Serotonin Syndrome vs.Neuroleptic Malignant Syndrome
  SS NMS
Causative agents Serotonin agonists or antagonists Dopamine antagonists or withdrawal of dopamine agonists
Onset Rapid (hours) Gradual (days)
Relationship to drug dose Usually overdose or the effect of using a combination of several agents Can occur with normal dosing, even after years of treatment with the same agent
Level of consciousness Agitation, hypervigilance, delirium Encephalopathy, stupour, coma, mutism
Pupils Dilated Normal
Other cranial nerves Usually unaffected Dysphagia, aspiration
Tone Increased Increased ("lead pipe")
Reflexes Increased Decreased
Clonus Present (a diagnostic discriminator) Absent
Temperature Raised Raised
Mucosa Siallorhoea Siallorhoea
Cardiovascular findings Tachycardia and hypertension Haemodynamically unstable, may be either high or low
Biochemistry Rhabdomyolysis; CK rise Rhabdomyolysis, CK rise
Low serum iron
Acid-base Normal Acidosis
Haematology May be normal Raised white cell count
Bowel sounds Vigorously hyperactive Reduced, sluggish
Management Cyproheptadine, olanzapine, chlorpromazine Amantadine, bromocryptine, dantrolene
     

References

Kateon, Hayley. "Differentiating serotonin syndrome and neuroleptic malignant syndrome." Mental Health Clinician 3.3 (2013): 129-133.

Nimmagadda, Seshagiri Rao, David Hugh Ryan, and Stephen Lawrence Atkin. "Neuroleptic malignant syndrome after venlafaxine." The Lancet 355.9200 (2000): 289-290.

Dunkley, E. J. C., et al. "The Hunter Serotonin Toxicity Criteria: simple and accurate diagnostic decision rules for serotonin toxicity." Qjm 96.9 (2003): 635-642.

Sternbach, Harvey. "The serotonin syndrome." The American journal of psychiatry 148.6 (1991): 705.

Lappin, Richard I., and Elizabeth L. Auchincloss. "Treatment of the serotonin syndrome with cyproheptadine.New England Journal of Medicine 331.15 (1994): 1021-1022.

Graudins, Andis, Andrew Stearman, and Betty Chan. "Treatment of the serotonin syndrome with cyproheptadine." Journal of Emergency Medicine 16.4 (1998): 615-619.

Gillman, P. K. "The serotonin syndrome and its treatment." Journal of Psychopharmacology 13.1 (1999): 100-109.

Jensen, Klaus. "The effect of antiserotonin (cyproheptadine) and antihistamine on cutaneous allergy." Allergy 15.4 (1960): 293-305.

Davis, John M., et al. "Electroconvulsive therapy in the treatment of the neuroleptic malignant syndrome." Convulsive therapy (1991).

Granato, Jerome E., et al. "Neuroleptic malignant syndrome: successful treatment with dantrolene and bromocriptine.Annals of neurology 14.1 (1983): 89-90.

Question 15.2 - 2019, Paper 1

A 25-year-old female is admitted with a reduced level of consciousness and suffers a brief seizure in the Emergency Department.

Her ECG is shown on page 14 (ECG 15.2).

a)    Describe the abnormalities.    (20% marks)

b)    What is the most likely diagnosis? What urgent treatment is required and what is the mechanism of action of the treatment?    (30% marks)

c)    Which drug would you avoid using to treat her convulsion and why?    (10% marks)
 

ECG 15.2

ECG-from-LITFL.jpg

College answer

a)
Broad QRS complex, first degree heart block, prolonged QT, dominant R wave in AVR

b)
Sodium bicarbonate is used to treat a suspected TCA overdose. Alkalinization increases the binding of TCA to plasma proteins reducing the amount of free drug and reduces the amount of ionisation of the drug reducing its ability to pass through cell membranes. Also reduces extracellular K concentration, causing hyperpolarisation and reducing the Na channel blockage.

c)
Phenytoin (Class 1b) should be avoided that it would potentiate sodium channel blockade
 

Discussion

Though it is impossible to guess which specific image the college used, one might be able to recapture their steps by googling "TCA overdose ECG" and picking out a top image result. That would probably end up being this classic ECG from LIFTL which was stolen and shamelessly re-posted here. The features of a TCA overdose listed there include: 

  • Sinus tachycardia with first-degree AV block (P waves hidden in the T waves, best seen in V1-2).
  • Broad QRS complexes.
  • Positive R’ wave in aVR.

Question 28.2 from the second paper of 2009 also asked specifically about "mechanism of effectiveness of sodium bicarbonate in the management of tricyclic antidepressant overdose." In summary, bicarbonate in TCA overdose works in the following ways:

  • Increased protein binding of TCAs in an alkaline bloodstream, thus decreasing the biologically active free fraction.
  • Increased availability of sodium in sodium bicarbonate, as a substrate for the voltage-gated channels.
  • Decreased binding of TCAs to the voltage-gated sodium channel
  • Correction of metabolic acidosis
  • Volume expansion because of the dilutional effect on TCA concentration
  • Cellular membrane hypopolarisation results from bicarbonate-induced intracellualr shift of potassium. 

The college focused on phenytoin in this scenario because the patient had a brief seizure, which brings up the question of antiepileptic choice. Phenytoin is such a good sodium channel blocker that it could be considered a Class I antiarrhythmic agent and a valid third line drug for refractory VT storm. What is more peculiar, is that of the currently available antiepileptic drugs, most either have a distince sodium channel blocker effect, or are thought to influence sodium channels in some other ambiguous round-about way. For instance, a 2017 paper by Brodie lists "phenytoin, carbamazepine, lamotrigine, oxcarbazepine, rufinamide, lacosamide and eslicarbazepine acetate" as classical sodium channel blockers. Sodium valproate, topiramate and zonisamide are also thought to have some sort of stabilising effect on sodium channels. 

Even more interesting is the fact that that historically phenytoin was proposed as the treatment to reverse cardiotoxicity due to TCA overdose. Hagerman & Hanashiro (1981) confessed to administering 50mg/min to five adults, to a total dose of about 5-7mg/kg. All conduction defects disappeared within about three quarters of an hour. So, in the 1980s, phenytoin was felt to inhibit the sodium channel blockade effects, rather than potentiating them. Even still, in 2010, Foianini et al recommended the use of Class I agents (lignocaine more so than phenytoin) in severe TCA cardiotoxicity. It appears that these drugs are indicated in cases where the cardiotoxicity is refractory to treatment with sodium bicarbonate or hypertonic saline, or in which these sodium-rich substances are contraindicated (metabolic alkalosis or hypernatremia).

Even more interestingly,

References

Hoffman, J. R., and C. R. McElroy. "Bicarbonate therapy for dysrhythmia and hypotension in tricyclic antidepressant overdose." Western Journal of Medicine134.1 (1981): 60.

Kerr, G. W., A. C. McGuffie, and S. Wilkie. "Tricyclic antidepressant overdose: a review." Emergency Medicine Journal 18.4 (2001): 236-241.

Brown, T. C., et al. "The use of sodium bicarbonate in the treatment of tricyclic antidepressant-induced arrhythmias." Anaesthesia and intensive care 1.3 (1973): 203-210.

McCabe, James L., et al. "Experimental tricyclic antidepressant toxicity: a randomized, controlled comparison of hypertonic saline solution, sodium bicarbonate, and hyperventilation." Annals of emergency medicine 32.3 (1998): 329-333.

Bou-Abboud, Elias, and Stanley Nattel. "Molecular mechanisms of the reversal of imipramine-induced sodium channel blockade by alkalinization in human cardiac myocytes." Cardiovascular research 38.2 (1998): 395-404.

Hoffman, Jerome R., et al. "Effect of hypertonic sodium bicarbonate in the treatment of moderate-to-severe cyclic antidepressant overdose." The American journal of emergency medicine 11.4 (1993): 336-341.

Dargan, Paul I., Mark G. Colbridge, and Alison L. Jones. "The management of tricyclic antidepressant poisoning." Toxicological reviews 24.3 (2005): 187-194.

Kingston, Michael E. "Hyperventilation in tricyclic antidepressant poisoning." Critical care medicine 7.12 (1979): 550-551.

Wrenn, Keith, Brian A. Smith, and Corey M. Slovis. "Profound alkalemia during treatment of tricyclic antidepressant overdose: a potential hazard of combined hyperventilation and intravenous bicarbonate." The American journal of emergency medicine 10.6 (1992): 553-555.

Brodie, Martin J. "Sodium channel blockers in the treatment of epilepsy." CNS drugs 31.7 (2017): 527-534.

Hagerman, Gordon A., and Paul K. Hanashiro. "Reversal of tricyclic-antidepressant-induced cardiac conduction abnormalities by phenytoin." Annals of emergency medicine10.2 (1981): 82-86.

Foianini, Anthony, Timothy Joseph Wiegand, and Neal Benowitz. "What is the role of lidocaine or phenytoin in tricyclic antidepressant-induced cardiotoxicity?.Clinical Toxicology48.4 (2010): 325-330.

Question 18.3 - 2019, Paper 1

What are the biochemical findings in methanol toxicity? Outline the specific management along with its physiological rationale.    (50% marks)

College answer

High anion gap metabolic acidosis, osmolar gap, elevated plasma methanol level.

Antidote therapy, often using ethanol or fomepizole, is directed towards delaying methanol metabolism until the methanol is eliminated from the patient’s system either naturally or via dialysis. Like methanol, ethanol is metabolized by ADH, but the enzyme’s affinity for ethanol is 10-20 times higher than it is for methanol. Fomepizole is also metabolized by ADH; however, its use is limited because of high cost and lack of availability

Dialysis: The toxic products of methanol and ethanol are formic acid and oxalic acid respectively. They are small molecules, are not protein bound and have low volume of distribution so are easily dialysable.

Folic acid – can accelerate the metabolism of formate via tetrahydrofolate.
 

Discussion

The characteristic features of toxic alcohol toxicity in general are:

  • High anion gap (all except isopropyl acohol)
  • High osmolar gap (all). Methanol is the alcohol molecule with the lowest molecular weight (32.04), and therefore a glass of methanol will raise the osmolar gap more than ethanol (MW= 46) or any of the others.
  • High toxic alcohol level is a fairly unimaginative biochemical feature to mention. By extension of the same concept, one may also list serum formaldehyde levels and serum formate levels. In case you're wondering, the upper range of normal formate levels is 0.4 mmol/L.

As for specific management:

Decontamination

  • Activated charcoal is useless. Absorption is too rapid.

Enhanced elimination

  • Haemodialysis: toxic alcohols and their metabolites are rapidly cleared in this manner
  • Folate and leucovorin enhance the clearance of formate; specifically formate binds with tetrahydrofolate to produce 10-formyl-tetrahydrofolate, which is then incorporated into purine metabolism (Morrow et al, 2015)
  • Alkalinization of urine with a bicarbonate infusion promotes dissociation of formic acid (it is less toxic in its ionised state) and improves its clearance by ion trapping in the urine

Specific antidotes

  • Alcohol -  the precise use of this substance in overdose is discussed in the chapter on ethylene glycol and its toxic acid metabolytes. 
  • In brief, one should sustain a blood ethanol concentration of 20 to 30 mmol/L (100 to 150 mg/dL) - this equates to a blood alcohol level of 0.1-0.15%.
  • Fomepizole as it is known, is basically a competitive antagonist to alcohol dehydrogenase. It does what ethanol would do, except it does so with great expense, and without ethanol intoxication. The advantage of using it is its lack of CNS effects - if the patient is confused already you do not want to add alcohol into the mix.

References

Morrow, Gregory P., et al. "In vivo kinetics of formate metabolism in folate-deficient and folate-replete rats." Journal of Biological Chemistry 290.4 (2015): 2244-2250.

Kraut, Jeffrey A., and Ira Kurtz. "Toxic alcohol ingestions: clinical features, diagnosis, and management." Clinical Journal of the American Society of Nephrology 3.1 (2008): 208-225.

Henderson, William R., and Jeffrey Brubacher. "Methanol and ethylene glycol poisoning: a case study and review of current literature." Cjem 4.1 (2002): 34-40.

Hovda, Knut Erik, Petter Urdal, and Dag Jacobsen. "Increased serum formate in the diagnosis of methanol poisoning." Journal of analytical toxicology 29.6 (2005): 586-588.

Question 30 - 2019, Paper 2

With respect to salicylate toxicity:

a)    List four severe complications.    (20% marks)

b)    List the associated haematological abnormalities.    (10% marks)

c)    List the options for enhancing salicylate removal and briefly explain the rationale for each option listed.    (50% marks)

d)    When assessing a patient with salicylate toxicity, how would you interpret a declining serum salicylate level?    (20% marks)
 

College answer

a)

List four severe complications

Pulmonary oedema

Cerebral oedema

Arrhythmias

Hyperpyrexia

Shock and cardiovascular collapse

Acid-base disturbance (high anion gap metabolic acidosis and respiratory alkalosis)

b)

List the associated haematological abnormalities Hypoprothrombinaemia

Thrombocytopaenia

c)

List the options for enhancing salicylate removal, and briefly outline the rational for each option listed:

Haemodialysis. Most of the drug is protein-bound, and is concentration dependant. The volume of distribution is small, and binding site saturation leads to large levels of free drug, which is easily dialyzable

Multiple-dose charcoal. Many aspirin forms are slow release and after ingestion they clump together in the GI tract, forming a large slow release preparation. It is also poorly soluble in the stomach leading to delayed absorption.

Forced alkaline diuresis. Renal excretion of salicylates becomes important when the metabolic pathways become saturated. There is a 10-20x increase in elimination when the urine pH increased from 5 to 8. Current role is questionable as haemodialysis is more efficient at removal, with less metabolic disturbance. Reasonable as initial therapy whilst waiting for circuit prime and line insertion.

d)

Give your interpretation of a declining serum salicylate level

It may indicate that the drug is moving into the tissues, and not necessarily being eliminated This means that clinical assessment is paramount

Discussion

This question is identical to Question 8 from the second paper of 2016, except that the wording of section (d) is slightly different for some reason.

a)

Salicylate toxicity has a whole list of complications:

  • pulmonary oedema
  • cerebral oedema
  • myocardial depression and shock
  • hypoglycaemia
  • seizures
  • haemorrhage from gastric ulceration
  • muscle rigidity leading to respiratory depression

c)

  • Raised PT: The classical coagulopathy which develops (asked about in the SAQs) is a prothrombin deficiency, leading to a prolonged PT and increased INR. According to UpToDate, this is because of hepatotoxicity and interference with the synthesis of vitamin K dependent factors. In addition to this, Question 8 from the second paper of 2016
  • Platelet dysfunction (due to COX enzyme inhibition)
  • Haemolytic anaemia (either by an autoimmune mechanism similar to that of methyldopa, or by oxidative damage as in G6PD - as per Sanford-Driscoll et al, 1986).

c)

Severe toxicity from salicylates has several treatment options:

Decontamination

  • Multiple dose activated charcoal is recommended by the UpToDate toxicology authors. Aspirin is well adsorbed by charcoal. Three 25g doses separated by two hours is the recommended regimen.
  • Whole bowel irrigation is relevant in the context of sustained-release preparations, and has been useful in animal models.

Direct  and indirect antidotes

  • There is nothing specific. Urinary alkalinisation is generally held to be the nearest thing to a direct antidote.

Enhancement of clearance

  • Alkalinise the urine. This is vital. An alkaline blood environment also prevents the movement of salicylate into the CSF.  Raising the urine pH from 5 to 8 can increase total salicylate excretion by twenty times.
  • Haemodialysis may be required in severe cases, particularly where you cannot give any more bicarbonate (i.e. the patient is already fluid overloaded) or where the overdose is supermassive (levels in excess of 100mg/dL). Even though salicylate is highly protein bound this technique can usually move enough molecules to make a difference. One must also keep in mind the nonlinear kinetics of elimination - the higher the dose, the longer the half-life, and therefore the more prominent the effects of extracorporeal clearance.
  • Multiple dose charcoal  as mentioned above

d) A declining salicylate level means nothing. Serial salicylate level measurement is meaningless, because:

  • It is highly protein bound, and the free fraction changes depending on the dose (as binding sites are saturated)- knowing the total level tells you nothing about the bioavailable fraction
  • It is poorly correlated with severity of intoxication (according to A.K.Done, 1960 - even the Done Nomogram has been largely abandoned because of this)
  • Acidosis causes the trapping of salicylate in the CNS, which would not be apparent from serum levels

Salicylate levels may be declining because

  • It is clearing renally or by hepatic metabolism
  • Absorption from a bezoar is diminishing
  • The intracellular uptake of salicylate has resulted in decreased serum levels

References

O'Malley, Gerald F. "Emergency department management of the salicylate-poisoned patient." Emergency medicine clinics of North America 25.2 (2007): 333-346.

Pinedo, H. M., L. B. van de Putte, and E. A. Loeliger. "Salicylate-induced consumption coagulopathy." Annals of the rheumatic diseases 32.1 (1973): 66.

Shapiro, Shepard, Milton H. Redish, and Harold A. Campbell. "Studies on Prothrombin: IV. The Prothrombinopenic Effect of Salicylate in Man."Experimental Biology and Medicine 53.2 (1943): 251-254.

Pearlman, Brian L., and Rashi Gambhir. "Salicylate Intoxication." Postgraduate medicine 121.4 (2009).

Rothschild, Bruce M. "Hematologic perturbations associated with salicylate." Clinical Pharmacology & Therapeutics 26.2 (1979): 145-152.

Sanford-Driscoll, Marcia, and Leroy C. Knodel. "Induction of hemolytic anemia by nonsteroidal antiinflammatory drugs." Annals of Pharmacotherapy 20.12 (1986): 925-934.

Mandelli, M., and G. Tognoni. "Monitoring plasma concentrations of salicylate." Clinical pharmacokinetics 5.5 (1980): 424-440.

Done, Alan K. "SALICYLATE INTOXICATION Significance of Measurements of Salicylate in Blood in Cases of Acute Ingestion." Pediatrics 26.5 (1960): 800-807.

Kashani, John, and Richard D. Shih. "Salicylate Overdose.Encyclopedia of Intensive Care Medicine (2012): 2011-2014.

Question 9 - 2020, Paper 1

A normally well 19-year-old female (65 kg) is admitted to your ICU after she had an intentional ingestion of 50 tablets of (her mother's) verapamil 180 mg (sustained release). The ingestion was 4 hours ago.

On admission, she is conscious, feels lightheaded, and has a heart rate of 40 beats/minute and a blood pressure of 90/40 mmHg.

Describe your management. Include in your answer how she is likely to deteriorate, and what general and specific therapies you would employ as her condition worsens.
 

College answer

Overarching Statement

This is a significant overdose of a non-dihydropyridine CCB, which would result in both vasodilatation and decreased inotropy/chronotropy. She already has symptomatic hypotension and bradycardia, which is likely to deteriorate and be prolonged due to the sustained release preparation ingested.

Immediate resuscitation –

  • early central access & likely to require intubation early
  • administration of IV crystalloid bolus for hypotension
  • atropine/glycopyrrolate for bradycardia
  • catecholamine support (adrenaline & noradrenaline); vasopressin

Gastrointestinal decontamination-

    • single dose activated charcoal (despite ingestion 4 hours ago) 1g/kg up to 50g- if deteriorating LOC would need intubation and NG insertion.
    • whole bowel irrigation- recommended as SR preparation.

Early contact with Poisons Information Centre (or equivalent) for advice.

Lipid “sink” therapy

  • IV Lipid emulsion (20% intralipid). Described in the context of lipid soluble poisons, including verapamil.
  • o (Bolus: 1.5ml/kg over 2 mins, Infusion: 1.5ml/kg.hr-1)

Specific therapies - Simultaneous rather than stepwise therapy in this case given severity of CCB poisoning.

  •  
  • Calcium- 10% Calcium chloride (10-20ml via CVC, followed by 0.25mmol/kg/hr, doses not expected). Monitor serum ionized calcium
  • Glucagon- useful as this patient is bradycardic (increases intracellular cAMP). 1-5mg IV push, repeat up to 15mg total. Hourly infusion based on bolus dose required to achieve response to bradycardia.
  • High Insulin Euglycaemic Therapy (HIET) -Has positive inotropic effects which is required in this case, overcomes relative insulin resistance created by CCBs.
    • Bolus- Insulin 1unit/kg IV with dextrose 25-50g, repeated to avoid hypoglycaemia, potassium supplements
    • Infusion- Insulin 1 units/kg/hr IV; titrate upwards every 30 mins until hypotension corrected or maximum does of 10 units/kg/hr reached
    • Dextrose- 0.5g/kg/hr; check every 30 mins and titrate to euglycaemia Potassium- ongoing supplementation
  • Methylene blue if unresponsive vasoplegia (need assessment of cardiac output)
  • Transvenous pacing for bradycardia
  • Mechanical circulatory support- V-A ECMO- maintains organ perfusion and can maintain perfusion pressure.

Marks were allocated more for specific management strategies than general resuscitation. Drug doses were not required.

Mention of Lipid Sink therapy essential to score greater than 4 marks

Discussion

Expected pattern of deterioration

  • This patient will, at some stage (soon), have a cardiac arrest if she is not treated appropriately. 50 × 180 = 9,000mg, which is a very high dose. For comparison, in a study of 65 case of sustained-release verapamil toxicity, the lowest dose associated with death was 4800mg and the highest dose associated with survival was 14,400mg.
  • The other clinical features to expect will be:
    • Common cardiovascular effects for all calcium channel blockers:
      • Hypotension
      • Prolonged PR interval
      • Heart blocks, usually 1st degree
    • Metabolic effects:
      • Hypoinsulinaemia (insulin release is regulated by calcium entry into islet beta cells via L-type channels)
      • Insulin resistance
      • Hyperglycaemia (in contrast to hypoglycaemia of beta-blocker overdose) is a marker of severity
      • Impaired cardiac fatty acid metabolism - CCBs force a switch to the use of carbohydrates
    • Other extracirculatory effects
      • Constipation
      • Hyperkalemia
      • Acute lung injury

Specific management

  • Decontamination, even though this is well past the usual 2 hour window, might still have a role to play because (judging by the fact that the patient is still not dead) complete absorption has not yet occurred. 1g/kg of activated charcoal should be the immediate treatment, followed by repeated 0.5g/kg doses if there is still evidence of ongoing absorption.
  • Direct and indirect antidotes:
    • Intravenous calcium is the direct antagonist, and classically you infuse these people full of calcium, but it may turn out to be remarkably ineffective. Generally speaking, people infuse about 0.2mmol/hr in order to avoid severe hypercalcemia.
    • High dose insulin euglycaemic therapy  seems promising, as animal studies have found ti to be superior to atropine, adrenaline, glucagon and calcium (Engebretsen et al, 2011).
  • Enhanced clearance
    • Haemoperfusion is the only recourse for this highly protein-bound drug
    • Lipid emulsion: among systematic reviews of intravenous lipid emulsion as a rescue therapy, verapamil is listed as an indication (Cave and Harvey, 2009).

General supportive management

  • Intubation is rarely indicated, as CCBs do not tend to cause coma, or even aspiration-inducing nausea for that matter.
  • Mechanical ventilation may be required if there is pulmonary oedema, but again this is rarely an issue.
  • Vasopressors and inotropes  may be useful in some cases, and from a mechanistic point of view it seems to make sense. However, usually there is little benefit. For instance, animal studies of nifedipine-poisoned pigs found that phenylephrine did not add anything to the effects of high-dose insulin (Engebretsen et al 2011).
  • Milrinone has been used in the past, but unfortunately it causes too much peripheral vasodilation to be useful.
  • Levosimendan, a calcium channel sensitiser, has been used to some effect in several case series (eg. Varpula et al, 2009)
  • Transvenous pacing may be possible, but the ventricle may not capture. Bradycardia, but not hypotension, can be managed in this way.
  • IABP has been used in cases where nothing you do seems to help, and particularly in case where there has been a beta blocker co-ingestion (in one case report from 2009, the authors were unaware of the CCB poisoning story until well into the course of treatment for an unexplained complete heart block and cardiogenic shock).
  • ECMO may be the only answer to a complete failure of the circulation.

References

Barrow, P. M., P. L. Houston, and D. T. Wong. "Overdose of sustained-release verapamil." BJA: British Journal of Anaesthesia 72.3 (1994): 361-365.

Mégarbane, Bruno, et al. "Predictors of mortality in verapamil overdose: usefulness of serum verapamil concentrations." Basic & clinical pharmacology & toxicology 108.6 (2011): 385-389.

Henry, Philip D. "Comparative pharmacology of calcium antagonists: nifedipine, verapamil and diltiazem." The American journal of cardiology 46.6 (1980): 1047-1058.

Doyon, Suzanne, and James R. Roberts. "The use of glucagon in a case of calcium channel blocker overdose." Annals of emergency medicine 22.7 (1993): 1229-1233.

Isbister, G. K. "Delayed asystolic cardiac arrest after diltiazem overdose; resuscitation with high dose intravenous calcium." Emergency medicine journal 19.4 (2002): 355-357.

Proano, Larry, William K. Chiang, and Richard Y. Wang. "Calcium channel blocker overdose." The American journal of emergency medicine 13.4 (1995): 444-450.

Engebretsen, Kristin M., et al. "High-dose insulin therapy in beta-blocker and calcium channel-blocker poisoning." Clinical toxicology 49.4 (2011).

Varpula, Tero, et al. "Treatment of serious calcium channel blocker overdose with levosimendan, a calcium sensitizer." Anesthesia & Analgesia 108.3 (2009): 790-792.

Frierson, John, et al. "Refractory cardiogenic shock and complete heart block after unsuspected verapamil‐sr and atenolol overdose." Clinical cardiology 14.11 (1991): 933-935.

Garg, Suneel K., et al. "Management of life-threatening calcium channel blocker overdose with continuous veno-venous hemodiafiltration with charcoal hemoperfusion." Indian journal of critical care medicine: peer-reviewed, official publication of Indian Society of Critical Care Medicine 18.6 (2014): 399.

Doepker, Bruce, et al. "High-dose insulin and intravenous lipid emulsion therapy for cardiogenic shock induced by intentional calcium-channel blocker and beta-blocker overdose: a case series." The Journal of emergency medicine 46.4 (2014): 486-490.

Cave, Grant, and Martyn Harvey. "Intravenous lipid emulsion as antidote beyond local anesthetic toxicity: a systematic review." Academic Emergency Medicine 16.9 (2009): 815-824.

Question 27 - 2020, Paper 1

Outline how the pathophysiological changes in septic shock affect the pharmacokinetlcs and pharmacodynamics of commonly used antimicrobials.

College answer

The major changes in pharmacokinetic parameters of critically ill patients include alterations in volume of distribution (Vd) and clearance (Cl). Subsequently, these alterations affect the concentrations of antimicrobials in the body and the extent to which they are cleared.

The Vd is the volume in which the total amount of drug would have to be evenly distributed in to equal the same concentration as in the plasma. The toxins produced by various bacteria often lead to endothelial damage and result in increased capillary permeability. This leads to the phenomenon of “third spacing” where fluid shifts into the interstitial space from the intravascular space. These fluid shifts will increase the Vd of hydrophilic antimicrobials. Generally speaking, hydrophilic antimicrobials have a low Vd and therefore are greatly affected by these fluid shifts. Since lipophilic antimicrobials have a larger Vd, they typically distribute further into tissues and are less affected by these fluid shifts. Patients in the ICU often have hypotension as a result of septic shock, which requires the administration of fluid boluses. Additionally, heart failure and renal failure lead to more oedematous states where patients can retain large amounts of fluid. These situations also lead to increases in Vd of hydrophilic drugs.

Changes in protein binding can also have a substantial effect on the Vd, especially for drugs that are highly protein bound. Only unbound or free drug is microbiologically active. Hypoalbuminemia in critically ill patients can result in decreased binding of drugs and subsequently higher free concentrations of drugs. While free drug will distribute into tissues, critically ill patients often have greater amounts of fluid in the interstitial space causing the antimicrobial concentrations in the tissues to remain low.

The administration of large volumes of fluid and use of vasopressors leads to a hypermetabolic state in which cardiac output and glomerular filtration rate are increased. The term often used to describe this enhanced elimination is augmented renal clearance. These physiological changes affect the clearance of drugs and can lead to sub-therapeutic levels of antimicrobials that are typically cleared by the kidneys. In contrast, decreased organ perfusion in the presence of end organ damage can lead to kidney and/or liver failure in which concentrations of these antimicrobials would be increased. Inadequate clearance or metabolism of these drugs would lead to accumulation and potential toxicity. Typically, equations such as Cockroft-Gault are used to estimate renal function; however, these are often not good predictors of renal function in critically ill patients due to the acute and rapid changes such patients often experience. Since many antimicrobials are dosed based on renal function it is even more challenging to ensure adequate doses are being administered. The most accurate way to calculate renal function is the use of 8- or 12-hour creatinine collections. In situations where renal replacement therapy is utilized, careful consideration of timing and supplemental dosing post-dialysis would be needed depending on the antimicrobial agent

Marks were awarded to answers which dealt with the following:

  1. Effects of fluid shifts on volume of distribution
  2. Effects on protein binding
  3. Effects on organ function leading to an increase or decrease in clearance.

Discussion

This question is identical to Question 10 from the second paper of 2015. An SEO-wrecking cut and paste is therefore permissible here, considering especially that the college reproduced their comments verbatim. In essence, just like last time, the examiners ask about septic shock, but then go on to discuss pharmacological changes which are common to all critically ill patients.

These are as follows:

Pharmacokinetic changes:

  • Factors which decrease the antibiotic peak concentration:
    • Suboptimal gut absorption.
    • Increased volume of distribution (patients are typically fluid-overloaded)
    • Poor penetration to the site of action (poor tissue perfusion and generalised oedema)
  • Factors which increase the antibiotic peak concentration
    • Increased free fraction (decreased protein binding due to low albumin)
    • Diminished clearance (renal and hepatic failure)
  • Factors which increase the antibiotic half-life
    • Diminished clearance (renal and hepatic failure)
  • Factors which decrease the antibiotic half-life
    • Renal replacement therapy (enhances clearance)
    • Increased hepatic clearance (hyperdynamic circulation)
    • Increased glomerular filtration rate (hyperdynamic circulation)
    • Increased rate of drug metabolism due to a "hypermetabolic state" induced by trauma,  burns and exogenous catecholamine infusions

Pharmacodynamic changes:

  • Increased nephrotoxicity from aminoglycosides, if the renal function is already impaired
  • Increased cardiotoxicity from bleomycin and vancomycin
  • Increased risk of QT prolongation and arrhythmia with fluoroquinolones in the context of cardiac ischaemia, profound hypothermia, or extreme electrolyte derangement
  • Increased bone marrow toxicity from linezolid, cotrimoxazole, gancyclovir, chloramphenicol, beta-lactams of all sorts...
  • With a disrupted blood-brain barrier, an increased risk of seizures from high-dose beta-lactams, due to enhanced penetration.  
  • Worsening shock due to dapsone-induced methaemoglobinaemia and thus diminished oxygen-carrying capacity.

As such, this model answer to a question about sepsis would have also answered Question 1 from the first paper of 2000, which asks about pharmacological changes in critical illness in a broader sense. There are a few pharmacological peculiarities which develop exclusively (or almost exclusively) in the context of sepsis, and these are summarised below using the excellent article by De Paepe et al (2002)

Change to pharmacology which are unique to sepsis and septic shock

  • Increased volume of distribution due to sepsis-associated "capillary leak" results in a decreased effective concentration of antimicrobials.
  • Decreased bioavailability of basic drugs: because α-1-acid glycoprotein is an acute phase reactant
  • Increased penetration of formerly impenetrable tissues due to their inflamed state, as in the enhanced penetration of β-lactams into the CNS which is associated with meningitis
  • Impaired hepatic metabolism due to inhibition of CYP-450 enzymes by  endotoxin-mediated release of nitric oxide

References

Craig, William A. "Pharmacokinetic/pharmacodynamic parameters: rationale for antibacterial dosing of mice and men." Clinical infectious diseases (1998): 1-10.

Ulldemolins, Marta, et al. "Antibiotic dosing in multiple organ dysfunction syndrome." CHEST Journal 139.5 (2011): 1210-1220.

Trotman, Robin L., et al. "Antibiotic dosing in critically ill adult patients receiving continuous renal replacement therapy." Clinical infectious diseases 41.8 (2005): 1159-1166.

Drusano, George L. "Antimicrobial pharmacodynamics: critical interactions of'bug and drug'." Nature Reviews Microbiology 2.4 (2004): 289-300.

De Paepe, Peter, Frans M. Belpaire, and Walter A. Buylaert. "Pharmacokinetic and pharmacodynamic considerations when treating patients with sepsis and septic shock." Clinical pharmacokinetics 41.14 (2002): 1135-1151.

Piafsky, Kenneth M., et al. "Increased plasma protein binding of propranolol and chlorpromazine mediated by disease-induced elevations of plasma α1 acid glycoprotein.New England Journal of Medicine 299.26 (1978): 1435-1439.

Muller, Claudia M., et al. "Nitric oxide mediates hepatic cytochrome P450 dysfunction induced by endotoxin." The Journal of the American Society of Anesthesiologists 84.6 (1996): 1435-1442.

Question 3 - 2022, Paper 1

List the one pharmacological intervention for each of the following medications in the context of toxic ingestion leading to haemodynamic collapse. Outline the rationale for use of the pharmacological intervention including the mechanism of action. 

a) Digoxin (25% marks)

b) Tricyclic anti-depressants (25% marks)

c) Beta blockers (25% marks)

d) Lignocaine (25% marks)

College answer

Not available.

Discussion

This is not a new theme, but a new way of asking about the same (important) theme, a laudable development from the viewpoint of good assessment design. Beta blocker overdose digoxin toxicity and  tricyclic antidepressant overdose are covered in detail elsewhere, and local anaesthetic toxicity is one of the syllabus items from the First Part exam, but it is good to see it migrate into the Fellowship papers because - let's face it - a first year ICU trainee is not going to be left to manage local anaesthetic toxicity with "haemodynamic collapse" on their own, i.e. one might argue that these topics belong in a senior curriculum.

Digoxin: Digoxin-specific Fab fragments are used as a "pharmacological intervention", and the article in UpToDate recommends that digoxin antibodies be used in every poisoning, even those who do not present with "haemodynamic collapse". Incidentally, that's obviously a colloquialism without any sort of a precise AHA/ESC definition, but we can let that slide because most intensivists will intuitively grasp it and relate. For example, in the case of digoxin, "haemodynamic collapse" looks like life-threatening arrhythmias and bradycardia.

Rationale for digoxin-specific Fab fragments in digoxin overdose:

  • Remove free digoxin from the active target sites: the Fab has a 100 – 1000 times higher affinity for digoxin than does Na+/K+ ATPase.
  • Increase removal from tissues: The circulating Fab acts as a digoxin sink, increasing the gradient for free digoxin to enter the circulation; this increases the renal clearance of digoxin by 20-30% (Chan and Buckley, 2014).
  • Increase renal clearance: Digoxin/Fab complexes are removed by both renal clearance and hepatic metabolism, but it's mainly renal: the digoxin-antibody complexes are filtered through the glomeruli  (which is surprising, consider their size) and reabsorbed in the proximal tubules while the digoxin is excreted. 
  • The serum digoxin assay will thereafter measure both the free drug and the Fab-bound fraction, and is therefore not to be believed.

Rationale for sodium bicarbonate in tricyclic antidepressant overdose:

  • Increase protein binding of TCAs in an alkaline bloodstream, thus decreasing the biologically active free fraction.
  • Increase the availability of sodium in sodium bicarbonate, as a substrate for the voltage-gated channels. (this corrects the QRS prolongation and prevents arrhythmias)
  • Decreased binding of TCAs to the voltage-gated sodium channel - apparently this binding is affected by subtle changes in pH, and this receptor family has a greater affinity for TCAs at acidic pH. 
  • Correction of metabolic acidosis  to enhance cardiac contractility by improving catecholamine sensitivity
  • Volume expansion (dilutes TCA concentration)
  • Cellular membrane hypopolarisation results from the bicarbonate-induced intracellular shift of potassium. Apparently, this somehow "decreases sodium channel blockade by voltage-dependent drug-binding changes".

Rationale for high dose insulin euglycaemic therapy in beta-blocker overdose:

  • Inotropic effect: Insulin is a potent positive inotrope in high doses because of its effects on various calcium-handling pathways, particularly those mediated by PI3K (Engebretsen et al, 2011).
  • Afterload reducing effect: Insulin produces vasodilation, which improves local microcirculation (due to enhancement of endothelial nitric oxide synthase activity) - apparently this can "achieve perfused capillary density similar to that of exercising muscle
  • Metabolic effect: Insulin assists myocardial uptake of carbohydrates, which is the preferred fuel substrate of the heart under stressed conditions (whereas normally free fatty acids are preferred).

Rationale for lipid infusion in local anaesthetic toxicity:

  • Lipid sink: the highly lipid-soluble local anaesthetic molecules are absorbed into the lipid emulsion droplets, which decreases the free fraction of the drug in the circulation
  • Tissue extraction: because the free fraction in the circulation drops, redistribution from target tissues (CNS, myocardium) will occur, reducing toxicity in those organs
  • Lipid shuttle: the fatty droplets of lipid emulsion act as a carrier which delivers the local anaesthetic to the liver, enhancing the rate of elimination (apparently this is also referred to as a "lipid subway")
  • Metabolic changes in the myocardium:  the increased fatty acid supply reverses local-anaesthetic-induced reduction in fatty acid metabolism in the cardiac mitochondria
  • Inoconstrictor effects though the inhibition of nitric oxide release and some positive inotropic effects, which appears to be an intrinsic property of the lipid emulsion
  • Reversal of cardiac sodium channel blockade by a mechanism apparently related to fatty acid-mediated modulation of cardiac sodium channels

References

UpToDate has a nice article about digoxin toxicity..

Williamson, Kristin M., et al. "Digoxin toxicity: an evaluation in current clinical practice." Archives of internal medicine 158.22 (1998): 2444-2449.

Chan, B. S. H., and N. A. Buckley. "Digoxin-specific antibody fragments in the treatment of digoxin toxicity." Clinical Toxicology 52.8 (2014): 824-836.

Bou-Abboud, Elias, and Stanley Nattel. "Molecular mechanisms of the reversal of imipramine-induced sodium channel blockade by alkalinization in human cardiac myocytes." Cardiovascular research 38.2 (1998): 395-404.

Hoffman, Jerome R., et al. "Effect of hypertonic sodium bicarbonate in the treatment of moderate-to-severe cyclic antidepressant overdose." The American journal of emergency medicine 11.4 (1993): 336-341.

Engebretsen, Kristin M., et al. "High-dose insulin therapy in beta-blocker and calcium channel-blocker poisoning." Clinical toxicology (2011).

Christie, Linsey E., John Picard, and Guy L. Weinberg. "Local anaesthetic systemic toxicity." Bja Education 15.3 (2015): 136-142.

Ok, Seong-Ho, et al. "Lipid emulsion for treating local anesthetic systemic toxicity." International journal of medical sciences 15.7 (2018): 713.